SlideShare a Scribd company logo
1 of 117
Download to read offline
1 
CÁC BÀI TOÁN CÓ NHIỀU LỜI GIẢI 
Võ Quốc Bá Cẩn 
Bài 1. (Chọn đội tuyển Rumani 1999) Cho 1 2 ,, , n aa a là các số thực 
dương thỏa mãn 1 2 1. n a aa Chứng minh rằng 
1 2 
1 1 
1 1 
1 
1. 
a n a n an n 1 
Chứng minh. 
Cách 1. Nhân cả hai vế của bất đẳng thức cho 1 n và để ý rằng 
1 
1 , 
1 1 
i 
i i 
n a 
a n a n 
ta viết được nó dưới dạng 
1 2 
1 2 
1. 
1 1 1 
n 
n 
a a a 
a n a n a n 
Tới đây, sử dụng bất đẳng thức Cauchy-Schwarz 
1 2 
1 2 
2 
1 2 
1 2 
1 1 1 
, 
( 1) ( 1) ( 1) 
n 
n 
n 
n 
a a a 
a n a n a n 
a a 
a n a n 
a 
a n 
ta đưa được bài toán về chứng minh 
2 
1 2 1 2 ( 1). n n a a a a a a n n 
Do 
2 
1 2 
1 1 
2 
n 
n i j 
i j n 
i 
i 
a a a a a a nên bất đẳng thức 
trên tương đương với 
1 
( 1) 
. 
i j 2 
i j n 
n n 
a a 
Bất đẳng thức này đúng theo bất đẳng thức AM-GM
2 
( 1) 
2 
1 1 
( 1) ( 1) 
. 
2 2 
n n 
i j i j 
i j n i j n 
n n n n 
a a a a 
Bài toán được chứng minh xong. 
Đẳng thức xảy ra khi và chỉ khi 1 2 1. n a a a 
Cách 2. Giả sử bất đẳng thức đã cho sai. Khi đó tồn tại các số thực dương 
1 2 ,, , n aa a thỏa mãn 1 2 1 n aa a sao cho 
1 2 
1 1 
1 1 
1 
1, 
1 n a n a n a n 
hay 
1 2 
1 2 
1. 
1 1 1 
n 
n 
a a a 
a n a n a n 
Từ đây sử dụng bất đẳng thức AM-GM, ta có 
1 2 
1 2 3 
2 3 
3 
1 
1 
1 
1 
1 1 2 3 
1 
1 
1 1 1 1 
( 1) 
( 1)( 1) ( 1) 
1 
( 1) . 
( 1 
1 
) 
n 
n 
n n 
n 
n n 
i 
i 
a 
a n 
a a a 
n a a a 
a n a n a n 
n 
a n a n a n 
a n 
n 
a n 
a n 
a 
Suy ra 
1 
1 
1 
1 
( 1 
1. 
( 1 
) 
) 
i 
n 
n 
i a 
a 
a 
n 
n 
Tương tự ta cũng có 
2 3 
1 1 1 
2 3 
1 1, 1 1, 1 1. 
( 1) ( 
( 1) 
1) ( 1 
( 1) ( 
, 
) 
1) n 
n n n 
n n n 
i i i 
i i i 
n 
a a a 
a n a n a n 
a n a n a n 
Nhân các bất đẳng thức này lại theo vế ta thu được 1 1 (vô lý). Mâu thuẫn 
này chứng tỏ điều giả sử ở trên là sai, hay nói một cách khác, bài toán được 
chứng minh. 
Cách 3. Thực hiện biến đổi tương tự như cách 1, ta phải chứng minh
3 
1 2 
1 2 
1. 
1 1 1 
n 
n 
a a a 
a n a n a n 
Sử dụng bất đẳng thức AM-GM, ta có 
1 
1 1 1 
1 
1 1 1 2 
1 2 3 
1 
1 
1 1 1 1 
1 
1 2 3 
1 
1 
1 1 1 1 
1 2 3 
1 ( 1) ( 1) 
( 1) 
. 
n 
n 
n 
n 
n n 
n 
n 
n n n n 
n n n n n 
n 
n 
n 
n n n n 
n n 
n 
n 
n n 
n 
a a a 
a n a n a a a n a a 
a 
a n a a a 
a 
a a a a 
a a 
Tương tự 
1 
2 2 
1 1 1 
2 
1 2 
1 
3 3 
1 1 1 
3 
1 2 
1 
1 1 1 
1 2 
, 
1 
, 
. 
1 
, 
1 
n 
n 
n n n 
n n n 
n 
n 
n 
n n n 
n n n 
n 
n 
n 
n n 
n n n 
n n n n 
n 
a a 
a n 
a a a 
a a 
a n 
a a a 
a a 
a n 
a a a 
Cộng n bất đẳng thức trên lại theo vế, ta có ngay điều phải chứng minh. 
Cách 4. Ta sẽ chứng minh mệnh đề tổng quát như sau: Nếu 1 2 , , , n a a a là 
các số thực dương thỏa mãn 1 2 1 n a a a và 1, n m n thì 
1 2 
1 1 1 
. 
1 n n n n n a m a m 
n 
a m m 
Kết quả bài toán ứng với trường hợp 1. n m n 
Chứng minh. Ta sử dụng phương pháp quy nạp Toán học.
4 
Với 2, n bất đẳng thức trở thành 
1 2 2 2 2 
1 1 
, 
1 
n 
a m a m m 
đúng vì 
1 
1 2 2 2 2 1 2 2 1 2 
2 
1 2 
1 2 2 1 2 
1 1 2 1 2 
1 1 1 
( 1) ( 1) 
0. 
( )(1 )(1 ) 
a 
a m a m m a m m a m 
a m 
a m m a m 
Giả sử bất đẳng thức đúng với 2.nk Ta sẽ chứng minh nó cũng đúng 
với 1,nk tức là: Nếu 1 2 1 , , , , k k a a a a là các số thực dương thỏa 
mãn 1 2 1 1 k k a a a a và 1 , k m k thì 
1 1 2 1 1 1 1 1 
1 1 1 1 1 
. 
1 k k k k k k k 
k 
a m a m a m a m m 
Không mất tính tổng quát giả sử 1 2 1, k k a a a a suy ra 1 1. k a 
Đặt 1 
1 2 
1 
1 1, 2, , , 
1 
k , i , k . 
k i k 
k 
k 
a i k 
a m 
b 
b 
a a b m 
a b 
Khi đó ta có 1 2 1 k bb b và 1 1 k k m m k k nên theo giả thiết quy 
nạp, ta sẽ có 
1 2 
1 1 
. 
1 
1 
k k k k k b m b m 
k 
b m m 
Thay i 
i 
a 
b 
b 
và k 1 
k 
m 
m 
b 
vào rồi rút gọn, ta thu được ngay 
1 1 2 1 1 1 
1 1 
. 
1 
k k k k k 
k 
a m a m a m b m 
Từ đánh giá này suy ra ta chỉ cần chứng minh (chú ý 1 
1 
k k 
a 
b 
) 
1 1 1 
1 
. 
1 1 
k 
k 
k k k 
k b k 
b m m b m 
Bất đẳng thức này tương đương với
5 
1 1 
1 1 
1 1 
1 
1 1 
(1 ) (1 ) 
1 , 
1 
1 (1 ) 
, 
1 
(1 ) ( )(1 ) (1 ) 0, 
k 
k k 
k 
k k 
k 
k 
k k 
k k 
k k 
b m k m 
k 
m b b m 
b k b 
m b b m 
b b m b b k m b 
1 2 
1 
2 1 1 
1 
(1 ) (1 ) 0, 
(1 ) (1 2 ) ( 1) 0. 
k k k 
k 
k k 
k 
b m b b kb b b b k 
b m b kb k k b b 
Bất đẳng thức cuối cùng đúng do 
1 1 1 
1 
1 
( 1) (1 ) (1 2 ) 
(1 2 ). 
k k k 
k 
k 
k k b b k b b k b kb 
m b kb 
Như vậy, ta đã chứng minh được bất đẳng thức cũng đúng với n k 1. 
Theo nguyên lý quy nạp, ta có nó đúng với mọi 2. n 
Bài 2. Cho a, b, c, d là các số thực dương thỏa mãn abcd 1. Chứng 
minh bất đẳng thức sau 
2 2 2 2 
1 1 1 1 
1. 
(1 a) (1 b) (1 c) (1 d) 
Chứng minh. 
Cách 1. Trước hết ta chứng minh bổ đề sau. 
Bổ đề. Nếu , xy là các số thực dương, thì 
2 2 
1 1 1 
. 
(1 x) (1 y) 1 xy 
Chứng minh. Cách 1. Sử dụng bất đẳng thức Cauchy-Schwarz, ta có 
2 ( )(1 ) 
(1 ) 1 (1 ) , 
x x y xy 
x xy 
y y 
suy ra 
2 
1 
. 
(1 ) ( )(1 ) 
y 
x x y xy 
Tương tự ta cũng có
6 
2 
1 
. 
(1 ) ( )(1 ) 
x 
y x y xy 
Cộng hai bất đẳng thức này lại, ta được 
2 2 
1 1 1 
. 
(1 ) (1 ) ( )(1 ) 1 
y x 
x y x y xy xy 
Bổ đề được chứng minh. 
Cách 2. Tiến hành biến đổi trực tiếp, ta có 
2 2 2 2 
2 2 2 2 
2 2 
2 2 
1 1 1 ( ) 2 1 
(1 ) (1 ) 1 (1 ) (1 ) (1 ) 
( ) ( 1) 
0. 
(1 ) (1 ) (1 ) 
xy x y x y xy 
x y xy x y xy 
xy x y xy 
x y xy 
Trở lại bài toán. Sử dụng bổ đề trên ta được 
2 2 
2 2 
1 1 1 
, 
(1 ) (1 ) 1 
1 1 1 1 
. 
(1 ) (1 ) 1 1 1 
1 
a b ab 
ab 
c d cd ab 
ab 
Từ đó suy ra 
2 2 2 2 
1 1 1 1 1 
1. 
(1 ) (1 ) (1 ) (1 ) 1 
ab 
a b c d ab 
Đẳng thức xảy ra khi và chỉ khi a b c d 1. 
Cách 2. Đặt 
5 4 2 3 5 3 2 5 2 4 
1 1 1 
x , y , z 
a b c ab c a bc 
và 1, t ta dễ dàng 
kiểm tra được 2 2 2 2 
, , , . 
yz zt tx xy 
a b c d 
x y z t 
Thay bất đẳng thức đã 
cho, ta viết được nó lại thành 
4 4 4 4 
2 2 2 2 2 2 2 2 
1. 
( ) ( ) ( ) ( ) 
x y z t 
x yz y zt z tx t xy 
Sử dụng bất đẳng thức Cauchy-Schwarz, ta được
7 
4 4 4 4 
2 2 2 2 2 2 2 2 
2 2 2 2 2 
2 2 2 2 2 2 2 2 
( ) ( ) ( ) ( ) 
( ) 
. 
( ) ( ) ( ) ( ) 
x y z t 
x yz y zt z tx t xy 
x y z t 
x yz y zt z tx t xy 
Mặt khác 
2 2 2 2 2 2 2 2 2 2 2 2 
2 2 2 2 2 2 
2 2 2 2 2 2 2 2 2 2 2 2 
2 2 2 2 2 2 
( ) ( ) ( )( ) ( )( ) 
( )( ), 
( ) ( ) ( )( ) ( )( ) 
( )( ). 
x yz z tx x y x z z t z x 
x y z t x z 
y zt t xy y z y t t x t y 
x y z t y t 
Kết hợp với trên, ta suy ra 
4 4 4 4 
2 2 2 2 2 2 2 2 
2 2 2 2 2 
2 2 2 2 2 2 2 2 2 2 2 2 
( ) ( ) ( ) ( ) 
( ) 
1. 
( )( ) ( )( ) 
x y z t 
x yz y zt z tx t xy 
x y z t 
x y z t x z x y z t y t 
Cách 3. Đặt x 1, y a, z ab và t abc, ta dễ thấy , , 
y z 
a b 
x y 
t 
c 
z 
và . 
x 
d 
t 
Thay vào bất đẳng thức đã cho, ta được 
2 2 2 2 
2 2 2 2 
1. 
( ) ( ) ( ) ( ) 
x y z t 
x y y z z t t x 
Gọi P là vế trái của bất đẳng thức trên. Sử dụng lần lượt các bất đẳng thức 
Cauchy-Schwarz và AM-GM, ta có 
2 
2 2 2 2 2 2 2 2 
2 2 2 2 2 
2 2 2 2 
2 
2 2 2 2 
2 
( ) ( ) ( ) ( ) 
( ) ( ) ( ) ( ) ( ) ( ) ( ) ( ) 
( ) 
( ) ( ) ( ) ( ) 
( ) ( ) ( ) ( ) 
4 ( ) ( ) 
x x t y y x z z y t t z 
P 
x y x t y z y x z t z y t x t z 
x y z t xy yz zt tx 
x y z t x t y z 
x y z t x t y z 
x y z t 2 2 2 
1. 
(x t) (y z)
8 
Cách 4. Đặt 4 4 4 4 
1 1 1 1 
a , b , c , d 
x y z t 
( , , , 0) x y z t thì ta cũng 
có xyzt 1. Khi đó 
8 8 6 
2 4 2 4 2 3 2 
1 
. 
(1 ) ( 1) ( ) ( ) 
x x x 
a x x xyzt x yzt 
Suy ra bất đẳng thức cần chứng minh có thể viết lại thành 
6 6 6 6 
3 2 3 2 3 2 3 2 
1. 
( ) ( ) ( ) ( ) 
x y z t 
x yzt y ztx z txy t xyz 
Tới đây, sử dụng bất đẳng thức Cauchy-Schwarz, ta đưa được bài toán về 
chứng minh 
(x3 y3 z3 t3)2 (x3 yzt)2 (y3 ztx)2 (z3 txy)2 (t3 xyz)2. 
Thực hiện khai triển và rút gọn, ta được 
2 3 3 2 3 2 2 2. 
sym 
x y x yzt y z t 
Bất đẳng thức này đúng vì theo bất đẳng thức AM-GM, ta có 
3 3 3 
3 3 3 3 
3 3 3 3 3 3 
2 2 2 3 3 
4 
2 2 , 
3 3 
2 
. 
3 3 
sym 
sym 
y z t 
x yzt x x y 
y z z t t y 
y z t x y 
Phép chứng minh hoàn tất. 
Bài 3. (Olympic Toán Ukraine 2001) Chứng minh rằng với mọi số thực 
dương a, b, c, x, y, z, bất đẳng thức sau đây luôn được thỏa mãn 
(ay az bz bx cx cy)2 4(ab bc ca)(xy yz zx). 
Chứng minh. 
Cách 1. Áp dụng bất đẳng thức AM-GM, ta có 
2 
( ) ( ) 
4 ( ) ( ) 
( ) ( ) 
( ) ( ) . 
bc y z yz b c 
VP a y z x b c 
y z y z 
bc y z yz b c 
a y z x b c 
y z y z 
Do đó, để chứng minh bất đẳng thức đã cho, ta chỉ cần chứng minh được
9 
( ) ( ) 
( ) ( ) . 
bc y z yz b c 
ay az bz bx cx cy a y z x b c 
y z y z 
Bất đẳng thức này tương đương với 
( ) ( ) 
, 
bc y z yz b c 
bz cy 
b c y z 
2 2 ( ) 
. 
b z c y yz b c 
b c y z 
Tới đây, sử dụng bất đẳng thức Cauchy-Schwarz, ta được 
2 2 2 1 1 
(b z c y) (b c) . 
z y 
Từ đó suy ra 
2 2 ( ) 
. 
1 1 
b z c y b c yz b c 
b c y z 
z y 
Phép chứng minh được hoàn tất. 
Cách 2. Lấy căn bậc hai của hai vế và chú ý rằng 
ay az bz bx cx cy (a b c)(x y z) (ax by cz), 
ta có thể viết lại bất đẳng thức cần chứng minh dưới dạng 
ax by cz 2 (ab bc ca)(xy yz zx) (a b c)(x y z). 
Đến đây, sử dụng bất đẳng thức Cauchy-Schwarz, ta có 
2 2 2 2 2 2 
2 2 2 2 2 2 
( )( ) 2( ) 2( ) 
[ 2( )][ 2( )] 
( )( ) . 
VT a b c x y z ab bc ca xy yz zx 
a b c ab bc ca x y z xy yz zx 
a b c x y z VP 
Cách 3. Tương tự như cách 2, ta phải chứng minh rằng 
ax by cz 2 (ab bc ca)(xy yz zx) (a b c)(x y z). 
Để ý rằng đây là một bất đẳng thức thuần nhất cho các bộ số (a, b, c) và 
(x, y, z). Vì thế, không giảm đi tính tổng quát, ta hoàn toàn có thể giả sử 
rằng a b c x y z 1. Lúc này, áp dụng bất đẳng thức AM-GM, 
ta có
10 
2 2 2 2 2 2 
2 2 2 2 2 2 
2 2 
( ) ( ) 
2 2 2 
2( ) 2( ) 
2 2 
( ) ( ) 
1 ( )( ). 
2 2 
a x b y c z 
VT ab bc ca xy yz zx 
a b c ab bc ca x y z xy yz zx 
a b c x y z 
a b c x y z 
Bài 4. (IMO 2008) Cho ,, 1 x y z và 1. xyz Chứng minh rằng 
2 2 2 
1. 
1 1 1 
x y z 
x y z 
Chứng minh. 
Cách 1. Đặt , , , 
1 1 1 
x y z 
a b c 
x y z 
khi đó ta có 
, , . 
1 1 1 
a b c 
x y z 
a b c 
Do giả thiết 1xyz nên ( 1)( 1)( 1), abc a b c suy ra 
a b c ab bc ca 1. 
Bất đẳng thức cần chứng minh được đưa về a2 b2 c2 1. Để ý rằng 
1 2 1 2(a b c ab bc ca) 1, 
ta có thể viết lại bất đẳng thức trên dưới dạng 
a2 b2 c2 2(a b c ab bc ca) 1. 
Không mấy khó khăn, ta có thể phân tích bất đẳng thức cuối thành 
(a b c 1)2 0 
là một bất đẳng thức hiển nhiên đúng. Chứng minh hoàn tất. 
Cách 2. Do x, y, z 1 và xyz 1 nên tồn tại các số thực a, b, c sao cho 
2 2 2 
, , 
bc ca ab 
x y z 
a b c 
(chẳng hạn 
3 3 3 
1 1 1 
a ,b ,c 
x y z 
). Khi đó, 
bất đẳng thức đã cho được viết lại dưới dạng thuần nhất là 
4 4 4 
2 2 2 2 2 2 
1. 
( ) ( ) ( ) 
a b c 
a bc b ca c ab 
Áp dụng bất đẳng thức Cauchy-Schwarz, ta có
11 
2 2 2 2 
2 2 2 2 2 2 
( ) 
. 
( ) ( ) ( ) 
a b c 
VT 
a bc b ca c ab 
Lại có 
(a2 b2 c2)2 (a2 bc)2 (b2 ca)2 (c2 ab)2 (ab bc ca)2 0, 
nên kết hợp với trên, ta dễ dàng suy ra được điều phải chứng minh. 
Cách 3. Dễ thấy tồn tại các số thực phân biệt ,,a b c sao cho , 
a 
x 
b 
,. 
bc 
yz 
ca 
Thay vào, bất đẳng thức cần chứng minh trở thành 
2 2 2 
2 2 2 
1. 
( ) ( ) ( ) 
a b c 
a b b c c a 
Áp dụng bất đẳng thức Cauchy-Schwarz, ta có 
2 2 2 VT (a b) (a c) a(a c) b(b a) c(c b) . 
Đến đây, với chú ý ở đẳng thức xy yz zx 0 trong đó 
x (a b)(a c), y (b c)(b a), z (c a)(c b), 
ta thấy 
2 2 2 2 2 2 2 2 
2 
( ) ( ) 2( ) 
( ) , 
a b a c x y z x y z xy yz zx 
x y z 
mà x y z (a b)(a c) a(a c) b(b a) c(c b), nên từ 
trên, ta có ngay điều phải chứng minh. 
Bài 5. Cho A, B, C là ba góc của một tam giác nhọn. Chứng minh rằng 
cos2 cos2 cos2 1 
. 
cos 1 cos 1 cos 1 2 
A B C 
A B C 
Chứng minh. 
Cách 1. Đặt x cotA, y cotB và z cotC thì ta có x, y, z 0 và 
xy yz zx 1. Sử dụng bất đẳng thức AM-GM, ta có
12 
2 
2 2 
2 2 2 
2 2 2 
2 
3 3 
2 2 
2 
3 3 
2 
cos 1 1 
cos 1 1 1 1 1 
1 
1 ( )( ) 
. 
2( ) 2( ) 
x 
A x x x x x 
A x x x x x 
x 
x x 
x x 
x x y x z 
x x 
x 
x y x z 
Tương tự ta cũng có 
2 3 3 
2 
2 3 3 
2 
cos 
, 
cos 1 2( ) 2( ) 
cos 
. 
cos 1 2( ) 2( ) 
B y y 
y 
B y z y x 
C z z 
z 
C z x z y 
Từ đây suy ra 
2 3 3 3 3 3 3 
2 2 2 
2 2 2 2 2 2 
2 2 2 
cos 
cos 1 2( ) 2( ) 2( ) 
2 2 2 
1 
. 
2 2 
A x y y z z x 
x y z 
A x y y z z x 
x xy y y yz z z zx x 
x y z 
xy yz zx 
Bài toán được chứng minh xong. 
Đẳng thức xảy ra khi và chỉ khi tam giác đã cho là tam giác đều. 
Cách 2. Đặt x cosA, y cosB và z cosC thì ta có x, y, z 0 và 
x2 y2 z2 2xyz 1. Bất đẳng thức cần chứng minh trở thành 
2 2 2 2 2 2 
1. 
1 1 1 
x y z 
x y z 
Thay 1 x2 y2 z2 2xyz và để ý rằng 
2 2 
2 2 (1 ) 
, 
1 1 
x x x 
x 
x x 
ta có 
thể viết lại bất đẳng thức trên dưới dạng 
2(1 ) 2(1 ) 2(1 ) 
2 . 
1 1 1 
x x y y z z 
xyz 
x y z 
Áp dụng bất đẳng thức AM-GM, ta có
13 
2 2 2 2 2 2 
3 
(1 ) (1 ) (1 ) (1 )(1 )(1 ) 
3 . 
1 1 1 (1 )(1 )(1 ) 
x x y y z z x y z x y z 
x y z x y z 
Sử dụng các kết quả quen thuộc trong tam giác nhọn 
(1 cos )(A 1 cosB)(1 cosC) cosAcosB cosC, 
3 
cos cos cos , 
2 
A B C 
ta thu được (1 x)(1 y)(1 z) xyz và 
3 
. 
2 
x y z Từ đó dẫn đến 
2 2 2 2 2 2 
3 
3 
(1 ) (1 ) (1 ) (1 )(1 )(1 ) 
3 
1 1 1 (1 )(1 )(1 ) 
3 
(1 )(1 )(1 ) 
9 9 
2 . 
3 3 
3 
2 
x x y y z z x y z x y z 
x y z x y z 
xyz 
x y z 
xyz xyz 
xyz 
x y z 
Cách 3. Thay 
2 2 2 
cos 
2 
b c a 
A 
bc 
vào, ta được 
2 
2 2 2 
2 2 2 2 2 
2 2 2 2 2 
cos 2 ( ) 
. 
cos 1 2[ ( ) ] 
1 
2 
b c a 
A bc b c a 
A b c a bc b c a bc 
bc 
Do đó, bất đẳng thức cần chứng minh tương đương với 
2 2 2 2 2 2 2 2 2 2 2 2 
2 2 2 2 2 2 
( ) ( ) ( ) 
1. 
( ) ( ) ( ) 
b c a c a b a b c 
bc b c a bc ca c a b ca ab a b c ab 
Áp dụng bất đẳng thức Cauchy-Schwarz, ta được 
2 2 2 2 
2 2 2 
( ) 
. 
( ) ( ) ( ) ( ) 
a b c 
VT 
bc b c ca c a ab a b abc a b c 
Mà 
(a2 b2 c2)2 bc(b c)2 abc(a b c) a2(a b)(a c) 0 
theo bất đẳng thức Schur bậc 4 nên kết hợp với trên, ta có ngay điều phải 
chứng minh.
14 
Bài 6. Chứng minh rằng với mọi số thực dương , , , a b c ta luôn có 
3 3 
1 1 1 3 
. 
a(b 1) b(c 1) c(a 1) abc 1 abc 
Chứng minh. 
Cách 1. Áp dụng kết quả quen thuộc (x y z)2 3(xy yz zx), ta có 
thể đưa bài toán về chứng minh 
2 
3 2 2 2 3 
1 3 
, 
( 1)( 1) 
1 
ab b c 
a b c abc 
hay là 
2 
3 2 2 2 3 
3 
. 
( 1)( 1)( 1) 
1 
a b c ab bc ca 
abc a b c 
a b c abc 
Bây giờ, để ý rằng 
(a 1)(b 1)(c 1) (abc 1) (a b c ab bc ca), 
a b c ab bc ca 33 abc 33 a2b2c2 
và hàm số ( ) 
t 
f t 
t k 
luôn tăng với mọi t, k 0. Do đó, ta dễ dàng thu 
được 
3 3 2 2 2 
3 3 2 2 2 
2 
3 2 2 2 3 
3 3 
( 1)( 1)( 1) ( 1) 3 3 
3 
. 
1 
a b c ab bc ca abc a b c 
abc a b c abc abc abc a b c 
a b c abc 
Phép chứng minh hoàn tất. Đẳng thức xảy ra khi và chỉ khi a b c. 
Cách 2. Để ý rằng 
1 1 ( 1) 
1 , 
( 1) ( 1) 1 
abc a b c 
a b a b a 
ta có 
3 
3 
1 1 1 1 ( 1) 
3 
( 1) ( 1) ( 1) ( 1) 1 
3 
3 . 
abc abc abc a b c 
a b b c c a a b b 
abc 
abc
15 
Từ đó, ta được 
3 
3 
3 3 
3 
3 3 
1 1 1 3 
. 
( 1) ( 1) ( 1) 1 1 
abc 
abc 
a b b c c a abc abc abc 
Cách 3. Đặt 3, abc k khi đó ta dễ dàng chứng minh được tồn tại các số 
dương ,, x y z sao cho , , . 
ky kz kx 
a b c 
x y z 
Với phép đặt ẩn này, bất 
đẳng thức của ta được đưa về 
1 1 1 3 
, 
( 1) 
1 1 1 
ky kz kz kx kx ky k k 
x y y z z x 
tương đương 
3 
. 
1 
x y z 
y kz z kx x ky k 
Áp dụng bất đẳng thức Cauchy-Schwarz, ta có 
2 
2 
( ) 
( ) ( ) ( ) 
( ) 3 
. 
( 1)( ) 1 
x y z x y z 
y kz z kx x ky x y kz y z kx z x ky 
x y z 
k xy yz zx k 
Bất đẳng thức được chứng minh xong. 
Bài 7. Cho a, b, c, d là các số thực thỏa mãn ad bc 1. Chứng minh 
a2 b2 c2 d2 ac bd 3. 
Chứng minh. 
Cách 1. Thay 3 3(ad bc) vào, ta viết được bất đẳng thức dưới dạng 
a2 b2 c2 d2 ac bd 3(ad bc), 
tương đương 
a2 b2 c2 d2 a 3d c b 3c d . 
Sử dụng bất đẳng thức AM-GM, ta có
16 
2 2 
2 2 
3 3 
3 , 3 . 
4 4 
d c c d 
a d c a b c d b 
Từ đó suy ra 
2 2 
2 2 
2 2 2 2 
3 3 
3 3 
4 
. 
d c c d 
a d c b c d a b 
a b c d 
Chứng minh hoàn tất. 
Đẳng thức xảy ra khi và chỉ khi 
1 
2 3 . 
2 3 
ad bc 
a d c 
b c d 
Cách 2. Để ý rằng 
2 2 2 2 
2 2 2 2 
3 1 
( ) ( ) , 
4 4 
1 3 
( ) ( ) . 
4 4 
a c ac a c a c 
b d bd d b d b 
Do đó bất đẳng thức cần chứng minh có thể viết lại thành 
3(a c)2 (d b)2 (a c)2 3(d b)2 4 3. 
Sử dụng bất đẳng thức AM-GM, ta được 
2 2 
2 2 
3( ) ( ) 2 3( )( ), 
( ) 3( ) 2 3( )( ). 
a c d b a c d b 
a c d b a c d b 
Từ đây suy ra 
VT 2 3 (a c)(d b) (a c)(d b) 4 3(ad bc) 4 3. 
Cách 3. Sử dụng giả thiết ad bc 1, ta có 
(a2 b2)(c2 d2) (ac bd)2 (ad bc)2 (ac bd)2 1. 
Áp dụng bất đẳng thức AM-GM, suy ra 
2 2 2 2 2 2 2 2 2 2 2 2 
2 
( ) ( ) 2 ( )( ) 
2 ( ) 1. 
a b c d a b c d a b c d 
ac bd 
Do đó ta ta chỉ cần chứng minh 
2 1 x2 x 3 với x ac bd.
17 
Rõ ràng vế trái của bất đẳng thức này luôn dương, vì thế ta có thể lấy bình 
phương hai vế để thu được bất đẳng thức tương đương 
2 
2 1 x2 x 3. 
Ta có 
2 
2 2 2 2 
2 2 2 
2 
2 
2 1 4(1 ) 4 1 
(1 ) 4 1 4 3 
1 2 3 3. 
x x x x x x 
x x x x 
x x 
Bài toán được chứng minh xong. 
Bài 8. (Chọn đội tuyển Iran 2009) Cho ,,a b c là các số thực dương thỏa 
mãn a b c 3. Chứng minh rằng 
2 2 2 2 2 2 
1 1 1 3 
. 
a b 2 b c 2 c a 2 4 
Chứng minh. 
Cách 1. Bất đẳng thức cần chứng minh tương đương với 
2 2 2 2 2 2 
2 2 2 2 2 2 
3 
. 
2 2 2 2 
a b b c c a 
a b b c c a 
Sử dụng bất đẳng thức Cauchy-Schwarz, ta được 
2 
2 2 2 2 2 2 
2 2 2 2 2 2 
2 2 2 2 2 2 2 
2 2 2 
( 2) ( 2) ( 2) 
( )( ) 
. 
3 
a b b c c a 
VT 
a b b c c a 
a b c a b a c 
a b c 
Do đó ta chỉ cần chứng minh 
2(a2 b2 c2) 2 (a2 b2)(a2 c2) 3(a2 b2 c2 3). 
Bất đẳng thức này tương đương với 
2 2 2 2 2 2 2 
2 2 2 2 2 2 2 2 
2 2 2 2 2 2 2 
2 ( )( ) 9, 
2 ( )( ) ( ) , 
( )( ) . 
a b a c a b c 
a b a c a b c a b c 
a b a c a b c ab bc ca
18 
Bất đẳng thức cuối đúng vì theo bất đẳng thức Cauchy-Schwarz, ta có 
(a2 b2)(a2 c2) (a2 bc) a2 b2 c2 ab bc ca. 
Bài toán được chứng minh xong. 
Đẳng thức xảy ra khi và chỉ khi a b c 1. 
Cách 2. Tương tự như cách 1, ta phải chứng minh 
2 2 2 2 2 2 
2 2 2 2 2 2 
3 
, 
2 2 2 2 
a b b c c a 
a b b c c a 
tương đương 
2 2 2 
2 2 2 
2 2 2 
2 2 2 2 2 2 
( ) ( ) ( ) 3 
. 
2( ) 2( ) 2( ) 2 
( ) ( ) ( ) 
a b b c c a 
a b b c c a 
a b b c c a 
a b b c c a 
Áp dụng bất đẳng thức Cauchy-Schwarz, ta được 
2 
2 2 2 
2 2 2 
2 2 2 2 2 2 
4( ) 
. 
2( ) 2( ) 2( ) 
( ) ( ) ( ) 
a b c 
VT 
a b b c c a 
a b b c c a 
a b b c c a 
Như vậy, bất đẳng thức đã cho sẽ được chứng minh nếu ta có 
2 2 2 
2 2 2 
2 2 2 2 2 2 
2( ) 2( ) 2( ) 
( ) ( ) ( ) 24. 
a b b c c a 
a b b c c a 
a b b c c a 
Đến đây, ta thấy 
2 2 2 2 4 1 
12 ( ) ( ) ( ) ( ) 
3 3 
a b a b c a b a b 
và 
2 2 2 2 2 2 
2 2 2 2 2 2 2 2 2 2 2 2 
2( ) 2( ) 2( ) 2( ) 2( ) 2( ) 
12 
a b b c c a a b b c c a 
a b b c c a a b b c c a 
nên bất đẳng thức này tương đương với 
2 2 2 
2 2 2 2 2 2 
6 6 6 
(a b) 1 (b c) 1 (c a) 1 0. 
a b b c c a 
Với giả thiết a b c, ta thấy ngay bất đẳng thức cuối hiển nhiên đúng 
nếu a2 b2 6, và vì thế bài toán của ta cũng được chứng minh trong 
trường hợp này. Xét trường hợp ngược lại a2 b2 6, khi đó ta có 
2 2 
1 1 
a b 2 8 
và
19 
do 
2 2 2 2 2 2 2 2 
2 
1 1 1 1 1 1 
2 2 2 2 8 2 
1 1 
( 0 6). 
8 2 
a c b c a b b b 
b 
Điều này dẫn đến 
2 2 2 2 2 2 
1 1 1 1 1 1 3 
, 
a b 2 b c 2 c a 2 8 8 2 4 
và bài toán của ta được giải quyết hoàn toàn. 
Cách 3. Trước hết, ta chứng minh kết quả sau: Nếu ,,x y z là các số thực 
dương, thì 
1 1 1 3 
. 
2( ) 
x y z 
y z z x x y xy yz zx x y z 
Nhân hai vế của bất đẳng thức này cho xy yz zx và để ý rằng 
, 
xy yz zx yz 
x 
y z y z 
ta viết được nó dưới dạng 
3( ) 
. 
2( ) 
yz zx xy xy yz zx 
y z z x x y x y z 
Nhân tiếp hai vế cho x y z rồi sử dụng bất đẳng thức AM-GM, ta có 
( ) ( ) 
, 
4 
( ) ( ) 
, 
4 
( ) ( ) 
. 
4 
yz x y z xyz x y z 
yz yz 
y z y z 
zx x y z yzx y z x 
zx zx 
z x z x 
xy x y z zxy z x y 
xy xy 
x y x y 
Cộng ba bất đẳng thức này lại, ta được 
3 
( ) ( ). 
2 
xy yz zx 
x y z xy yz zx 
x y y z z x 
Từ đây ta thu được kết quả như trên. 
Bây giờ, áp dụng bất đẳng thức trên với x a2 1, y b2 1, z c2 1, 
ta được
20 
2 2 2 
2 2 2 2 2 2 2 
2 2 2 
2 2 2 2 2 2 2 2 
2 2 2 
2 2 2 2 
2 2 2 
1 3 3 
2 ( 1)( 1) 2( 3) 
3 3 
2( ) 3 2( 3) 
3 3 
. 
2( 3) 
2( ) 3 
3 
a b c 
a b a b a b c 
a b c 
a b a b c a b c 
a b c 
ab a b c 
a b c 
Vậy ta chỉ cần chứng minh 
2 2 2 
2 2 2 2 
2 2 2 
3 3 3 
. 
2( 3) 4 
2( ) 3 
3 
a b c 
ab a b c 
a b c 
Đặt q ab bc ca, ta có a2 b2 c2 9 2q. Thay vào, bất đẳng 
thức trên được viết lại thành 
2 
3 
2 
9 2 3 3 3 
, 
2(9 2 3) 4 
2(9 2 ) 3 
3 
3( 3) 
0. 
4( 12 63)(6 ) 
q 
q q 
q 
q 
q q q 
Bất đẳng thức cuối đúng do 
( )2 
3. 
3 
a b c 
q ab bc ca 
Bài 9. Cho a, b, c, d là các số thực thỏa mãn 
(a2 1)(b2 1)(c2 1)(d2 1) 16. 
Chứng minh rằng 
3 ab ac ad bc bd cd abcd 5. 
Chứng minh. 
Cách 1. Để ý rằng 
2 2 2 2 
2 2 2 
( 1)( 1) (1 ) ( ) , 
( 1)( 1) ( 1) ( ). 
a b ab a b 
c d cd c d 
Từ đó, sử dụng bất đẳng thức Cauchy-Schwarz, ta được
21 
2 
2 
16 (1 )( 1) ( )( ) 
( 1) . 
ab cd a b c d 
ab ac ad bc bd cd abcd 
Từ đây ta suy ra 
3 ab ac ad bc bd cd abcd 5 
là điều phải chứng minh. 
Cách 2. Do a2 1 a2 i2 (a i)(a i) nên 
16 (a i)(a i) (a i) (a i) . 
Ta có 
2 3 4 
2 3 4 
( ) 
1 
1 , 
( ) 
1 
1 . 
sym 
sym 
sym 
sym 
sym 
sym 
a i abcd i abc i ab i a i 
abcd i abc ab i a 
abcd ab i a abc 
a i abcd i abc i ab i a i 
abcd i abc ab i a 
abcd ab i a abc 
Do đó tích (a i) (a i) có dạng (A Bi)(A Bi) A2 B2, 
từ đây suy ra 
2 2 
2 
16 1 1 , 
sym sym 
abcd ab a abc abcd ab 
suy ra 
3 ab ac ad bc bd cd abcd 5. 
Bài toán được chứng minh xong. 
Cách 3. Bất đẳng thức cần chứng minh tương đương với 
2 
4 1 4, 
( ) 1 4, 
( ) 1 16. 
ab ac ad bc bd cd abcd 
a b c d bcd bc bd cd 
a b c d bcd bc bd cd 
Áp dụng bất đẳng thức Cauchy-Schwarz, ta được
22 
2 2 2 
2 2 2 2 
( 1) ( ) ( 1) 
( 1)( 1)( 1)( 1) 16. 
VT a b c d bcd bc bd cd 
a b c d 
Suy ra ta chỉ cần chứng minh 
(b c d bcd)2 (bc bd cd 1)2 (b2 1)(c2 1)(c2 1). 
Thực hiện khai triển và rút gọn, ta được ngay một hằng đẳng thức đúng. 
Bài 10. (Olympic Toán Belarus 1998) Cho ,,a b c là các số thực dương. 
Chứng minh rằng 
1. 
a b c a b b c 
b c a b c a b 
Chứng minh. 
Cách 1. Bất đẳng thức cần chứng minh tương đương với 
( 2 )2 
1 2 . 
( )( ) 
a b c a b b c a b c 
b c a b c a b a b b c 
Sử dụng bất đẳng thức Cauchy-Schwarz, ta có 
2 2 2 2 2 
2 2 
2 
( ) 
1 
( 2 ) 
. 
( )( ) 
a b c a b c b a b c b 
b c a ab bc ca b ab bc ca b 
a b c 
a b b c 
Bài toán được chứng minh xong. 
Đẳng thức xảy ra khi và chỉ khi a b c. 
Cách 2. Nhân cả hai vế của bất đẳng thức cho , bc ta được 
2 
2 2 2 
2 2 2 
( ) ( ) ( ) ( ) 
2 , 
( ) 
, 
( ) 
2 2 0. 
a a b b b c c b c b c 
a b c 
b c a a b 
ac bc c b b c 
b c 
b a a c a b 
ac bc b c b c 
c c b b 
b a c a a b 
Bất đẳng thức cuối cùng đúng do 
2 2 
2 2 2 2 
2 2 , 2 2 , 
( ) 
. 
ac bc ac bc b b 
c c c b 
b a b a c c 
c c b c b 
b 
a a b a b
23 
Cách 3. Bất đẳng thức cần chứng minh tương đương với 
2 
1, 
2 
. 
( ) ( ) ( ) 
a a b b c c b 
b b c c b c a a b a b 
ac b bc a b 
b b c c b c a a b a b 
Áp dụng bất đẳng thức Cauchy-Schwarz, ta có 
2 
2 
( ) 
( ) ( ) 
( ) ( ) ( ) 
. 
ac b bc a b c 
b c a b 
b b c c b c a a b b 
ac ab bc 
b c a 
Bài toán được quy về chứng minh 
2 
2 ( ) 
( ) ( ) . 
ab bc ca a b a b c 
b c a b 
c a b a b b 
Bất đẳng thức này tương đương với 
( 2 )(2 ) 
2 2 2 2 4 2 , 
ab bc ca a b b c ac 
a b c a b c 
c a b b b 
2 
ab bc 
b 
c a 
(đúng theo AM-GM). 
Bài toán được chứng minh xong. 
Cách 4. Đặt , , 
a c 
x y 
b b 
ta có 
1 1 
, , . 
1 1 
c y a b x b c y 
a x b c y a b x 
Bất đẳng thức cần chứng minh có thể viết lại thành 
1 1 1 
1. 
1 1 
y x y 
x 
y x y x 
Sau khi khai triển và rút gọn, ta được 
x3y2 x2 x y3 y2 x2y 2xy 2xy2. 
Bất đẳng thức cuối này đúng vì theo AM-GM, ta có 
3 2 3 2 3 3 
2 , 2 2, 2 2 2 . 
2 2 
x y x x y x y y 
x y xy x y xy 
Cách 5. Bất đẳng thức cần chứng minh tương đương với
24 
2 2 2 2 
( ) 3 ( ) 2 , 
( )( ) 
2 2 2 . 
( )( ) 
a b c a b b c 
a b c a b c 
b c a b c a b 
a b c ab bc ca a b c a c 
a b c 
b c a c a b a b b c 
Do 
2( )2 2( )2 2( )2 
0 
2 
ab bc ca a b c b c a c a b 
a b c 
c a b abc 
và 
a2 b2 c2 (a b)2 (b c)2 (c a)2 
a b c 
b c a b c a 
nên ta chỉ cần chứng minh 
( )2 ( )2 ( )2 ( )( )2 
( )( ). 
a b b c c a a b c a c 
b c a a b b c 
Sử dụng bất đẳng thức Cauchy-Schwarz, ta có 
2 
( )2 ( )2 ( ) ( ) ( )2 
. 
a b b c a b b c a c 
b c b c b c 
Suy ra ta chỉ cần chứng minh 
( )2 ( )2 ( )( )2 
. 
( )( ) 
a c a c a b c a c 
b c a a b b c 
Bất đẳng thức cuối này đúng vì 
1 1 ( ) 
0. 
( )( ) ( )( ) 
a b c b a b c 
a b c a b b c a a b b c 
Bài 11. Chứng minh rằng với mọi a, b, c dương, ta đều có 
4 2 2 4 2 2 4 2 2 
2 2 2 
2 2 2 
. 
2 2 2 
a b c b c a c a b 
a b c 
a bc b ca c ab 
Chứng minh. 
Cách 1. Sử dụng tính thuần nhất, ta có thể chuẩn hóa cho abc 1. Bất đẳng 
thức cần chứng minh trở thành 
6 6 6 
3 3 3 
2 2 2 
.. 
( 2) ( 2) ( 2) 
a b c 
a b c 
a a b b c c 
Áp dụng bất đẳng thức Holder, ta có
25 
3 
(a6 1 1)(a2 a 1)(a2 1 a) 3 a10 3 a 3 a a(a3 2)3, 
từ đó suy ra 
6 3 
3 2 2 
2 2 3 
1. 
( 2) 1 1 
a a 
a 
a a a a a a 
Bằng cách thiết lập hai bất đẳng thức tương tự cho hai biểu thức còn lại, ta 
có thể đưa bài toán về chứng minh 
2 2 2 
1 1 1 
1. 
a a 1 b b 1 c c 1 
Đặt 2 2 2 
, , 
yz zx xy 
a b c 
x y z 
với x, y, z 0, ta được 
4 4 4 
4 2 2 2 4 2 2 2 4 2 2 2 
1. 
x y z 
x x yz y z y y zx z x z z xy x y 
Tới đây, sử dụng bất đẳng thức Cauchy-Schwarz kết hợp với kết quả cơ bản 
x2y2 y2z2 z2x2 xyz(x y z), ta có 
2 2 2 2 
4 4 4 2 2 2 2 2 2 
2 2 2 2 
4 4 4 2 2 2 2 2 2 
( ) 
( ) ( ) ( ) 
( ) 
1. 
( ) 2( ) 
x y z 
VT 
x y z xyz x y z y z z x x y 
x y z 
x y z y z z x x y 
Bài toán được chứng minh xong. 
Đẳng thức xảy ra khi và chỉ khi a b c. 
Cách 2. Áp dụng bất đẳng thức Holder, ta có 
(a4 b2c2 b2c2)(a b c)(a c b) (a2 2bc)3, 
suy ra 
4 2 2 2 
2 
2 2 
. 
2 
a b c a bc 
a bc a b c 
Từ đó, ta được 
2 2 2 2 2 2 
. 
a bc b ca c ab 
VT a b c 
a b c a b c a b c 
Cách 3. Trước hết, ta sẽ chứng minh 
2 2 2 
. 
2 2 2 
a b c a b c 
a bc b ca c ab ab bc ca
26 
Thật vậy, do 
2 2 
( )( ) 
2 ( 2 )( ) 
a a a a b a c 
ab bc ca a bc a bc ab bc ca 
nên bất 
đẳng thức này tương đương với 
2 2 2 
( )( ) ( )( ) ( )( ) 
0. 
2 2 2 
a a b a c b b c b a c c a c b 
a bc b ca c ab 
Không mất tính tổng quát, giả sử . a b c Khi đó 
2 
( )( ) 
0 
2 
c c a c b 
c ab 
nên ta chỉ cần chứng minh 
2 2 
( )( ) ( )( ) 
0. 
2 2 
a a b a c b b c b a 
a bc b ca 
Do ( ) 0 
a 
a c b c 
b 
và a b 0 nên 
2 
2 2 2 2 
2 2 2 
2 2 
( )( ) ( )( ) ( )( ) ( )( ) 
2 2 ( 2 ) 2 
2 ( ) ( )( ) 
0. 
( 2 )( 2 ) 
a a b a c b b c b a a a b b c b b c b a 
a bc b ca b a bc b ca 
c a b b c a ab b 
b a bc b ca 
Quay trở lại bài toán chính. Bình phương hai vế, bất đẳng thức cần chứng 
minh tương đương với 
4 2 2 4 2 2 4 2 2 
2 
2 2 2 
2 ( 2 )( 2 ) 
2 2 . 
2 ( 2 )( 2 ) 
a b c a b c b c a 
a ab 
a bc a bc b ca 
Áp dụng kết quả vừa chứng minh ở trên, ta có 
4 2 2 2 
2 
2 2 
2 
2 
2 
2 2 
2 2 2 2 
2 3 
2 2 
3 
2 
3 ( ) 
( ) 
. 
2( ) 
a b c a bc 
a bc 
a bc a bc 
a 
a ab abc 
a bc 
abc a b c 
a ab 
ab bc ca 
a b c 
a a b c 
ab bc ca 
Vậy ta chỉ cần chứng minh
27 
4 2 2 4 2 2 
2 2 
( 2 )( 2 ) 
. 
( 2 )( 2 ) 
a b c b c a 
ab bc ca 
a bc b ca 
Sử dụng bất đẳng thức AM-GM, ta có 
4 2 2 4 2 2 4 2 2 
3 
2 2 2 
( 2 )( 2 )( 2 ) 
3 . 
( 2 )( 2 )( 2 ) 
a b c b c a c a b 
VT 
a bc b ca c ab 
Mặt khác, ta lại có 
2 2 
4 2 2 2 2 2 2 ( 2 ) 
2 ( ) ( ) ( ) 
3 
a bc 
a b c a bc bc nên 
VT 3 (a2 2bc)(b2 2ca)(c2 2ab). 
Cuối cùng ta chỉ cần chứng minh 
(a2 2bc)(b2 2ca)(c2 2ab) (ab bc ca)3. 
Khai triển và rút gọn, ta được 
a3b3 3a2b2c2 4abc a3 3abc ab(a b). 
Bất đẳng thức cuối này đúng vì ta có a3b3 3a2b2c2 abc ab(a b) 
(bất đẳng thức Schur bậc 3 áp dụng cho ab, bc, ca ) và 
4abc a3 2abc ab(a b) (theo AM-GM). 
Bài toán được chứng minh xong. Bài 12. Chứng minh rằng với các số không âm ,,a b c thỏa mãn không có 
hai số nào đồng thời bằng 0, ta luôn có 
2 2 2 2 2 2 
( ) ( ) ( ) 
2. 
a b c b c a c a b 
b bc c c ca a a ab b 
Chứng minh. 
Cách 1. Áp dụng bất đẳng thức AM-GM, ta có 
2 2 2 2 2 2 
( ) 4 ( )( ) 4 ( ) 
, 
( ) ( )( ) 
a b c a b c ab bc ca a ab bc ca 
b bc c b bc c ab bc ca b c a b c 
nên để chứng minh bất đẳng thức đã cho, ta chỉ cần có 
2 2 2 
4 ( ) 4 ( ) 4 ( ) 
2, 
( )( ) ( )( ) ( )( ) 
a ab bc ca b ab bc ca c ab bc ca 
b c a b c c a a b c a b a b c 
tức là
28 
( )2 
. 
2( ) 
a b c a b c 
b c c a a b ab bc ca 
Nhưng bất đẳng thức này hiển nhiên đúng theo Cauchy-Schwarz nên phép 
chứng minh của ta vì thế được hoàn tất. Dễ thấy đẳng thức xảy ra khi và chỉ 
khi a b c hoặc ,0 a b c cùng hoán vị. 
Cách 2. Sử dụng bất đẳng thức Cauchy-Schwarz, ta có 
2 2 
2 
2 2 
( ) ( ) 
( ) . 
a b c a b bc c 
a b c 
b bc c b c 
Do đó, bất đẳng thức cần chứng minh được đưa về 
2 2 
2 ( ) 
( ) 2 . 
a b bc c 
a b c 
b c 
Mặt khác, ta lại có 
Cauchy - Schwarz 
( 2 2) 1 
2 2 ( ) 2 
9 
2 ( ) ( ) 
9 
4( ) , 
a b bc c 
a b c abc 
b c b c 
abc 
a b c 
a b c 
abc 
ab bc ca 
a b c 
nên ta chỉ cần chứng minh được 
2 9 
( ) 4( ) , 
abc 
a b c ab bc ca 
a b c 
tức là 
2 2 2 9 
2( ). 
abc 
a b c ab bc ca 
a b c 
Đây chính là bất đẳng thức Schur bậc 3 ở dạng phân thức nên nó hiển nhiên 
đúng. Và vì thế, bất đẳng thức của ta được chứng minh xong. 
Cách 3. Nhân cả hai vế của bất đẳng thức đã cho với ab bc ca và thực 
hiên phép tách 2(ab bc ca) a(b c) b(c a) c(a b), ta thấy 
2 2 2 2 
2 2 2 2 
( )( ) ( )[ ( ) ( )] 
( ) 
( )( ) ( )( ) 
. 
a b c ab bc ca a b c b a b c a c 
a b c 
b bc c b bc c 
ab a b b c ca c a b c 
b bc c b bc c 
Do đó
29 
2 2 
2 2 2 2 
2 2 2 2 
2 
2 2 2 2 
( )( ) 
2( ) 
( )( ) ( )( ) 
( )( ) ( )( ) 
( ) ( ) 
, 
( )( ) 
a b c ab bc ca 
ab bc ca 
b bc c 
ab a b b c ca c a b c 
b bc c b bc c 
ab a b b c ab a b a c 
b bc c a ac c 
ab a b ab bc ca 
a ac c b bc c 
và ta tìm được 
2 
2 2 2 2 2 2 
( ) ( ) 
2 . 
( )( ) 
a b c ab a b 
b bc c a ac c b bc c 
Do ,,a b c là các số không âm nên hiển nhiên vế phải không thể nào bé hơn 
0, từ đó, ta dễ dàng suy ra được bất đẳng thức cần chứng minh. 
Cách 4. Bài toán sẽ được chứng minh xong nếu ta có 
2 2 2 2 
1 1 2( ) 
. 
( ) ( ) ( ) 
a b 
a ac c b bc c ab a b bc b c ca c a 
(1) 
Thật vậy, giả sử bất đẳng thức trên đúng, khi đó 
2 2 2 2 2 2 
2 2 2 2 
2 2 2 2 
( ) 
1 1 
2( ) 
2. 
( ) ( ) ( ) 
a b c ab ca 
b bc c b bc c b bc c 
ab ab 
b bc c c ca a 
ab 
a ac c b bc c 
a b 
ab 
ab a b bc b c ca c a 
Bây giờ ta sẽ đi chứng minh (1). Áp dụng bất đẳng thức AM-GM, ta có 
2 2 2 2 2 2 2 2 
1 1 2 
. 
a ac c b bc c (a ac c )(b bc c ) 
Suy ra ta chỉ cần chứng minh 
2 2 2 2 ( ) ( ) ( ) 
( )( ) . 
ab a b bc b c ca c a 
a ac c b bc c 
a b 
Theo bất đẳng thức Cauchy-Schwarz
30 
2 2 2 2 
2 2 2 2 
2 
( )( ) 
( )( ) 
( )( ) ( ) , 
ac bc a ac c b bc c 
ac a ac c bc b bc c 
a c b c ab a b c c 
từ đó suy ra 
2 2 2 2 2 
2 
2 2 
2 
( )( ) 
2 
( ) ( ) ( ) 
. 
a ac c b bc c ab a b ab c c 
ab 
ab a b c c 
a b 
a b 
ab c c 
a b 
ab a b bc b c ca c a 
a b 
Bài toán được chứng minh xong. 
Bài 13. Cho ,,x y z là các số dương thỏa mãn 2 4 7 2 . x y z xyz Tìm 
giá trị nhỏ nhất của biểu thức P x y z. 
Lời giải. 
Cách 1. Áp dụng bất đẳng thức AM-GM, ta có 
1 
6 10 14 30 
1 1 7 
5 3 15 
2 2 
2 4 7 6 10 14 30 
3 5 2 3 5 2 
2 
30 
3 5 2 
x y z x y z 
x y z 
x y z 
và 
1 
6 5 4 15 
2 1 4 
5 3 15 
2 2 
2( ) 6 5 4 15 
3 5 2 3 5 2 
2 
15 . 
3 5 2 
x y z x y z 
x y z 
x y z 
Từ đây ta có
31 
2 
2 2 
2 
1 1 7 2 1 4 
5 3 15 5 3 15 
2 
(2 4 7 ) 2( ) 
( ) 
8 
2 2 
30 15 2 3 5 2 3 5 2 30 15 125 3 5 2 , 
8 8 4 
x y z x y z 
P x y z 
xyz 
x y z x y z 
x y z 
xyz xyz 
suy ra 
15 
. 
2 
P Mặt khác, dễ thấy bộ 
5 
( , , ) 3, , 2 
2 
x y z thỏa mãn điều 
kiện của đề bài và 
15 
. 
2 
P Do đó 
15 
min . 
2 
P 
Cách 2. Từ giả thiết suy ra 
2 4 
2 7 
x y 
z 
xy 
và 2xy 7 0. Do đó 
2 4 
. 
2 7 
x y 
P x y 
xy 
Sử dụng bất đẳng thức AM-GM, ta có 
2 2 
2 4 11 7 2 4 2 
2 7 2 2 2 7 
11 2 7 2( 7) 11 2 7 
. 
2 2 (2 7) 2 
x y x y 
P x y x y 
xy x x xy x 
xy x x 
x x 
x x x xy x x 
Đẳng thức xảy ra khi 
2 7 2( 2 7) 
, 
2 2 7 
xy x 
xy 
hay 
7 2 2 7 
. 
2 
x 
y 
x 
Mặt khác, theo bất đẳng thức Cauchy-Schwarz thì 
2 
2 ( 7)(9 7) 3 7 
7 
4 4 
x x 
x (Đẳng thức xảy ra khi x 3 ). 
Suy ra 
11 3 7 9 3 3 15 
6 
2 2 2 2 2 
x 
P x x 
x x x 
(Đẳng thức xảy 
ra khi x 3 ). Vậy 
15 
min 
2 
P đạt được khi
32 
2 
2 4 
2 7 3 
7 2 7 5 
. 
2 2 
3 2 
x y 
z 
xy x 
x 
y y 
x 
x z 
Cách 3. Ta đã biết tính chất sau: Nếu ()fx khả vi bậc hai và lồi trên ( , ) ab 
thì với mọi , , ( ), ax b y ta có 
f (x) f (y) f (y)(x y). 
Dưới đây, ta sẽ sử dụng tính chất này để giải bài toán đã cho. 
Đặt 
7 
7 , 
2 
x a y b và 
2 7 
7 
z c thì ta có a b c abc và 
7 
(14 7 4 ). 
14 
P a b c 
Do a, b, c 0 và a b c abc nên tồn tại , , 0, 
2 
A B C thỏa mãn 
A B C và a tanA, b tanB, c tanC, suy ra 
7 
(14 tan 7 tan 4 tan ). 
14 
P A B C 
Do f (x) tanx khả vi bậc hai và lồi trên 0, 
2 
nên theo tính chất trên, 
với mọi 0, , 
2 
x, y ta có 
tanx tany (tany) (x y) tany (tan2 y 1)(x y). 
Thay (x, y) lần lượt bởi 
3 5 
, arctan , , arctan , , arctan 7 , 
7 7 
A B C 
ta được
33 
2 
2 
2 
3 3 3 3 16 3 
tan 1 arctan arctan , 
7 7 7 7 7 7 
5 5 5 5 32 5 
tan 1 arctan arctan , 
7 7 7 7 7 7 
tan 7 7 1 a 
A A A 
B B B 
C C rctan 7 7 8 C arctan 7 . 
Từ đó suy ra (chú ý rằng 
3 5 
arctan arctan arctan 7 
7 7 
) 
7 3 5 
15 7 32 arctan arctan arctan 7 
14 7 7 
7 15 
15 7 32( ) . 
14 2 
P A B C 
A B C 
Đẳng thức xảy ra khi và chỉ khi 
5 
3, , 2. 
2 
x y z 
Bài 14. Cho ,,a b c là các số thực không âm thỏa mãn không có hai số nào 
đồng thời bằng 0. Chứng minh rằng 
2 2 2 
1 1 1 2 2 
. 
a bc b ca c ab ab bc ca 
Chứng minh. 
Cách 1. Không mất tính tổng quát, giả sử a max{a, b, c}. Khi đó, sử 
dụng bất đẳng thức Holder, ta có 
VT2 (b c)3(a2 bc) a3(b2 ca) a3(c2 ab) (2a b c)3. 
Bài toán được đưa về chứng minh 
(2a b c)3(ab bc ca) 8(b c)3(a2 bc) 8a3(b2 c2 ab ac). 
Đến đây, sử dụng tính thuần nhất, ta chuẩn hóa cho b c 1. Bất đẳng 
thức trên được lại thành 
(2a 1)3(a bc) 8(a2 bc) 8a3(1 2bc a), 
hay 
bc[(2a 1)3 16a3 8] a(2a 1)3 8a2 8a3(a 1) 0.
34 
Ta có (2a 1)3 (b c 1)3 8 và 
a(2a 1)3 8a2 8a3(a 1) a(a 1)2 3a3 0 
nên bất đẳng thức cuối hiển nhiên đúng. Phép chứng minh được hoàn tất. 
Cách 2. Không mất tính tổng quát, ta có thể giả sử a max{a, b, c}. Khi 
đó có hai trường hợp xảy ra. 
+ Trường hợp 1. . b c a Dễ thấy a2 b2 c2 2(ab bc ca). 
Do đó, áp dụng bất đẳng thức Cauchy-Schwarz, ta được 
2 2 2 2 
2 2 2 
1 9 
3 3 3 
. 
a bc a bc b ca c ab 
a b c ab bc ca ab bc ca 
+ Trường hợp 2. a b c. Khi đó, cũng sử dụng bất đẳng thức Cauchy 
-Schwarz, ta có 
2 2 2 2 2 2 
1 1 4 2 2 
. 
b ca c ab b ca c ab b c a(b c) 
Do đó, ta chỉ cần chứng minh được 
2 2 2 
2( ) 
2 2 2. 
( ) 
ab bc ca ab bc ca 
a bc b c a b c 
Ta có 2 2 2 
( ) ( ) 
1 1 
ab bc ca a b c a a b c a b c 
a bc a bc a a 
và 
2 2 2 
( ) 
, 
( ) ( ) ( ) 
ab bc ca a b c a 
b c a b c b c a b c a b c 
nên bất đẳng thức trên được suy ra từ 
2 
2 2 2. 
b c a 
a a b c 
Từ đây, bằng cách đặt 1, 
b c 
t 
a 
ta có thể đưa bài toán về chứng 
minh một bất đẳng thức một biến và việc chứng minh không mấy khó khăn 
(chỉ cần AM-GM). Xin được dành lại cho bạn đọc.
35 
Cách 3. Nếu có một số bằng 0 trong ,,a b c thì bất đẳng thức hiển nhiên 
đúng. Vì vậy ta chỉ cần xét 0. abc Khi đó, đặt 
11 
, xy 
ab 
và 
1 
z , 
c 
ta có thể viết lại bất đẳng thức cần chứng minh dưới dạng 
2 2 2 
( ) ( ) ( ) 
2 2. 
x x y z y x y z z x y z 
x yz y zx z xy 
Theo bất đẳng thức AM-GM thì 
2 2 
( ) 2 2 ( ) 
. 
( ) 2( ) 
x x y z x x y z 
x yz x x y z x yz 
Do đó, ta chỉ cần chứng minh được 
2 
( ) 
1. 
3 2 
x x y z 
x xy xz yz 
Đến đây, thực hiện khai triển và rút gọn, ta thu được bất đẳng thức đúng 
2(x y)2(y z)2(z x)2 12xyz(x y)(y z)(z x) 2x2y2z2 0. 
Bài toán được chứng minh xong. 
Bài 15. Cho ,,a b c là các số thực dương. Chứng minh rằng 
3 
( 1) ( 1) ( 1) ( 1)( 1)( 1). 
2 
a b b c c a a b c 
Chứng minh. 
Cách 1. Chia hai vế của bất đẳng thức cho (a 1)(b 1)(c 1), ta được 
1 1 1 3 
. 
1 1 1 1 1 1 2 
a b c 
a c b a c b 
Sử dụng bất đẳng thức AM-GM, ta có 
1 1 1 1 1 1 
, , 
1 1 2 1 1 1 1 2 1 1 
1 1 1 
. 
1 1 2 1 1 
a a b b 
a c a c b a b a 
c c 
c b c b 
Cộng ba bất đẳng thức này lại theo vế, ta được 
1 1 1 1 1 3 
. 
1 1 2 1 1 1 2 
a a b c 
a c a b c
36 
Bài toán được chứng minh xong. 
Đẳng thức xảy ra khi và chỉ khi 1. a b c 
Cách 2. Áp dụng bất đẳng thức Cauchy-Schwarz, ta có 
VT2 3 a(b 1) b(c 1) c(a 1) . 
Như vậy, bất đẳng thức của ta sẽ được chứng minh nếu ta có 
4 a(b 1) b(c 1) c(a 1) 3(a 1)(b 1)(c 1), 
tức là 
a b c ab bc ca 3abc 3. 
Xét trường hợp ngược lại a b c ab bc ca 3abc 3. Khi đó, 
lấy bình phương hai vế, ta viết lại bất đẳng thức cần chứng minh dưới dạng 
9 
2 ( 1)( 1) ( 1)( 1)( 1). 
4 
a ab ab b c a b c 
Theo bất đẳng thức AM-GM thì 
2 ( 1)( 1) ( 1) ( 1) 
3 3. 
ab b c ab c b 
a b c ab bc ca abc 
Do đó, ta chỉ cần chứng minh được 
9 
2( ) 3 3 ( 1)( 1)( 1), 
4 
a b c ab bc ca abc a b c 
tương đương 
a b c ab bc ca 3abc 3 (đúng). 
Bài toán được chứng minh xong. 
Cách 3. Sử dụng bất đẳng thức Cauchy-Schwarz, ta được 
( 1) ( 1) ( 1) ( 1) ( 1) 
( 1)( 2 1). 
a b b c a b b c 
a bc b 
Từ đó bài toán được quy về chứng minh 
3 
( 1)( 2 1) ( 1) ( 1)( 1)( 1). 
2 
a bc b c a a b c 
Đơn giản cả hai vế cho a 1, ta được 
3 
2 1 ( 1)( 1). 
2 
bc b c b c 
Đến đây, sử dụng tiếp bất đẳng thức Cauchy-Schwarz và kết hợp với bất 
đẳng thức AM-GM, ta có
37 
2 
( 2)(2 1) 
( 2 1) ( 1) 1 ( 1) 
1 1 
( 2) (2 1) 3 
( 1) ( 1)( 1). 
4( 1) 2 
c c c 
VT bc b c b 
c c 
c c 
b b c 
c 
Bài 16. Cho các số thực không âm ,,x y z thỏa mãn không có hai số nào 
đồng thời bằng 0. Chứng minh bất đẳng thức sau 
2 1 . 
( )( )( ) 
x y z xyz 
y z z x x y x y y z z x 
Chứng minh. 
Cách 1. Không mất tính tổng quát, giả sử z min{x, y, z}. Khi đó, ta sẽ 
chứng minh 
2 . 
2 
x y x y 
y z z x x y z 
(1) 
Thật vậy, theo bất đẳng thức Holder, ta có 
2 
2( ) 2( ) ( )2. 
x y 
x y z y z x x y 
y z z x 
Mặt khác, do z min{x, y, z} nên 
2 2 2 
2 
2 
2 
( ) ( ) ( 2 ) ( ) 
( ) 
( 2 ) ( ) 
4 
( ) ( 2 ) 
. 
4 
x y z y z x xy x y z z x y 
x y 
x y z z x y 
x y x y z 
Kết hợp với trên, ta có ngay 
3 
2 2 
( ) 
2 . 
( ) ( ) 2 
x y x y x y 
y z z x x y z y z x x y z 
Bây giờ, sử dụng (1) kết hợp với đánh giá 
2 
2 
2 
1 1 ( ) 
, 
( )( ) ( ) 4 ( ) ( 2 ) 
1 1 
( ) 
xy x y 
x z y z z x y z z x y z x y z 
xy x y
38 
ta đưa được bài toán về chứng minh 
2 
( ) 
2 2 1 . 
2 ( 2 ) 
x y z z x y 
x y z x y x y z 
Đặt 
2 
1, 
z 
t 
xy 
ta phải chứng minh 
2 
2 
2 1 
1 2 2( 1) 
t t 
t t 
hay 2 
4 2 2 
2 4 . 
1 2 1 ( 1) 
t t t 
t t t 
Nếu 0 t thì bất đẳng thức trở thành đẳng thức. Xét trường hợp 0, t bất 
đẳng thức này có thể được thu gọn thành 
2 
2 2 (2 3) 
2 
2 1 ( 1) 
t t t t 
t t 
hay là 2 
1 4 2(2 3) 
. 
2 2 ( 1) ( 1) 
t 
t t t 
Theo bất đẳng thức AM-GM thì 
1 2 
. 
2t(t 1) 3t 1 
Lại có 
2 
2 2 
1 8 2(2 3) (3 5)( 1) 
0, 
2 3 1 ( 1) (3 1)( 1) 
t t t 
t t t t 
nên bất đẳng thức trên hiển nhiên đúng. Bài toán được chứng minh xong. 
Đẳng thức xảy ra khi và chỉ khi x y z hoặc x y, z 0 cùng các 
hoán vị tương ứng. 
Cách 2. Đặt a x, b y, c z, ta phải chứng minh 
2 2 2 2 2 2 
2 2 2 2 2 2 2 2 2 2 2 2 
2 1 . 
( )( )( ) 
a b c a b c 
b c c a a b a b b c c a 
Áp dụng bất đẳng thức Holder, ta có 
VT2 a(b2 c2) b(c2 a2) c(a2 b2) (a b c)3. 
Do đó, ta chỉ cần chứng minh được 
3 2 2 2 
2 2 2 2 2 2 
( ) 4 
4 . 
( ) ( ) ( ) ( )( )( ) 
a b c a b c 
ab a b bc b c ca c a a b b c c a 
Do (a b c)3 4 ab(a b) a(a b)(a c) 3abc 3abc và
39 
2 2 2 2 2 2 
2 2 2 2 2 
2 2 2 
2 
4 4 9 
( ) 8 2 
9 
9 
, 
2 
a b c a b c abc abc a 
a b a a b a a a b 
abc 
a a 
nên ta chỉ cần chứng minh 
2 2 2 
3 9 
, 
( ) ( ) ( ) 2( )( ) 
abc abc 
ab a b bc b c ca c a a b c a b c 
tương đương 
abc (a b)(a b)2 (b c)(b c)2 (c a)(c a)2 0 (đúng). 
Bài toán được chứng minh xong. 
Cách 3. Bình phương hai vế, bất đẳng thức đã cho có thế được viết lại thành 
4 
2 4 . 
( )( ) ( )( )( ) 
x xy xyz 
y z x z y z x y y z z x 
Sử dụng các bất đẳng thức Cauchy-Schwarz và AM-GM, ta được 
( )( ) 
( )( ) ( )( ) ( )( ) 
( ) ( ) 
( )( )( ) 
( ) 2 ( ) 6 
( )( )( ) ( )( )( ) 
4 
1 . 
( )( )( ) 
xy xy x z y z xy xy z 
x z y z x z y z x z y z 
xy x y z xy x y 
x z y z x y 
xy x y xyz xy x y xyz 
x z y z x y x y y z z x 
xyz 
x y y z z x 
Do đó, ta chỉ cần chứng minh được 
4 
2. 
( )( )( ) 
x y z xyz 
y z z x x y x y y z z x 
Đây chính là bất đẳng thức Schur bậc 3 ở dạng phân thức, vì thế nó hiển 
nhiên đúng. Và như vậy, ta có điều phải chứng minh. 
Cách 4. Sử dụng bất đẳng thức Holder, ta có
40 
2 3 
2 
3 
3 
( )( ) ( )( )( ) 
( ) 
. 
( )( )( ) 
x x x 
y z x y x z y z x y x z 
x y z 
x y y z z x 
Từ đó với chú ý 
( )( ) 
1 , 
( )( )( ) ( )( )( ) 
xyz x y z xy yz zx 
x y y z z x x y y z z x 
ta đưa được bài toán về chứng minh 
( )2 2 
4 . 
( )( ) 
x y z x 
xy yz zx x y x z 
Không mất tính tổng quát, giả sử z min{x, y, z}. Khi đó ta có 
2 2 2 2 
2 2 2 
2 
2 2 2 
( ) 
3 
3 
4 2 2 2 
. 
( )( ) 
x y z x y z xy yz zx 
xy yz zx xy yz zx 
x y z xy yz zx 
xy yz zx z 
x y z xy yz zx 
x z y z 
Vậy chỉ cần chứng minh 
(x y)(x2 y2 4z 2 2xy 2yz 2zx) 4 x2(y z). 
Sau khi khai triển và rút gọn, ta thu được bất đẳng thức hiển nhiên đúng 
(x y)2(x y 2z) 0. 
Bài 17. Cho ,,a b c là các số thực dương. Chứng minh rằng 
a3 b3 c3 3abc ab 2(a2 b2) bc 2(b2 c2) ca 2(c2 a2). 
Chứng minh. 
Cách 1. Không mất tính tổng quát, giả sử c min{a, b, c}. Áp dụng bất 
đẳng thức AM-GM, ta có 
2 2 2 2 2 2 
2 2 2 2 
2 2 2 
2 2 
2( ) 2( ) 
2 2 
3 3 2 
, 
2 
b b c a c a 
b b c a c a 
a b c 
từ đó suy ra
41 
2 2 2 2 3 2 2 3 
2( ) 2( ) ( ). 
2 
bc b c ca c a c c a b 
Bài toán được quy về chứng minh 
3 3 3 2 2 3 2 2 3 
3 2( ) ( ), 
2 
a b c abc ab a b c c a b 
hay 
2a3 2b3 2ab 2(a2 b2) 3c(a b)2. 
Ta có 
3 3 2 2 
2 2 
3 3 2 2 2 2 
2 2 
2 2 2 
2 2 2 2( ) 
2( ) ( )( ) ( ) 2( ) 4 
2 
( )( ) ( ) 2( ) 4 . 
2 
a b ab a b 
a b 
a b a b a b a b a b ab 
a b 
a b a b a b a b ab 
Do 
2 2 
( )( )2 2 ( )2, 
2 
a b 
a b a b c a b c và 
(a b) 2(a2 b2) 4ab (a b)2 4ab (a b)2 0, 
nên hiển nhiên 
2 2 
( )( )2 ( ) 2( 2 2) 4 3 ( )2. 
2 
a b 
a b a b a b a b ab c a b 
Bài toán được chứng minh xong. 
Đẳng thức xảy ra khi và chỉ khi a b c. 
Cách 2. Không mất tính tổng quát, giả sử a b c. Sử dụng bất đẳng thức 
AM-GM, ta có 
2 2 2 2 2 2 2 2 2 2 
2 2 
2 2 
2 2( ) ( ) 2 , 2 2( ) ( ) 2 , 
( ) 
2 2( ) 2 . 
ab a b a a b ab bc b c c b c cb 
ca c a 
ca c a abc 
b 
Cộng ba bất đẳng thức trên lại theo vế, ta thu được 
2 2 
2 2 3 3 2 ( ) 
2 2( ) 3 ( ) 2 . 
ca c a 
ab a b a c b a c abc 
b 
Bài toán quy về chứng minh 
2 2 
3 3 3 3 3 2 ( ) 
2 2 2 6 3 ( ) 2 , 
ca c a 
a b c abc a c b a c abc 
b
42 
hay tương đương với 
2 2 
3 3 3 2 ( ) 
2 4 3 ( ) . 
ca c a 
a c b abc b a c 
b 
Bằng một vài tính toán đơn giản, ta có thể viết lại nó trở thành 
2 2 
( )( ) 2 0, 
a c 
a b b c b a c 
b 
đúng vì (a b)(b c) 0, 
2 2 2 
2 2 0. 
a c a 
b a c a b 
b b 
Cách 3. Áp dụng bất đẳng thức AM-GM, ta có 
2 2 2 
2 2 
2 
2 2 2 
2 2 
2 
2 2 
2 2 
2( ) ( ) 
2 2( ) 2 ( ) 
2 ( ) ( ) , 
2( ) ( ) 
2 2( ) 2 ( ) 
2 ( ) ( ) , 
2( ) 
2 2( ) 2 ( ) 
a b ab a b 
ab a b ab a b ab a b 
a b a b 
ab a b b a b 
b c bc b c 
bc b c bc b c bc b c 
b c b c 
bc b c c b c 
c a c 
ca c a ca c a ca c a 
c a 
2 
2 
( ) 
2 ( ) ( ) . 
a c a 
c a 
ca c a c c a 
Cộng ba bất đẳng thức trên lại theo vế, ta thu được 
2 2 2 2 2 
2 2 2 3 3 
2 2( ) 2 ( ) ( ) ( ) ( ) 
3 3 3 2 . 
ab a b ab a b b a b c b c c a c 
a b a c b c b c 
Do đó, ta chỉ cần chứng minh 
2a3 2b3 2c3 6abc 3a2b 3a2c 3b2c b3 2c3. 
Sau khi thu gọn, ta thu được bất đẳng thức hiển nhiên đúng 
(a b)2(2a b 3c) 0. 
Cách 4. Ta viết lại bất đẳng thức như sau 
3 3 3 2 2 
2 
2 
2 2 
3 ( ) 2( ) , 
1 ( ) 
( ) ( ) . 
2 2( ) 
a b c abc bc b c bc b c b c 
bc b c 
b c b c a 
b c b c 
Do 2(b2 c2) b c 2(b c) nên ta chỉ cần chứng minh
43 
( )2 ( )2 ( )2 0, a b c S b c S c a S a b 
trong đó 
2 
, a 
bc b 
S b c a c a 
b c b c 
, b S c S tương tự. 
Không mất tính tổng quát, giả sử . a b c Ta có 
2 2 
0, 0. b c 
c a 
S a b S b c 
c a a b 
Từ đó suy ra (chú ý rằng (a c)2 (b c)2 ) 
( )2 ( )2 ( )2 ( )2( ). a a b a b S b c S b c S c a b c S S 
Lại có 
2 
2 2 
1 1 
0, 
a b 
bc ca b a 
S S c c 
b c c a b c c a 
c 
b c c a 
nên kết hợp với trên, ta suy ra điều phải chứng minh. 
Bài 18. Cho ,,ab c là các số dương thỏa mãn 1. abc Chứng minh rằng 
2 2 2 
1 1 1 2 
1. 
(1 a) (1 b) (1 c) (1 a)(1 b)(1 c) 
Chứng minh. 
Cách 1. Đặt 2 2 
, 
yz zx 
a b 
x y 
và 2 
xy 
c 
z 
với x, y, z 0. Khi đó, bất đẳng 
thức cần chứng minh có thể viết lại thành 
4 4 4 2 2 2 
2 2 2 2 2 2 2 2 2 
2 
1. 
( ) ( ) ( ) ( )( )( ) 
x y z x y z 
x yz y zx z xy x yz y zx z xy 
Áp dụng bất đẳng thức Cauchy-Schwarz, ta thu được 
4 4 4 2 2 
2 2 2 2 2 2 2 2 2 2 2 2 
2 2 2 
2 2 2 2 2 2 
( ) 
( ) ( )( ) ( )( )( ) 
2 
1 . 
( )( )( ) 
x x x y z 
x yz x y x z x y y z z x 
x y z 
x y y z z x 
Do đó, bài toán quy về chứng minh 
(x2 y2)(y2 z2)(z2 x2) (x2 yz)(y2 zx)(z2 xy).
44 
Bất đẳng thức cuối này đúng vì ta có x2 yz (x2 y2)(x2 z2) theo 
bất đẳng thức Cauchy-Schwarz. Bài toán được chứng minh xong. 
Đẳng thức xảy ra khi và chỉ khi a b c 1. 
Cách 2. The nguyên lý Dirichlet, ta biết rằng trong ba số ,,a b c luôn có hai 
số hoặc cùng 1 hoặc cùng 1. Không mất tính tổng quát, ta giả sử hai số 
đó là a và ,b thế thì (1 )(1 ) 0. a b Sử dụng bất đẳng thức Cauchy- 
Schwarz, ta được 
2 2 
1 1 1 1 1 
. 
(1 ) (1 ) 1 
(1 ) 1 (1 ) 1 
a b a b ab 
ab ab 
b a 
(1) 
Mặt khác ta lại có 
(1 a)(1 b) 2(1 ab) (1 a)(1 b) 2(1 ab). (2) 
Từ (1) và (2) suy ra ta chỉ cần chứng minh 
2 
1 1 1 
1. 
1 ab (1 c) (1 ab)(1 c) 
Thay 
1 
ab 
c 
ta thu được một hằng đẳng thức đúng. 
Cách 3. Đặt 
1 1 1 1 
, 
1 2 1 2 
x y 
a b 
và 
1 1 
. 
1 2 
z 
c 
Ta có 
1 1 1 
1 , , 1, , , . 
1 1 1 
x y z 
x y z a b c 
x y z 
Do abc 1 nên (1 x)(1 y)(1 z) (1 x)(1 y)(1 z), suy ra 
x y z xyz 0. 
Như vậy, ta phải chứng minh 
(1 x)2 (1 y)2 (1 z)2 (1 x)(1 y)(1 z) 4 
với 1 x, y, z 1 và x y z xyz 0. 
Bất đẳng thức này tương đương với 
x2 y2 z2 2(x y z) xy yz zx 0. 
Do 
( )2 2 2 2 
2 
x y z x y z 
xy yz zx nên bất đẳng thức trên có 
thể viết lại thành x2 y2 z2 4(x y z) (x y z)2 0.
45 
Thay x y z xyz vào, ta được x2 y2 z2 x2y2z2 4xyz. Bất 
đẳng thức cuối này đúng theo AM-GM 
x2 y2 z2 x2y2z2 44 x4y4z 4 4 xyz 4xyz. 
Bài 19. Cho , , , , , a b c x y z là các số thực dương thỏa mãn 
ax by cz xyz. 
Chứng minh rằng 
x y z a b b c c a. 
Chứng minh. 
Cách 1. Giả thiết đã cho có thể được viết lại dưới dạng 1, 
a b c 
yz zx xy 
do đó 
1, 1, 1. 
a b b c a c 
yz zx zx xy yz xy 
Từ đây ta suy ra 
, , . 
b c a c a b 
x y z 
z y z x y x 
Sử dụng kết quả này kết hợp với bất đẳng thức am, ta được 
2( ) 
2 2 2 . 
b c a c a b 
x y z x y z 
z y z x y x 
a b b c c a 
z x y 
z x y 
a b b c c a 
Từ đây ta có điều phải chứng minh. 
Cách 2. Bất đẳng thức đã cho có thể dược viết dưới dạng thuần nhất như sau 
( )2( ) 2 
. 
x y z ax by cz 
a b b c c a 
xyz 
Không mất tính tổng quát, giả sử x max{x, y, z}. Bây giờ, sử dụng lần 
lượt các bất đẳng thức AM-GM và Cauchy-Schwarz, ta có
46 
2 
4 ( )( ) 2( )[ ( ) 2 2 ] 
2 2 
2( ) 2 2 2 . 
x y z ax by cz y z a y z by cz 
VT 
xyz yz 
a c a b 
y z a c a b 
y z 
Bài toán được quy về chứng minh 
2 2 
2 a 2c a 2b a b b c c a . 
Bất đẳng thức này đúng, vì theo bất đẳng thức Cauchy-Schwarz ta có 
2( 2 ) 2( 2 ). 
a b b c c a a b b a c c 
a b a c 
Cách 3. Giả thiết đã cho được có thể được dưới dạng 1. 
a b c 
yz zx xy 
Đặt a yzu, b zxv và c xyw. Ta quy được bài toán về chứng minh 
z(yu xv) x(zv yw) y(zu xw) x y z 
với u v w 1. 
Áp dụng bất đẳng thức Cauchy-Schwarz 
VT2 (x y z) (yu xv) (zv yw) (zu xw) , 
do vậy ta chỉ cần chứng minh 
x(v w) y(w u) z(u v) (x y z)(u v w). 
Bất đẳng thức này tương đương với xu yv zw 0, hiển nhiên đúng. 
Bài 20. (Chọn đội tuyển Moldova 2006) Cho ,,a b c là độ dài ba cạnh của 
một tam giác. Chứng minh rằng 
2 1 2 1 2 1 0. 
b c a 
a b c 
c a b 
Chứng minh. 
Cách 1. Bất đẳng thức cần chứng minh tương đương với 
2 2 2 
2 2 2. 
a b b c c a 
a b c 
c a b 
Sử dụng bất đẳng thức Cauchy-Schwarz, ta có
47 
2 2 2 2 2 2 2 2 2 2 2 
2 2 2 
( ) ( ) 
. 
( ) 
a b b c c a a b b c c a a b b c c a 
c a b a bc b ca c ab abc a b c 
Do đó, ta chỉ cần chứng minh được 
(a2b b2c c2a)2 abc(a b c)(a2 b2 c2). 
Không mất tính tổng quát, giả sử b là số hạng nằm giữa a và c. Khi đó, áp 
dụng bất đẳng thức AM-GM, ta được 
2 2 2 2 
2 2 2 [ ( ) ( )] 
( )( ) . 
4 
ac a b c b a b c 
abc a b c a b c 
Ta cần chứng minh 
2(a2b b2c c2a) ac(a b c) b(a2 b2 c2). 
Thu gọn và phân tích nhân tử, ta được 
(a b)(b c)(a b c) , . 
hiển nhiên đúng do ( )( ) 0 a b b c (theo giả thiết của b ) và a b c 
(theo giả thiết của đề bài). Vậy ta có điều phải chứng minh. 
Đẳng thức xảy ra khi và chỉ khi tam giác đã cho là tam giác đều. 
Cách 2. Nhân hai vế của bất đẳng thức với , abc ta viết được nó dưới dạng 
a3b2 b3c2 c3a2 abc(a2 b2 c2). 
Ta có 
3 2 3 3 2 2 2 3 3 
3 2 2 3 
2 2 
2 2 ( 2 ) ( ) 
( ) ( ) 
( ) ( ). 
a b a bc a b c bc a b c 
a b c a b c 
a b c a b c 
Mà ,,ab c là ba cạnh của một tam giác nên hiển nhiên , , a b c b c a 
c a b 0, từ đó suy ra a2(b c)2(a b c) 0, tức là 
a3b2 b3c2 c3a2 abc(a2 b2 c2). 
Cách 3. Không mất tính tổng quát, giả sử c max{a, b, c}. Khi đó, sử 
dụng bất đẳng thức AM-GM, ta có 
2 
2 1 2 2 2. 
c b c 
b ca ca b bc ca b 
a a 
Và như thế, ta chỉ cần chứng minh 
( ) 2 2 2 2 1 0. 
b c a 
f a a bc ca b c 
c b
48 
Ta thấy f (a) là một tam thức bậc hai và hệ số cao nhất của nó không 
dương. Vì vậy ()fa sẽ đạt cực tiểu tại biên. Mà c b a c nên 
f (a) min{f (c b), f (c)}. 
Tính toán trực tiếp, ta dễ thấy 
( )3 ( )4 
( ) 0, ( ) 0. 
c b b c 
f c f c b 
b bc 
Từ đó suy ra ( ) 0. fa Bài toán được chứng minh xong. 
Cách 4. Đặt 
11 
, ab 
xy 
và 
1 
c . 
z 
Bất đẳng thức cần trở thành 
2 2 2 
1 1 1 
1 1 1 0, 
z x y 
x y y z z x 
tương đương 
E(x, y, z) yz2(z y) zx2(x z) xy2(y x) 0. 
Không mất tính tổng quát, giả sử a min{x, y, z}, khi đó từ phép đặt ẩn 
suy ra x max{x, y, z}. Ta sẽ chứng minh 
E(x, y, z) E(y, y, z). 
Ta có 
3 3 2 2 2 3 2 2 2 
2 
( , , ) ( , , ) ( ) ( ) ( ) ( ) 
( )( )( ). 
E x y z E y y z z x y z x y y x y y x y 
x y x z xz yz y 
Do (x y)(x z) 0 và 
2 
2 2 
2 ( ) ( ) 
(2 ) 0 
b c b a a b c 
xz yz y y z y 
b c b c 
nên hiển nhiên E(x, y, z) E(y, y, z). Mặt khác, ta lại có 
E(y, y, z) yz(y z)2 0, 
nên bài toán được chứng minh xong. 
Bài 21. Chứng minh rằng với mọi số dương a, b, c, ta luôn có 
1 1 1 1 
1 1 1 1 
1 1 
1 1 3. 
a b b c 
b c c a 
c a 
a b
49 
Chứng minh. 
Cách 1. Đặt 
1 1 1 
x a 1, y b 1, z c 1 
b c a 
thì bất đẳng thức 
của ta trở thành 
xy yz zx 3. 
Theo nguyên lý Dirichlet, ta thấy rằng tồn tại ít nhất hai số trong ba số 
,,x y z sao cho hiệu của chúng với 1 cùng dấu. Giả sử hai số đó là x và ,y 
khi đó ta có ( 1)( 1) 0, x y suy ra 1.xyx y Vì vậy, ta chỉ cần 
chứng minh được 
(x y 1) z(x y) 3 hay 
4 
. 
1 
x y 
z 
Bất đẳng thức này hiển nhiên đúng vì 
1 1 
1 1 1 4 4 
2 . 
1 1 1 
a c 
c a 
x y a b a 
c b c z 
c c 
a a 
Phép chứng minh của ta được hoàn tất. 
Đẳng thức xảy ra khi và chỉ khi x y z 1. 
Cách 2. Bằng phép khai triển trực tiếp, ta có thể dễ dàng viết lại bất đẳng 
thức cần chứng minh dưới dạng 
1 2 
2 1 3 0. 
b 
a 
a 
ab 
abc abc 
Đặt 3 k abc 0, ta thấy tồn tại các số dương x, y, z sao cho , 
kx 
a 
y 
, . 
ky kz 
b c 
z x 
Với phép đổi biến này, bất đẳng thức trên trở thành 
2 
2 
2 
1 2 
2 3 0, 
x x y 
k k 
yz k y k x 
tương đương 
3 2 2 2 
2 
2 1 
f (k) x k x y 2k xy 3xyz 0. 
k k
50 
Ta có 
3 
2 2 
3 
2( 1) 
( ) , 
k 
f k k x y xy 
k 
phương trình ( ) 0 fk chỉ 
có một nghiệm dương duy nhất là 
2 
0 2 
. 
xy 
k 
xy 
Ngoài ra, ta thấy rằng qua 
0 k thì ()fk đổi dấu từ âm sang dương nên đây cũng chính là giá trị cực tiểu 
của ( ),fk tức là 0 ( ) ( ) f k f k với mọi 0.k Vì vậy, để chứng minh bất 
đẳng thức đã cho, ta chỉ cần chứng minh được 
2 
2 
0, 
xy 
f 
x y 
tương đương 
2 2 2 2 2 2 2 2 
3 3 3 
2 2 2 2 2 2 
( ) ( ) 
3 . 
x y y z z x xy yz zx 
x y z xyz 
xy yz zx x y y z z x 
Để ý rằng 
2 2 
2 1 1 
( ) ( ) 
m n 
m n m n 
n m m n 
với mọi m, n 0 và 
x 3 3xyz xy(x y) (x y z)(x y)2 z(x z)(y z), 
ta có thể viết lại bất đẳng thức cần chứng minh như sau 
2 
2 2 2 
2 2 
( )( ) ( )( ) 
1 1 
( ) ( ) ( ) . 
x y z x y z x z y z 
x y y z x z 
x y xy 
Không mất tính tổng quát, giả sử z min{x, y, z}. Khi đó, ta có 
2 2 2 2 2 
2 2 2 2 
( ) ( ) ( ) 
( )( ) 0, , 
( ) 
1 1 1 1 
, 
x z y z xy x z x y 
z x z y z 
x y z x z y x y 
x y xy x y xy 
nên 
2 2 
2 2 
2 2 
1 1 ( ) 
( ) ( )( ) . 
x y x y z 
VP x y x y z x y VT 
x y xy x y 
Bất đẳng thức được chứng minh xong. 
Cách 3. Không mất tính tổng quát, ta có thể giả sử c là số hạng nằm giữa a 
và b. Khi đó, ta có đánh giá sau
51 
1 1 1 1 ( )( ) 
1 1 1 1 0. 
a c c b 
b c c b 
c a c a ac 
Do đó, bất đẳng thức đã cho sẽ được chứng minh nếu bất đẳng thức mạnh 
hơn đúng 
1 1 1 1 
1 1 1 1 
1 1 
1 1 3, 
a b c a 
b c a b 
c b 
c a 
tức là 
1 1 1 1 1 
a 1 b 1 c 1 a b 2 3. 
b a c a b 
Theo bất đẳng thức AM-GM, 
1 
c 1 1, 
c 
1 1 
a b 2 0, 
a b 
suy ra 
1 1 1 1 1 
c 1 a b 2 a b 2. 
c a b a b 
Vì vậy, ta chỉ cần chứng minh được 
1 1 1 1 
a 1 b 1 a b 2 3 
b a a b 
hay 
1 1 
a b 4. 
b a 
Theo bất đẳng thức AM-GM, dễ thấy bất đẳng thức cuối hiển nhiên đúng. 
Từ đó, ta có điều phải chứng minh. 
Bài 22. Cho a, b, c, x, y, z là các số thực dương thỏa mãn 
a b c x y z. 
Chứng minh rằng 
ax2 by2 cz2 xyz 4abc. 
Chứng minh. 
Cách 1. Đặt , 
2 2 
z x x y 
p b q c và . 
2 
y z 
r a Dễ thấy 
trong ba số này luôn có hai số cùng dấu với nhau. Không mất tính tổng quát, 
giả sử pq 0. Ta có 
, , . 
2 2 2 
z x x y y z 
b p c q a x y z b c p q 
Thay vào bất đẳng thức đã cho, ta được
52 
2 2 2 2 2 
2 
2 2 
2 
4 
2 2 
4 
2 2 2 2 
4 ( ) 2 ( ) 2 ( ) 4 
4 
2 
y z x z 
ax by cz xyz abc p q x p y 
x y y z x z x y 
q z xyz p q p q 
pq p q p x y q x z pqx 
x z 
q p 4 2 4 4( 2 2 ) 0. 
2 
x y 
p q pqx q b p c pqx 
Bài toán được chứng minh xong. Ta dễ thấy đẳng thức xảy ra khi và chỉ khi 
p q r 0, tức , , . 
2 2 2 
y z z x x y 
a b c 
Cách 2. Ta thấy nếu x2 4bc thì ax2 4abc, suy ra 
ax2 by2 cz2 xyz ax2 4abc. 
Bất đẳng thức hiển nhiên đúng. Vì vậy, không mất tính tổng quát, ta chỉ cần 
xét x2 4bc. Đặt 
u a b c x y z. 
Thay z u x y và a u b c vào, sau khi thu gọn, ta viết được bất 
đẳng thức dưới dạng 
2 2 2 
2 2 
( ) ( 4 ) 2 ( ) ( )( 4 ) 
( ) ( ) 0. 
f u cu x bc c x y xy u b c x bc 
by c x y xy x y 
Ta thấy ()fu là 1 tam thức bậc hai của u với hệ số cao nhất dương. Lại có 
( 2 4 )(2 )2 0, f x bc c x y 
nên hiển nhiên f (u) 0. Chứng minh hoàn tất. 
Cách 3. Dễ thấy kết quả bài toán được suy ra từ hằng đẳng thức sau 
2 2 2 
2 2 2 
2( )( 4 ) 
( ) ( ) ( ) 2 ( ), 
yzu zxv xyw ax by cz xyz abc 
xu v w yv w u zw u v uvw x y z a b c 
trong đó u 2ax yz, v 2by zx và w 2cz xy. 
Cách 4. Giả sử bất đẳng thức đã cho sai, tức 
ax2 by2 cz2 xyz 4abc. 
Đặt ax2 by2 cz2 xyz 4kabc với 0 k 1. Dễ thấy 
x2 4kbc, y2 4kca, z2 4kab,
53 
suy ra tồn tại , , 0, 
2 
A B C sao cho 
cos , cos , cos . 
2 2 2 
x y z 
A B C 
kbc kca kab 
Thay vào ax2 by2 cz2 xyz 4kabc, ta được 
cos2 A cos2 B cos2C 2cosAcosBcosC 1, 
suy ra ,,A B C là ba góc của một tam giác. Đến đây, sử dụng bất đẳng thức 
cơ bản trong tam giác: Nếu ,,A B C là ba góc của một tam giác và ,,m n p 
là các số thực, thì 
2np cosA 2pmcosB 2mn cosC m2 n2 p2, 
ta được 
2 cos 2 cos 2 cos 
( ) . 
x y z k bc A ca B ab C 
k a b c a b c 
Điều này mâu thuẫn với giả thiết của bài toán. Vậy điều giả sử ở trên là sai. 
Ta có điều phải chứng minh. 
Cách 5. Ta có (b c x) (c a y) (a b z) a b c 0. Do 
vậy, trong các số b c x, c a y, a b z phải có ít nhất một số 
dương. Không mất tính tổng quát, ta giả sử b c x 0. Bây giờ, ta thấy 
rằng trong trường hợp x2 4bc thì kết quả bài toán là hiển nhiên nên ở đây 
ta sẽ xét 4bc x2. Ta có bất đẳng thức đã cho tương đương với 
2ax2 2by2 2cz 2 x (y z)2 y2 z 2 8abc, 
2ax2 (2b x)y2 (2c x)z2 x(y z)2 8abc. 
Với mọi số thực p, q, u, v thỏa mãn p q 0, ta có 
2 2 
2 2 ( ) ( ) 
0. 
pq u v pu qv 
pu qv 
p q p q 
Sử dụng kết quả này với chú ý rằng (2b x) (2c x) 2(b c x) 0, 
ta được 
2 
2 2 (2 )(2 )( ) 
(2 ) (2 ) . 
2( ) 
b x c x y z 
b x y c x z 
b c x 
Như vậy ta chỉ còn phải chứng minh
54 
2 
2 2 (2 )(2 )( ) 
2 ( ) 8 . 
2( ) 
b x c x y z 
ax x y z abc 
b c x 
Bất đẳng thức này tương đương với 
2 2 (2 )(2 ) 
( ) 2 (4 ), 
2( ) 
b x c x 
y z x a bc x 
b c x 
2 2 
2 ( ) (4 ) 
2 (4 ). 
2( ) 
y z bc x 
a bc x 
b c x 
Do 2 40bc x nên ta có thể viết lại bất đẳng thức cuối thành 
(y z)2 4a(b c x), 
đúng theo vì AM-GM, (y z)2 (a b c x)2 4a(b c x). 
Bài 23. Cho ,,a b c là các số thực không âm. Chứng minh rằng 
2 2 2 3 
( ) . 
2 
a b c a bc b ca c ab 
Chứng minh. 
Cách 1. Không mất tính tổng quát, giả sử a b c. Nếu b c 0 thì bất 
đẳng thức hiển nhiên đúng. Vì vậy ta chỉ cần xét trường hợp 0 b là đủ. 
Khi đó, sử dụng bất đẳng thức AM-GM, ta có 
2 2 
2 2 
2 
2 
2 , 2 , 
2 . 
a bc b ca 
a bc a c b ca b c 
a c b c 
c ab 
c ab b c 
b c 
Suy ra, ta chỉ cần chứng minh được 
2 2 2 ( ) 
3( ) 2 3 , 
a bc b c a b c 
a b c a b c 
a c b c 
hay là 
2 2 2 
. 
a bc b c 
a b 
a c b c 
Bất đẳng thức này hiển nhiên đúng do 
a2 bc a2 ac 
a 
a c a c 
và 
2 2 2 
. 
b c b bc 
b 
b c b c
55 
Vây ta có điều phải chứng minh. Với giả thiết a b c, ta có đẳng thức 
xảy ra khi và chỉ khi , 0. a b c 
Cách 2. Giả sử a b c. Ta dễ thấy 2 2 
2 
c 
a bc a ac a và 
Cauchy - Schwarz b2 ca c2 ab 2(b2 c2 ab ac) ( ), 
nên bất đẳng thức của ta được suy ra từ 
2 2 3 2 
2( ) . 
2 
a b c 
b c ab ac 
Bình phương hai vế, ta có thể viết lại bất đẳng thức này dưới dạng 
a2 b2 4c2 2ab 12bc 4ca 0, 
(a b 2c)2 8c(b c) 0 (đúng). 
Bất đẳng thức được chứng minh xong. 
Cách 3. Giả sử a b c. Khi đó, bất đẳng thức Cauchy-Schwarz cho ta 
b2 ca c2 ab 2(b2 c2 ab ac). 
Do đó, ta chỉ cần chứng minh được 
2 2 2 3 
2( ) ( ). 
2 
a bc b c ab ac a b c 
Đặt ( ) 
2 
b c 
s s a và , p bc bất đẳng thức trên được viết lại thành 
2 a2 p 4 2s2 p as 3(a 2s), 
tương đương 
4 2s2 p as 3(a 2s) 2 a2 p. 
Bình phương hai vế và thu gọn, ta thu được bất đẳng thức tương đương là 
12(a 2s) a2 p 13a2 20as 4s2 20p, 
hay 
12(a 2s) a2 p a (a 2s)2 20p. 
Do (a 2s)2 0, 2 
2 2 
p p 
a p a 
a p a a 
nên ta chỉ cần chứng 
minh được
56 
6( 2 ) 
20 
a s p 
p 
a 
hay (6 7 ) 0. p s a 
Bất đẳng thức cuối này hiển nhiên đúng do . as 
Cách 4. Bằng cách sử dụng phép bình phương, ta có thể viết lại bất đẳng 
thức cần chứng minh dưới dạng 
5 a2 14 ab 8 (a2 bc)(b2 ca). 
Áp dụng bất đẳng thức Cauchy Schwarz, ta có 
2 
(a2 bc)(b2 ca) 3 (a2 bc)(b2 ca). 
Do đó, ta chỉ cần chứng minh 
2 
5 a2 14 ab 192 (a2 bc)(b2 ca), 
hay là 
25 a4 54 a2b2 340abc a 52 ab(a2 b2) 0. 
Để ý rằng 
2 
a2 2 ab a4 4 ab(a2 b2) 6 a2b2 4abc a, 
do đó 
2 
a4 a2 2 ab 4 ab(a2 b2) 6 a2b2 4abc a. 
Ta viết được bất đẳng thức trên lại thành 
2 
25 a2 2 ab 48 ab(a2 b2) 96 a2b2 240abc a 0, 
2 
25 a2 2 ab 48 ab(a b)2 240abc a 0 (đúng). 
Bất đẳng thức được chứng minh xong. 
Bài 24. Cho a, b, c là các số dương thỏa mãn a b c 1. Chứng minh 
a (b c)2 b (c a)2 c (a b)2 3. 
Chứng minh. 
Cách 1. Để ý rằng 
( ) 
3 3, 
2( ) 
a b c 
ab bc ca 
nên ta chỉ cần chứng minh 
mỗi hạng tử của a (b c)2 không nhỏ hơn mỗi hạng tử của tổng 
trên là được, tức là
57 
2 3 ( ) 
( ) . 
2( ) 
a b c 
a b c 
ab bc ca 
Ta thấy bất đẳng thức này tương đương với 
2 2 2 
2 2 
( ) 3 ( ) 
. 
( ) 4( ) 
a b c a b c 
a b c a b c ab bc ca 
Đây là một bất đẳng thức đồng bậc cho , , , a b c nên ta có thể bỏ qua giả thiết 
1 a b c để chuẩn hóa cho 1. bc Bây giờ với phép chuẩn hóa 
này và bằng phép đặt , x bc ta có thể viết lại bất đẳng thức trên dưới dạng 
2 
2 2 
1 4 3 
( ) 0. 
1 ( 1) 4( ) 
a x a 
f x 
a a a x 
Dễ thấy f (x) là một hàm lõm trên (do đạo hàm cấp hai của nó âm), nên 
nó có tính chất: Với mọi x [a, b] thì 
f (x) min{f (a), f (b)}. 
Mặt khác, từ phép chuẩn hóa và cách đặt trên, ta dễ dàng kiểm tra được 
1 
0 , 
4 
x vì thế 
1 
( ) min (0), . 
4 
f x f f 
Mà 
2 
2 
( 1) 
(0) 0 
4( 1) 
a 
f 
a 
và 
2 
2 
1 (2 1) 
0 
4 ( 1)(4 1) 
a a 
f 
a a 
nên hiển nhiên 
ta có f (x) 0. Và như thế, bất đẳng thức của ta được chứng minh xong. 
Dễ thấy đẳng thức xảy ra khi và chỉ khi 
1 
. 
3 
a b c 
Cách 2. Không mất tính tổng quát, ta có thể giả sử c min{a, b, c}. Do 
a b c 1 nên 
1 
0 . 
3 
c Ta có c (a b)2 c và 
a (b c)2 b (a c)2 2(a b) (a b 2c)2 
(tương đương (a b)2(3 8c) 0 đúng do 
1 
0 
3 
c ). 
Từ hai đánh giá này, ta đưa được bài toán về chứng minh
58 
c 2(1 c) (1 3c)2 3 . 
Bằng một số phép tính đơn giản, ta thấy bất đẳng thức này tương đương với 
c(1 3c)2(4 3c) 0 (hiển nhiên đúng). 
Cách 3. Bình phương hai vế và thu gọn (với chú ý a b c 1), ta có thể 
viết lại bất đẳng thức dưới dạng 
a (b c)2 b (a c)2 3(ab bc ca). 
Giả sử , a b c khi đó áp dụng bất đẳng thức Cauchy-Schwarz, ta có 
2 
2 
2 
2 
2 
( )( ) 
( )( ) ( )( ) ( )( ) 
1 ( ) ( )( ) 
1 ( ) ( )( ) 
1 ( ) ( )( ) . 
VT ab a c b c 
ab bc ca a c b c a b b c a b a c 
a c a b b c ab bc ca a b c 
a c a b b c a c a b b c 
a c a b b c a c 
Như thế, ta cần chứng minh 
2 
1 (a c)2 (a b)(b c) a c 3(ab bc ca). 
Do (a b c)2 3(ab bc ca) (a c)2 (a b)(b c) nên bất đẳng 
thức trên tương đương với 
2 
2(a c)2 a c 0 hay 
2 
2 a c 1 0. 
Ta có 
2 
2 a c 1 2(a c) (a b c) (a b) c 0, do đó 
bất đẳng thức trên hiển nhiên . Phép chứng minh hoàn tất. 
Bài 25. Cho các số thực không âm a, b, c, d. Chứng minh rằng 
a4 b4 c4 d4 2abcd a2b2 a2c2 a2d2 b2c2 b2d2 c2d2. 
Chứng minh. 
Cách 1. Không mất tính tổng quát, giả sử a b c d. Xét hàm số sau 
( ) 4 4 4 4 2 2 2, 
sym 
f a a b c d abcd a b 
Tính đạo hàm đến cấp hai của f (a), ta có
59 
f (a) 4a3 2bcd 2a(b2 c2 d2), 
f (a) 12a2 2(b2 c2 d2) 0. 
Từ f (a) 0, ta suy ra f (a) đồng biến và như thế, ta được 
3 2 2 2 
2 2 
( ) ( ) 4 2 2 ( ) 
2 ( ) ( ) 0, 
f a f b b bcd b b c d 
b b c d c d 
Vậy ()fa là hàm đồng biến với mọi ab nên 
4 4 4 2 2 2 2 2 2 
2 2 2 2 2 
( ) ( ) 2 2( ) 
( ) (2 )( ) 0. 
f a f b b c d b cd c d c d b 
b c d b cd d c d 
Bất đẳng thức của ta được chứng minh xong. 
Với giả thiết a b c d 0, ta dễ thấy đẳng thức xảy ra khi và chỉ khi 
a b c d hoặc a b c, d 0. 
Cách 2. Do tính đối xứng nên ta có thể giả sử a b c d, khi đó với 
chú ý rằng 
a4 b4 c4 d4 4abcd (a2 b2)2 (c2 d2)2 2(ab cd)2, 
và 
3 4 2 2 2 ( 2 2)2, 
sym sym 
a a b a b 
ta có thể viết lại bất đẳng thức cần chứng minh như sau 
(a2 c2)2 (b2 c2)² (a2 d2)2 (b2 d2)2 2(ab cd)2. 
Bây giờ, áp dụng các bất đẳng thức Cauchy-Schwarz và AM-GM, ta có 
2 2 2 2 2 2 
2 2 2 2 2 2 
2 
2 
( 2 ) (2 2 ) 
( ) ( ) 
2 2 
(2 2 ) 
2( ) , 
2 
a b d ab d 
a d b d 
ab cd 
ab cd 
nên bất đẳng thức trên là hiển nhiên. Phép chứng minh của ta được hoàn tất. 
Cách 3. Giả sử a b c d. Bất đẳng thức đã cho có thể viết lại thành 
a4 b4 c4 a2b2 b2c2 c2a2 d d(a2 b2 c2 d2) 2abc , 
hay tương đương 
(a2 b2)2 (b2 c2)2 (c2 a2)2 2d d(a2 b2 c2 d2) 2abc . 
Do 2 2 2 2 
, , , , 
( ) 2 ( )( ) 
a b c a b c 
a b d a b a b nên ta chỉ cần chứng minh
60 
2 2 2 
2 2 2 2 
( )( ) ( )( ) ( )( ) 
( ) 2 , 
a b a b b c b c c a c a 
d a b c d abc 
hay 
3 3 3 
2 2 2 2 
2( ) ( ) ( ) ( ) 
( ). 
a b c abc ab a b bc b c ca c a 
d a b c d 
Áp dụng bất đẳng thức Schur bậc 3, ta có 
ab(a b) bc(b c) ca(c a) a3 b3 c3 3abc, 
nên để chứng minh bất đẳng thức cuối, ta chỉ cần chứng minh 
a3 b3 c3 abc d(a2 b2 c2 d2). 
Bây giờ, sử dụng bất đẳng thức Chebyshev, ta thu được 
3 3 3 2 2 2 1 
( )( ). 
3 
a b c a b c a b c 
Do đó, ta cần chứng minh 
2 2 2 2 2 2 2 1 
( )( ) ( ), 
3 
a b c a b c abc d a b c d 
hay là 
(a2 b2 c2)(a b c 3d) 3(abc d3). 
Áp dụng bất đẳng thức AM-GM, ta có 
3 3 2 2 2 3 2 3 2 2 2 3 
2 2 2 2 2 2 
3 9 
3( ) ( )( 3 ), 
3 
VP abc d a b c d abc d a b c abc d 
a b c 
a b c d a b c a b c d 
nên bất đẳng thức trên là hiển nhiên đúng. Bài toán được chứng minh xong. 
Cách 4. Không mất tính tổng quát, giả sử a b c d. Khi đó, đặt 
4 2 2 2 2 2 2 
, , , , , , 
2 2 2 2 
, , , , , , 
4 2 
( )( ), 
( )( ), 
2 ( ), 
a b c a b c a b c 
a b c a b c a b c 
A a a b a b a c 
B d ab d a d a b a c 
C d abcd d ab bc ca 
ta dễ thấy
61 
4 4 4 4 2 2 2 2 2 2 2 2 2 2 2 2 
2 2 2 2 2 
, , , , 
2 2 
, , , , 
2 ( ) 
( )( ) ( )( ) 
( )( )( ) ( ) ( )( ) . 
a b c a b c 
a b c a b c 
a b c d abcd a b a c a d b c b d c d 
A B C a b a c d a b a c C 
a d a b a c ab bc ca a b a c C 
Vì a2 d2 b2 d2 c2 d2 0 nên 
2 2 
, , 
2 2 2 2 
2 2 2 2 
2 2 2 
( )( )( ) 
( )( )( ) ( )( )( ) 
( )( )( ) ( )( )( ) 
( )( ) 0. 
a b c 
a d a b a c 
a d a b a c b d b c b a 
b d a b a c b d b c b a 
b d a b 
Lại có 2 2 2 
, , 
( )( ) 0, 
a b c 
a b a c a b c ab bc ca nên để chứng 
minh bất đẳng thức đã cho, ta chỉ cần chứng minh được 0, C tức là 
d3 2abc d(ab bc ca) 0. 
Ta có 
3 3 
3 
2 ( ) (2 ) 
(2 ) 
( )( ) 0, 
d abc d ab bc ca d dab c ab ad bd 
d dab d ab ad bd 
d d a d b 
nên bất đẳng thức 0 C là hiển nhiên. Phép chứng minh hoàn tất. 
Bài 26. (Olympic Toán Hoa Kỳ 2003) Cho ,,a b c là các số thực dương. 
Chứng minh rằng 
2 2 2 
2 2 2 2 2 2 
(2 ) (2 ) (2 ) 
8. 
2 ( ) 2 ( ) 2 ( ) 
a b c b c a c a b 
a b c b c a c a b 
Chứng minh. 
Cách 1. Bất đẳng thức cần chứng minh tương đương với 
2 2 2 
2 2 2 2 2 2 
2 2 2 
2 2 2 2 2 2 
(2 ) (2 ) (2 ) 
3 3 3 1, 
2 ( ) 2 ( ) 2 ( ) 
2( ) 2( ) 2( ) 
1. 
2 ( ) 2 ( ) 2 ( ) 
a b c a b c a b c 
a b c a b c a b c 
b c a c a b a b c 
a b c b c a c a b 
Sử dụng bất đẳng thức Cauchy-Schwarz, ta có (b c)2 2(b2 c2), suy ra
62 
2 2 2 
2 2 2 2 2 2 2 2 
2( ) 2( ) ( ) 
. 
2 ( ) 2 2( ) 
b c a b c a b c a 
a b c a b c a b c 
Tương tự, ta cũng có 
2 2 2 2 
2 2 2 2 2 2 2 2 2 2 
2( ) ( ) 2( ) ( ) 
, . 
2 ( ) 2 ( ) 
c a b c a b a b c a b c 
b c a a b c c a b a b c 
Sử dụng các đánh giá này, ta đưa được bài toán về chứng minh 
(b c a)2 (c a b)2 (a b c)2 a2 b2 c2. 
Sau khi khai triển và rút gọn, ta thu được bất đẳng thức hiển nhiên đúng 
a2 b2 c2 ab bc ca. 
Bài toán được chứng minh xong. 
Đẳng thức xảy ra khi và chỉ khi a b c. 
Cách 2. Không mất tính tổng quát, ta có thể giả sử a b c 3. Khi đó, 
bất đẳng thức cần chứng minh được viết lại thành 
2 2 2 
2 2 2 2 2 2 
( 3) ( 3) ( 3) 
8. 
2 (3 ) 2 (3 ) 2 (3 ) 
a b c 
a a b b c c 
Ta có 
2 2 
2 2 2 2 
2 
( 3) 6 9 1 8 6 
2 (3 ) 3 6 9 3 3( 2 3) 
1 8 6 1 8 6 4 4 
. 
3 3 ( 1) 2 3 3 2 3 3 
a a a a 
a a a a a a 
a a 
a 
a 
Tương tự 
2 2 
2 2 2 2 
( 3) 4 4 ( 3) 4 4 
, . 
2 (3 ) 3 3 2 (3 ) 3 3 
b c 
b c 
b b c c 
Cộng ba bất đẳng thức này lại theo vế, ta được 
2 2 2 
2 2 2 2 2 2 
( 3) ( 3) ( 3) 4 
( ) 4 8. 
2 (3 ) 2 (3 ) 2 (3 ) 3 
a b c 
a b c 
a a b b c c 
Bài toán được chứng minh xong. 
Cách 3. Tương tự như cách 1, ta phải chứng minh 
2 2 2 
2 2 2 2 2 2 
( ) ( ) ( ) 1 
. 
2 ( ) 2 ( ) 2 ( ) 2 
b c a c a b a b c 
a b c b c a c a b 
Sử dụng bất đẳng thức Cauchy-Schwarz, ta có
63 
2 2 2 2 2 2 2 
2 2 
( ) 
2 ( ) ( ) . 
2 ( ) 
b c a 
b a b c b b c a a 
a b c 
Do đó ta chỉ cần chứng minh 
2(a2 b2 c2)2 b2 2a2 (b c)2 . 
Bất đẳng thức này tương đương với 
a4 b4 c4 a2b2 b2c2 c2a2 2(a3b b3c c3a), 
hiển nhiên đúng vì a4 a2b2 2a3b, b4 b2c2 2b3c, c4 c2a2 2c3a. 
Cách 4. Cũng như hai cách 1 và 3, ta biến đổi bất đẳng thức về dạng 
2 2 2 
2 2 2 2 2 2 
( ) ( ) ( ) 1 
. 
2 ( ) 2 ( ) 2 ( ) 2 
b c a c a b a b c 
a b c b c a c a b 
Đặt , 
b c c a 
x y 
a b 
và . 
a b 
z 
c 
Bất đẳng thức trên trở thành 
2 2 2 
2 2 2 
( 1) ( 1) ( 1) 1 
. 
2 2 2 2 
x y z 
x y z 
Áp dụng bất đẳng thức Cauchy-Schwarz, ta được 
2 2 2 2 
2 2 2 2 2 2 
( 1) ( 1) ( 1) ( 3) 
. 
2 2 2 6 
x y z x y z 
x y z x y z 
Bài toán được quy về chứng minh 
2(x y z 3)2 x2 y2 z2 6. 
Bất đẳng thức này tương đương với 
2(x2 y2 z2 2xy 2yz 2zx 6x 6y 6z 9) x2 6, 
hay là 
(x y z)2 2(xy yz zx) 12(x y z) 12 0. 
Do xy yz zx 33 x2y2z2 12 nên ta chỉ cần chứng minh 
(x y z)2 24 12(x y z) 12 0. 
Bất đẳng thức này tương đương với bất đẳng thức hiển nhiên đúng 
(x y z 6)2 0. 
Bài toán được chứng minh xong. 
Bài 27. Chứng minh rằng với mọi a, b, c dương, ta có
64 
2 2 2 
3. 
a b c 
a b b c c a 
Chứng minh. 
Cách 1. Sử dụng bất đẳng thức Cauchy-Schwarz, ta có 
2 
2 
2( ) 
( )( ) 
8( )( ) 
. 
( )( )( ) 
a a 
a c 
a b a b a c 
a b c ab bc ca 
a b b c c a 
Từ đó, bài toán được quy về chứng minh 
8(a b c)(ab bc ca) 9(a b)(b c)(c a). 
Để ý rằng (a b c)(ab bc ca) (a b)(b c)(c a) abc, do đó 
bất đẳng thức trên trên có thể viết lại thành 
8abc (a b)(b c)(c a) (đúng theo AM-GM). 
Bài toán được chứng minh xong. 
Đẳng thức xảy ra khi và chỉ khi a b c. 
Cách 2. Áp dụng bất đẳng thức AM-GM, ta có 
2 3 ( ) 2( ) 
4( ) 3( )( ) 
3 ( ) 3( ) 1 
4( ) 3 ( )( ) 
3 4( )( ) 3 4 4 
2 3( )( )( ) 2 3 3( )( )( ) 
3 4 4 
3. 
2 3 3 2 2 2 
a a b c ab bc ca 
a b ab bc ca a b b c 
a b c ab bc ca 
ab bc ca a b b c 
ab bc ca a b c abc 
a b b c c a a b b c c a 
abc 
ab bc ca 
Cách 3. Đặt 2, 2, 2 
b c a 
x y z 
a b c 
với x, y, z 0. Từ phép đặt, dễ thấy 
xyz 1. Bất đẳng thức cần chứng minh trở thành 
2 2 2 
2 2 2 
3. 
1 x 1 y 1 z 
Không mất tính tổng quát, giả sử x max{x, y, z}, suy ra yz 1 x. 
Ta sẽ chứng minh
65 
2 2 
2 2 2 2 
2 2 . 
1 1 1 1 
x 
y z yz x 
Thật vậy, theo bất đẳng thức Cauchy-Schwarz, ta có 
2 2 2 2 2 2 
2 2 2 2 1 1 
2 2 . 
1 y 1 z 1 y 1 z 1 y 1 z 
Mặt khác, 
2 
2 2 2 2 
1 1 2 ( 1)( ) 
0 
1 1 1 (1 )(1 )(1 ) 
yz y z 
y z yz y z yz 
nên 
kết hợp với trên, ta suy ra điều khẳng định. Bây giờ, từ khẳng định vừa 
chứng minh, ta đưa được bài toán về chứng minh 
2 
2 2 
2 3. 
1 1 
x 
x x 
Do 2 
2 2 
1 x 1 x 
(theo Cauchy-Schwarz) nên ta chỉ cần chứng minh 
2 2 
2 3 
1 1 
x 
x x 
hay 2 2x(1 x) 1 3x. 
Bất đẳng thức cuối đúng vì theo AM-GM, ta có 
2 2x(1 x) 2x (1 x) 1 3x. 
Cách 4. Đặt 
1 1 1 
, , . 
2 2 2 
a x b y c z 
a b b c c a 
Dễ thấy 
1 1 1 
1 , , 1, , , . 
1 1 1 
b x c y a z 
x y z 
a x b y c z 
Do 1 
b c a 
a b c 
nên (1 x)(1 y)(1 z) (1 x)(1 y)(1 z), suy ra 
x y z xyz 0. 
Vậy ta phải chứng minh 
1 x 1 y 1 z 3 
với 1 x, y, z 1 và x y z xyz 0. 
Nếu x y z 0, thì theo bất đẳng thức Cauchy-Schwarz ta có 
1 x 1 y 1 z 3(3 x y z) 3.
66 
Xét trường hợp x y z 0. Lúc này xyz 0, suy ra tồn tại một số âm 
trong ba số , , . x y z Giả sử 0,z khi đó dễ thấy (0). , ,1xy Sử dụng bất 
đẳng thức Cauchy-Schwarz, ta được 
1 x 1 y 2(2 x y). 
Bài toán được quy về chứng minh 
2(2 x y) 1 z 3. 
Bất đẳng thức này tương đương với 
2x 2y 4 2 1 1 z hay 
2( ) 
. 
2 2 2 4 1 1 
x y z 
x y z 
Do x y z(1 xy) nên bất đẳng thức trên có thể viết lại thành 
2 (1 ) 
, 
2 2 2 4 1 1 
z x z 
x y z 
tương đương 
2xy 2(1 xy) z 1 2x 2y 4. 
Do 
(1 )(1 ) 
1 
1 
x y 
z 
xy 
nên ta có 
(1 )(1 )(1 ) 1 . 
2 
x y 
xy x y xy 
Bất đẳng thức cuối đúng theo bất đẳng thức Cauchy-Schwarz 
2 
2 
(1 )(1 )(1 ) 1 (1 )(1 ) 
(1 ) (1 )(1 ) 
1 ( 1) 1 1 . 
2 
xy x y xy x xy x y xy 
x x xy y xy 
x y 
y x 
Bài 28. (Olympic Toán Anh 1986) Cho x, y, z là các số thực thỏa mãn 
đồng thời các điều kiện x y z 0 và x2 y2 z2 6. Tìm giá trị 
lớn nhất và giá trị nhỏ nhất của biểu thức 
P x2y y2z z2x. 
Lời giải. 
Cách 1. Từ giả thiết, dễ dàng suy ra xy yz zx 3. Từ đó suy ra
67 
x2y2 y2z2 z2x2 (xy yz zx)2 2xyz(x y z) 9. 
Bây giờ, ta xét biểu thức Q (x xy 1)2 (y yz 1)2 (z zx 1)2. 
Hiển nhiên 0. Q Khai triển ,Q ta được 
3 2 2 2 2 2 2 2 
3 6 9 2 ( 3) 2 0 2 12 2 . 
Q x x y xy x x y 
P P 
Do 0 Q nên ta có 6. P Lại thấy bộ số 
2 4 8 
2 cos , 2 cos , 2 cos 
9 
, 
9 
( ) 
9 
x y, z 
thỏa mãn đồng thời x y z 0, x2 y2 z2 x2y y2z z2x 6. 
Vì vậy ta có kết luận maxP 6. 
Tiếp theo ta sẽ đi tìm min. P Dễ thấy bộ số ( , , ) ( , , ) a b c x y z thỏa 
mãn điều kiện bài toán, tức là a b c 0 và a2 b2 c2 6. Do đó, 
từ kết quả của max , P ta suy ra a2b b2c c2a 6. Mà 
a2b b2c c2a ( x)2( y) ( y)2( z) ( z)2( x) P. 
Do vậy, ta có P 6. Dấu đẳng thức xảy ra khi 
2 4 8 
2 cos , 2 cos , 2 cos , 
9 9 9 
(a, b, c) 
tức 
2 4 8 
( , , ) 2 cos , 2 cos , 2 cos . 
9 9 9 
x y z Vậy minP 6. 
Cách 2. Tương tự như cách 1, ta chỉ cần chứng minh được bất đẳng thức 
x2y y2z z2x 6 
là bài toán được giải quyết xong. 
Đặt p x y z 0, q xy yz zx 3 và r xyz. Khi đó, dễ 
dàng tính được 
2 2 2 2 2 3 2 2 
3 2 2 
( ) ( ) ( ) 4 2(9 2 
4( 3) 27 27 
) 2 
(4 ) 
x y y z z x p q q q p p 7 
r 
r r 
r 
(x2y y2z z2x) (xy2 yz2 zx2) pq 3r 3r. 
Do đó
68 
2 2 2 2 2 
2 2 
2 2 
2 2 2 
2 3 ( ) 
3 ( ) 3 3 3(4 ) 
3 1 3 ( ) 4 12. 
x y x y xy x y xy r x y 
r x y r r 
r r 
Từ đây ta suy ra x2y y2z z2x 6. 
Cách 3. Cũng giống như hai lời giải trước, ở đây ta cũng sẽ chứng minh 
x2y y2z z2x 6 hay 3(x2y y2z z2x) 18. 
Sử dụng bất đẳng thức Cauchy-Schwarz, ta có 
2 2 2 2 2 
2 2 2 2 2 4 
2 
2 2 2 
3 (2 ) (2 ) 
6 (2 ) 6 (4 4 ) 
6 2 4 18. 
x y x xy z x xy z 
xy z x y xyz z 
x x y xyz x 
Bài toán được chứng minh xong. 
Cách 4. Dưới đây chúng ta sẽ cùng đến với một kỹ thuật khác để chứng 
minh 6.P Thay , z x y ta có x2 y2 ( x y)2 6, suy ra 
x 2 y2 xy 3 hay 
2 2 3 
3. 
2 4 
y y 
x 
Từ đây ta thấy tồn tại số thực a [ , ] sao cho 
3 sin 
2 , 
3 
3 cos 
2 
y 
x a 
y 
a 
hay 
3 sin cos 
. 
2 cos 
x a a 
y a 
Thay vào biểu thức , P ta được
69 
2 2 2 3 2 3 
3 2 
3 
3 2 2 3 
3 2 2 3 
3 3 
( ) ( ) 3 
3 sin cos 6 cos 3 sin cos 8 cos 
3 3 sin 9 sin cos 9 3 sin cos 3 cos 
3 3 sin 9(1 cos )cos 9 3 sin (1 sin ) 3 cos 
3 3(4 sin sin ) 3(3 cos 4 cos ) 3 3 sin 3 3 cos 
P x y y x y x y x x x y y 
a a a a a a 
a a a a a a 
a a a a a a 
a a a a a 3a. 
Theo bất đẳng thức Cauchy-Schwarz, ta có 
2 
3 3 sin 3a 3 cos 3a 3 3 ( 1)2 (sin2 3a cos2 3a) 6. 
Do đó 6. P 
Bài 29. Giả sử ,,x y z là các số thực thỏa x2 y2 z2 2. Chứng minh 
bất đẳng thức sau 
x y z xyz 2. 
Chứng minh. 
Cách 1. Ta viết lại bất đẳng thức dưới dạng 
x(1 yz) (y z) 2. 
Sử dụng bất đẳng thức Cauchy-Schwarz, ta được 
do 
2 2 2 2 2 2 2 
2 2 
2 2 
( ) (1 ) 1 (2 2 )(2 2 ) 
4 2 (1 ) 4 ( 1). 
2 
VT x y z yz yz yz y z 
y z 
y z yz yz 
Từ đây ta có điều phải chứng minh. 
Đẳng thức xảy ra khi và chỉ khi (x, y, z) là một hoán vị của bộ (1, 1, 0). 
Cách 2. Không mất tính tổng quát, giả sử x y z. Ta xét hai trường hợp. 
+ Trường hợp 1. x 0. Viết lại bất đẳng thức dưới dạng 
x(1 yz) 2 y z 0, 
ta thấy nó hiển nhiên đúng vì 
2 2 
0, 1 1 0 
2 
y z 
x yz và 
2 y z 2 2(y2 z2) 2 2 2 2. 
+ Trường hợp 2. 0 x y z. Dễ thấy 
1 
. 
3 
z Đặt t x y, ta có
70 
2 2 2 2 2 2 
. 
2 2 
t x y t z 
xy 
Do đó, bất đẳng thức cần chứng minh tương đương với 
2 2 2 
2 
2 
t z 
z t z hay f (t) zt2 2t z3 4z 4 0. 
Ta thấy ()ft là một tam thức bậc hai của , t hệ số cao nhất của nó 
dương. Lại có 
3 2 2 
2 
1 ( 4 4) ( 2 1)(1 ) 
1 2 
1 (1 ) 0. 
3 3 
f z z z z z z 
z 
Do đó hiển nhiên ta có f (t) 0, t . 
Bài toán được chứng minh xong. 
Cách 3. Bằng lập luận tương tự như cách 2 ở trên, ta cũng đưa được bài 
toán về xét trong trường hợp x, y, z 0. Không mất tính tổng quát, ta có 
thể giả sử 0 . x y z Nếu 1,z thì ta có 
2 xyz x y z (1 z)(1 xy) (1 x)(1 y) 0. 
Xét trường hợp 1. z Khi đó 
z (x y) 2 z2 (x y)2 2 1 xy 1 (1 xy) 2 xyz. 
Bất đẳng thức được chứng minh. 
Bài 30. (Chọn đội tuyển Việt Nam 1996) Chứng minh rằng với mọi số 
thực a, b, c, ta luôn có 
4 4 4 4 4 4 4 
( ) ( ) ( ) ( ). 
7 
b c c a a b a b c 
Chứng minh. 
Cách 1. Đặt a b 2z, c a 2y, b c 2x. Bất đẳng thức cần chứng 
minh được viết lại dưới dạng 
P (y z x)4 (z x y)4 (x y z)4 28(x4 y4 z4). 
Ta có
71 
2 
2 
2 
2 2 2 
2 2 2 2 2 2 
2 4 2 2 2 2 2 2 2 
2 2 2 
3 4 ( ) 4 ( ) 
3 4 16 4 
4 16 4 16 . 
P x yz xy xz 
x x yz xy xz xy xz yz 
x x xy x y xy 
x x y x x x y 
Do đó ta chỉ cần chứng minh được 
2 
x2 4 x2y2 7 x 4. 
Bất đẳng thức cuối này đúng do 
2 
x2 3 x 4 và x2y2 x 4. 
Cách 2. Để ý rằng 
a(a b c) b(a b c) c(a b c) (a b c)2 0. 
Do đó trong các số a(a b c), b(a b c), c(a b c) phải có ít nhất 
một số không âm. Không mất tính tổng quát, giả sử a(a b c) 0. Đặt 
4 4 4 4 4 4 4 
( , , ) ( ) ( ) ( ) ( ). 
7 
P a b c b c c a a b a b c 
Ta sẽ chứng minh ( , , ) , , . 
2 2 
b c b c 
P a b c P a Thật vậy, do 
4 4 3 2 2 2 3 3 
4 4 
3 
( , , ) ( ) 4 ( ) 6 ( ) 4 ( ) 
7 
4 
( ) , 
7 
P a b c b c a b c a b c a b c 
b c a 
nên ta có 
4 
4 4 
2 2 2 2 3 3 3 
2 2 2 2 2 2 
2 2 2 2 
3 ( ) 
( , , ) , , 
2 2 7 8 
3 2( ) ( ) 4( ) ( ) 
3 
(7 10 7 )( ) 3 ( ) 3 ( )( ) 
56 
3 
(7 10 7 )( ) 3 ( )( ) 0. 
56 
b c b c b c 
P a b c P a b c 
a b c b c a b c b c 
b bc c b c a b c a b c b c 
b bc c b c a a b c b c 
Vậy ta chỉ cần chứng minh được P(a, t, t) 0, với . 
2 
b c 
t Bất đẳng 
thức này tương đương với
72 
4 4 4 4 4 
2( ) 16 ( 2 ). 
7 
a t t a t 
Sau khi khai triển và rút gọn, ta được 
Q 5a4 28a3t 42a2t2 28at3 59t4 0. 
Bất đẳng thức cuối này đúng vì Q 2(a 2t)4 3(a 3t)2(a t)2 0. 
Cách 3. Đặt d a b c. Ta sẽ chứng minh bất đẳng thức chặt hơn là 
4 4 4 4 4 4 4 4 
( ) ( ) ( ) ( ). 
7 
b c c a a b a b c d 
Theo bất đẳng thức Cauchy-Schwarz thì 
2 2 
2 2 2 2 2 2 2 
2 2 2 2 2 
( ) ( ) ( ) ( ) 
3 3 
( ) 
. 
3 
b c c a a b a b c a b c 
VT 
a b c d 
Do đó ta chỉ cần chứng minh được 
2 2 2 2 2 4 4 4 4 7 
( ) . 
12 
a b c d a b c d 
Ta sẽ chứng minh bất đẳng thức này đúng với mọi a, b, c, d thỏa mãn 
a b c d 0. Thật vậy, do 
2 2 2 2 
2 
3( ) 3 6 
3( ) 0, 
sym 
a b c ab bc ca a ab 
a b c d 
nên trong các số 
2 2 2 2 2 2 
2 2 2 2 2 2 
3( ), 3( ), 
3( ), 3( ) 
a b c ab bc ca b c d bc cd db 
c d a cd da ac d a b da ab bd 
phải có ít nhất một số không âm. Vì đây là một bất đẳng thức đối xứng với 
bốn biến a, b, c, d nên không mất tính tổng quát, ta có thể giả sử 
a2 b2 c2 3(ab bc ca) 0. 
Đặt A a2 b2 c2 và B ab bc ca thì ta có A B, A 3B 0. 
Sử dụng bất đẳng thức Cauchy-Schwarz, ta có 
4 4 4 2 2 2 2 2 2 2 2 2 2 
2 2 2 2 2 2 2 
( ) 2( ) 
2 2 
( ) ( ) . 
3 3 
a b c a b c a b b c c a 
a b c ab bc ca A B
73 
Do đó ta chỉ cần chứng minh (chú ý rằng d A 2B ) 
2 2 2 2 7 2 
( 2 ) ( 2 ) . 
12 3 
A A B A B A B 
Bất đẳng thức này tương đương với 
1 
( 3 )( ) 0 
3 
A B A B (hiển nhiên đúng). 
Bài toán được chứng minh xong. 
Bài 31. Cho ,,a b c là các số thực không âm thỏa mãn 
a2 b2 c2 abc 4. 
Chứng minh rằng 
ab bc ca abc 2. 
Chứng minh. 
Cách 1. Theo nguyên lý Dirichtle, ta thấy trong ba số ,,a b c có hai số cùng 
1 hoặc cùng 1. Giả sử hai số đó là b và .c Thế thì (1 )(1 ) 0, a b c 
suy ra ab bc ca a bc abc. Mặt khác, theo bất đẳng thức AM-GM, 
ta lại có 
4 a2 b2 c2 abc a2 2bc abc 4 (a bc 2)(a 2). 
Do đó a bc 2, nên kết hợp với trên ta được 
ab bc ca a bc abc 2 abc. 
Bất đẳng thức được chứng minh. 
Đẳng thức xảy ra khi và chỉ khi a b c 1, hoặc a b 2, c 0 và 
các hoán vị tương ứng. 
Cách 2. Không mất tính tổng quát, ta giả sử a b c. Đặt a x y và 
b x y (x y 0). Khi đó, giả thiết bài toán có thể được viết lại thành 
x2(2 c) y2(2 c) 4 c2, và ta phải chứng minh 
(x2 y2)(1 c) 2(1 xc). 
Do 2 2 2 
2 
2 
c 
y c x 
c 
nên bất đẳng thức này tương đương với 
4 2 (4 2) 
(1 ) 2(1 ). 
2 
x c 
c xc 
c
74 
Do a b c nên dễ thấy c 1. Lại có 2 2 2 
0 2 , 
2 
c 
y c x 
c 
nên ta 
suy ra 2.xc Bây giờ, xét hàm số : 0, 2 f c như sau 
4 2 (4 2) 
( ) 2(1 ) (1 ). 
2 
x c 
f x cx c 
c 
Ta có 
8 (1 ) 
( ) 2 0, 
2 
x c 
f x c 
c 
nên f là hàm giảm, suy ra 
f (x) f 2 c . 
Như vậy, ta chỉ cần chứng minh 
f 2 c 0 hay 2 1 c 2 c (2 c)(1 c). 
Bất đẳng thức này tương đương với bất đẳng thức hiển nhiên đúng 
2 
c 1 2 c 0. 
Cách 3. Đặt a 2p, b 2q, c 2r thì được p2 q2 r2 2pqr 1. 
Suy ra tồn tại 0, 
2 
A, B, C thỏa A B C sao cho 
a 2cosA, b 2cosB, c 2cosC. 
Thay vào, bất đẳng thức cần chứng minh trở thành 
1 
cos cos cos cos cos cos 2 cos cos cos . 
2 
P A B B C C A A B C 
Không mất tính tổng quát, ta có thể giả sử , 
2 
A suy ra 1 2cosA 0. 
Từ đây ta được (chú ý rằng ta có cos cos 2 cos 
2 
B C 
B C theo bất 
đẳng thức Jensen cho hàm f (x) cos x lõm trên 0, 
2 
)
75 
2 
2 
2 
2 2 
cos (cos cos ) cos cos (1 2 cos ) 
cos cos 
cos (cos cos ) (1 2 cos ) 
2 
2 cos cos cos (1 2 cos ) 
2 2 
2 cos sin sin (1 2 cos ) 
2 2 
1 
2 cos sin sin (1 2 cos ) 
2 4 2 
3 
cos cos 
2 
P A B C B C A 
B C 
A B C A 
B C B C 
A A 
A A 
A A 
A A 
A A 
A A 
1 cos 1 
(1 2 cos ) . 
2 2 
A 
A 
Bất đẳng thức được chứng minh. 
Cách 4. Thay abc 4 (a2 b2 c2) vào, bất đẳng thức cần chứng minh 
có thể viết lại thành 
a2 b2 c2 ab bc ca 6. 
Ta sử dụng phản chứng. Giả sử bất đẳng thức này sai, khi đó tồn tại một bộ 
số không âm (a, b, c) thỏa mãn đồng thời a2 b2 c2 abc 4 và 
a2 b2 c2 ab bc ca 6. 
Lúc này ta có 
2 2 2 
2 2 2 
2 2 2 
2 2 2 2 2 2 3/2 
6( ) 6 6 
4 
6 6 6 
6( ) 6 6 
. 
( ) 
a b c abc 
a b c abc 
a b c abc 
a b c ab bc ca a b c ab bc ca 
suy ra 
2 2 2 
2 2 2 
3 6 
2( ) ( ) . 
abc 
ab bc ca a b c 
a b c ab bc ca 
Mặt khác theo bất đẳng thức Schur bậc 4 (ở dạng phân thức) thì 
2 2 2 
2 2 2 
6 ( ) 
2( ) ( ) . 
abc a b c 
ab bc ca a b c 
a b c ab bc ca 
Kết hợp với trên, ta suy ra 
2 2 2 2 2 2 
6 ( ) 3 6 
. 
abc a b c abc 
a b c ab bc ca a b c ab bc ca
76 
Do đó ta có 0 abc và 
2(a b c) 3(a2 b2 c2 ab bc ca). 
Điều này không thể xảy ra, bởi vì 
3 a2 ab 2(a b c)2 a2 b2 c2 ab bc ca 0. 
Như vậy, điều giả sử ở trên là sai. Bất đẳng thức được chứng minh. 
Cách 5. Từ giả thiết, ta thấy tồn tại các số thực không âm x, y, z thỏa mãn 
(x y)(y z)(z x) 0 sao cho 
2 2 2 
, , . 
( )( ) ( )( ) ( )( ) 
x y z 
a b c 
x y x z y z y x z x z y 
Thay vào, bất đẳng thức cần chứng minh trở thành 
4 
2 1. 
( ) ( )( ) ( )( )( ) 
xy xyz 
x y x z y z x y y z z x 
Sử dụng bất đẳng thức AM-GM, ta được 
1 1 
2 
( ) ( )( ) 
( )( ) ( )( ) 
( )( ) ( )( ) 
( ) 4 
1 . 
( )( ) ( ) 
xy xy 
x y x z y z x y x z y z 
xy xy 
x y x z y z y x 
xy zx 
x y x z x y x z 
x y z xyz 
x y x z x y 
Bất đẳng thức được chứng minh. 
Bài 32. Cho các số dương a, b, c thỏa mãn 
1 1 1 
1. 
a b 1 b c 1 c a 1 
Chứng minh rằng 
a b c ab bc ca. 
Chứng minh. 
Cách 1. Sử dụng bất đẳng thức Cauchy-Schwarz, ta có
77 
2 2 
2 2 
1 
. 
1 ( 1)( ) ( ) 
a b c a b c 
a b a b a b c a b c 
Cộng bất đẳng thức này với hai bất đẳng thức tương tự, ta được 
2 2 2 
2 
1 1 1 2( ) 
. 
1 1 1 ( ) 
a b c a b c 
a b b c c a a b c 
Do 
1 1 1 
1 
a b 1 b c 1 c a 1 
nên ta có 
a2 b2 c2 2(a b c) (a b c)2, 
từ đó suy ra 
a b c ab bc ca. 
Bất đẳng thức được chứng minh. 
Đẳng thức xảy ra khi và chỉ khi a b c 1. 
Cách 2. Do 
1 
1 
1 1 
a b 
a b a b 
nên giả thiết của bài toán có thể 
được viết lại thành 
2 . 
1 1 1 
a b b c c a 
a b b c c a 
Áp dụng bất đẳng thức Cauchy-Schwarz, ta có 
2 
2 
2 2 2 
( ) 2( ) 
. 
( )( 1) 
a b a b c 
VP 
a b a b a b c ab bc ca a b c 
Từ đó suy ra 
a2 b2 c2 ab bc ca a b c (a b c)2, 
hay a b c ab bc ca. Bài toán được chứng minh xong. 
Cách 3. Không mất tính tổng quát, giả sử a b c. Khi đó ta có 
1 1 1 
, 
a b 1 c a 1 b c 1 
ca cb c bc ba b ab ac a. 
Do đó, theo bất đẳng thức Chebyshev 
1 1 
3 3 ( ) ( ) . 
1 1 
a ab ac a ab ac a 
b c b c 
Vì 
1 1 1 
1, 
b c 1 c a 1 a b 1 
nên ta có
78 
3(a b c) (ab ac a), 
hay a b c ab bc ca. Bất đẳng thức được chứng minh. 
Cách 4. Yêu cầu của bài toán tương đương với việc chứng minh: Nếu các số 
dương ,,ab c thỏa mãn , ab bc ca a b c thì 
1 1 1 
1. 
a b 1 b c 1 c a 1 
Do 
1 
1 
1 1 
a b 
a b a b 
nên bất đẳng thức này tương đương với 
2. 
1 1 1 
a b b c c a 
a b b c c a 
Vì 1 
ab bc ca 
a b c 
nên ta có 
( )( ) 
. 
1 ( )( ) 
a b a b a b a b c 
a b ab bc ca a b a b c ab bc ca 
a b 
a b c 
Như vậy, ta chỉ cần chứng minh 
( )( ) 
2. 
( )( ) 
a b a b c 
a b a b c ab bc ca 
Bất đẳng thức này tương đương với 
( )( ) 1 1 
, 
( )( ) 2 2 
a b a b c 
a b a b c ab bc ca 
hay là 
2 2 
1. 
( )( ) 
a ab b 
a b a b c ab bc ca 
Sử dụng đánh giá cơ bản 2 2 2 3 
( ) ( 
4 
x xy y x y x, y ) kết hợp 
với bất đẳng thức Cauchy-Schwarz, ta được 
2 
2 
3 ( ) 
4 ( )( ) 
3 ( ) 
. 
4 ( )( ) 
a b 
VT 
a b a b c ab bc ca 
a b 
a b a b c ab bc ca 
Từ đó suy ra, ta chỉ cần chứng minh
79 
3(a b c)2 (a b)(a b c) ab bc ca . 
Sau khi khai triển và rút gọn, ta thu được bất đẳng thức hiển nhiên đúng 
a2 b2 c2 ab bc ca. 
Bài toán được chứng minh xong. 
Bài 33. Cho ,,x y z là các số thực dương. Chứng minh rằng 
(x2 y2)(y2 z2)(z2 x2)(xy yz zx)2 8x2y2z2(x2 y2 z2)2. 
Chứng minh. 
Cách 1. Để ý rằng 
2 2 2 2 2 2 2 2 
2 2 2 2 
( )( ) ( ) ( ) , 
2( ) ( ) ( ) . 
x y x z x yz x y z 
y z y z y z 
Do đó, sử dụng bất đẳng thức Cauchy-Schwarz, ta được 
2 
2 2 2 2 2 2 2 
2 
2 
2( )( )( ) ( )( ) ( )( ) 
( ) 2 . 
x y y z z x x yz y z x y z y z 
x y z xyz 
Bài toán được quy về chứng minh 
(xy yz zx) x 2(y z) 2xyz 4xyz(x 2 y2 z 2). 
Bất đẳng thức này tương đương với 
2( ) 2( ) 2( ) 2 4( 2 2 2) 
, 
x y z y z x z x y xyz x y z 
xyz xy yz zx 
hay 
2 2 2 2 2 2 4( 2 2 2) 
2 . 
x y y z z x x y z 
xy yz zx xy yz zx 
Do 
2 2 ( )2 
2 
x y x y 
xy xy 
nên ta có 
( )2 ( )2 ( )2 4( 2 2 2) 4( )2 
8 . 
x y y z z x x y z x y z 
xy yz zx xy yz zx xy yz zx 
Bất đẳng thức cuối này đúng theo bất đẳng thức Cauchy-Schwarz 
2 
( ) ( ) ( ) 4( )2 
. 
x y y z z x x y z 
VT 
xy yz zx xy yz zx 
Bài toán được chứng minh xong.
80 
Đẳng thức xảy ra khi và chỉ khi x y z. 
Cách 2. Bất đẳng thức cần chứng minh tương đương với 
2 
2 2 2 2 2 2 2 2 2 
, 
2 2 2 
x y y z z x x y z 
xy yz zx xy yz zx 
2 
( )2 ( )2 ( )2 ( )2 
1 1 1 1 . 
2 2 2 2( ) 
x y y z z x x y 
xy yz zx xy yz zx 
Không mất tính tổng quát, giả sử x y z. Khi đó ta có 
( )2 ( )2 ( )2 ( )2 ( )2 
1 1 1 1 . 
2 2 2 2 2( ) 
x y y z x y y z x z 
xy yz xy yz xy yz 
Lại có (x y)2 (y z)2 (x z)2 2(x y)(y z) (x z)2. Do đó, 
ta chỉ cần chứng minh được 
2 
( )2 ( )2 ( )2 
1 1 1 . 
2( ) 2 
x z x z x z 
xy yz zx xy yz zx 
Bất đẳng thức cuối này đúng, vì theo các bất đẳng thức Cauchy-Schwarz và 
AM-GM, ta có 
2 2 
( )2 ( )2 
1 1 . 
2 ( ) ( ) 
x z x z 
VT 
zx xy yz zx xy yz 
Cách 3. Bất đẳng thức cần chứng minh tương đương với 
2 
2 2 2 2 2 2 2 2 2 2 1 1 1 
(x y )(y z )(z x ) 8(x y z ) . 
x y z 
Đặt x2 y2 2a, y2 z2 2b, z2 x2 2c, thì ta có a, b, c là ba cạnh 
của một tam giác, và bất đẳng thức trên trở thành 
2 
2 1 1 1 
abc (a b c) . 
b c a c a b a b c 
Sử dụng bất đẳng thức Holder, ta có 
2 
3 3 1 
a (b c a) (a b c) . 
b c a 
Do đó ta chỉ cần chứng minh được
81 
abc a3(b c a) b3(c a b) c3(a b c). 
Đây chính là bất đẳng thức Schur bậc 4. 
Cách 4. Đặt 
1 1 1 
x , y , z . 
a b c 
Bất đẳng thức trở thành 
(a2 b2)(b2 c2)(c2 a2)(a b c)2 8(a2b2 b2c2 c2a2)2. 
Áp dụng bất đẳng thức AM-GM, ta có 
2 2 2 2 
2 2 2 2 2 2 
2 2 2 
2 2 2 2 2 2 
2 2 2 
2 2 2 2 2 2 
( ) 2 2 2 
4 4 4 
2 2 2 
4 4 4 
. 
a b c a b c bc ca ab 
b c c a a b 
a b c 
bc ca ab 
b c c a a b 
a b c 
b c c a a b 
Đặt m a2, n b2, p c2. Từ đánh giá trên suy ra ta chỉ cần chứng minh 
2 4 
( )( )( ) 8( ) . 
np 
m n n p p m m n p mn np pm 
n p 
Sau khi khai triển và rút gọn, ta thu được bất đẳng thức hiển nhiên đúng 
mn(m n)2 np(n p)2 pm(p m)2 0. 
Bài 34. Chứng minh rằng với mọi ,,a b c dương, ta đều có 
2 2 2 
1 1 1 
. 
a b c 
b c c a a b a bc b ca c ab 
Chứng minh. 
Cách 1. Sử dụng bất đẳng thức Cauchy-Schwarz, ta có 
2 
2 2 
2 
2 2 
( )( ) 
( ) ( ) 1 , 
( )( ) 
( ) ( ) 1 . 
b a bc b c 
a b a bc 
c c 
c a bc b c 
a c a bc 
b b 
Từ đó suy ra 
2 2 2 2 2 
1 1 1 
. 
( ) ( ) ( )( ) ( )( ) 
b c 
a b a c b c a bc b c a bc a bc 
Sử dụng đánh giá này, ta được
82 
2 2 2 2 
2 2 2 2 
2 
( ) ( ) ( ) ( ) 
1 1 
( ) ( ) ( ) ( ) 
1 
. 
b c b c 
b c b c b c b c 
a a 
a 
a b c a a b a c 
b c 
a 
a bc 
Đẳng thức xảy ra khi và chỉ khi . a b c 
Cách 2. Nhân 2 vế của bất đẳng thức với ab(a b) bc(b c) ca(c a) 
và chú ý rằng 
2 2 2 2 2 
2 
2 2 2 2 2 2 
2 2 2 
( ) ( )( ) ( ) ( ) 
, 
( ) [( )( ) ( )] ( ) 
( ) . 
ab a b b c a bc a b c a b c 
a bc 
b c b c b c 
a ab a b a b c a bc a b c a b c 
a b c 
a bc a bc a bc 
Ta viết được nó dưới dạng 
2 2 2 2 2 
2 
2 
( ) ( ) 
. 
a b c a b c 
a ab 
b c a bc 
Theo bất đẳng thức Cauchy-Schwarz thì 
( 2 2) ( ) 
. 
2 
a b c a b c 
ab 
b c 
Do vậy ta chỉ cần chứng minh 
2 2 2 
2 
2 
( ) 
. 
a b c 
a 
a bc 
Ta có 
2 2 2 2 
2 
2 2 
2 2 2 2 
2 
2 2 
2 2 2 2 
2 
2 2 2 2 
2 4 2 2 5 
2 2 
( ) 
1 
( )( ) 
( ) 
. 
( ) ( ) 
a b c a 
VT VP a 
a bc a bc 
c a a b 
a 
c ab b ca 
b c a bc a bc 
a 
b ca c ab b ca c ab 
a bc a b c abc a abc ab 
a bc a bc 
Theo bất đẳng thức AM-GM, ta dễ thấy 
5 5 5 
2 2 5 2 2 
. 
5 
a b c 
abc ab a b c a
83 
Bài toán được chứng minh xong. 
Cách 3. Không mất tính tổng quát, giả sử a min{a, b, c}. Ta có 
2 2 2 
1 1 ( )( ) 
. 
( )( ) 
a a a b a c 
b c a bc b c a bc b c a bc 
Do (a b)(a c) 0 nên ta chỉ cần chứng minh được 
2 2 
( )( ) ( )( ) 
0. 
( )( ) ( )( ) 
b c b a c a c b 
c a b ca a b c ab 
Bất đẳng thức này tương đương với 
(b c) (b2 a2)(c2 ab) (a2 c2)(b2 ca) 0, 
a(b c)2(b2 c2 a2 ab bc ca) 0 (đúng do a min{a, b, c} ). 
Bài 35. Cho các số thực , , . a b c Chứng minh rằng 
(a2 b2 c2)2 3(a3b b3c c3a). 
Chứng minh. 
Cách 1. Đặt A a3b, B ab3, C a2b2, D a2bc, khi đó 
ta có đẳng thức sau 
2 2 2 2 3 3 3 
2 2 
4( ) ( ) 3( ) 
( 3 ) 3( ) . 
C D a b c a b b c c a 
A B D C A B C D 
Mà CD nên từ trên, ta dễ dàng suy ra được điều phải chứng minh. 
Đẳng thức xảy ra a b c hoặc 2 2 2 4 2 
: : sin : sin : sin 
7 7 7 
a b c 
và các hoán vị tương ứng. 
Cách 2. Đặt b a x, c a y, ta có thể viết lại bất đẳng thức cần 
chứng minh dưới dạng 
2 2 2 3 2 2 3 
4 3 2 2 4 
( , , ) ( ) ( 5 4 ) 
3 2 0. 
f a x y x xy y a x x y xy y a 
x x y x y y 
Dễ thấy đây là một tam thức bậc hai của a với hệ số cao nhất không dương. 
Ta có 
3 2 2 3 2 2 2 4 3 2 2 4 
3 2 2 3 2 
( 5 4 ) 4( )( 3 2 ) 
3( 2 ) 0, 
f x x y xy y x xy y x x y x y y 
x x y xy y
84 
nên hiển nhiên ( , , ) 0 f a x y và bài toán được chứng minh. 
Cách 3. Ta có bất đẳng thức cần chứng minh tương đương với 
a4 a2b2 3 a2b2 a2bc 3 a3b a2bc 0. 
Thực hiện phép khai triển, phân tích bình phương, ta lần lượt có 
4 2 2 2 2 2 
2 2 2 2 
1 
( ) , 
2 
1 
(2 ) 
2 
a a b a b 
a b a bc bc ca ab 
và 
3 2 3 2 2 2 
2 2 2 2 
2 2 
( ) 
( ) ( ) 
3 
1 
( )(2 ). 
3 
a b a bc b c a bc bc b a 
ab bc ca 
bc b a b a 
a b bc ca ab 
Do đó bất đẳng thức trên có thể viết lại thành 
2 2 2 2 2 2 1 1 
( ) (2 ) ( )(2 ) 0, 
2 2 
a b bc ca ab a b bc ca ab 
tương đương 
2 2 2 1 
( 2 ) 0 
2 
a b bc ab ca (hiển nhiên đúng). 
Bài toán được chứng minh xong. 
Cách 4. Có thể kiểm tra được hằng đẳng thức sau 
2 2 2 2 2 2 2 3 3 3 
2 
3 3 3 2 2 2 2 2 2 
2 
3 3 3 2 2 2 2 2 2 
4( ) ( ) 3( ) 
( ) 5( ) 4( ) 
3 ( ) ( ) 2( ) 6 0. 
a b c ab bc ca a b c a b b c c a 
a b c a b b c c a ab bc ca 
a b c a b b c c a ab bc ca abc 
Từ hằng đẳng thức này, ta có thể suy ra ngay kết quả cần chứng minh. 
Bài 36. Cho a, b, c là các số thực không âm thỏa mãn ab bc ca 0. 
Chứng minh rằng 
2 2 2 
( ) ( ) ( ) 
2. 
a b c b c a c a b 
a bc b ca c ab 
Chứng minh.
Cac bai toan co nhieu loi giai
Cac bai toan co nhieu loi giai
Cac bai toan co nhieu loi giai
Cac bai toan co nhieu loi giai
Cac bai toan co nhieu loi giai
Cac bai toan co nhieu loi giai
Cac bai toan co nhieu loi giai
Cac bai toan co nhieu loi giai
Cac bai toan co nhieu loi giai
Cac bai toan co nhieu loi giai
Cac bai toan co nhieu loi giai
Cac bai toan co nhieu loi giai
Cac bai toan co nhieu loi giai
Cac bai toan co nhieu loi giai
Cac bai toan co nhieu loi giai
Cac bai toan co nhieu loi giai
Cac bai toan co nhieu loi giai
Cac bai toan co nhieu loi giai
Cac bai toan co nhieu loi giai
Cac bai toan co nhieu loi giai
Cac bai toan co nhieu loi giai
Cac bai toan co nhieu loi giai
Cac bai toan co nhieu loi giai
Cac bai toan co nhieu loi giai
Cac bai toan co nhieu loi giai
Cac bai toan co nhieu loi giai
Cac bai toan co nhieu loi giai
Cac bai toan co nhieu loi giai
Cac bai toan co nhieu loi giai
Cac bai toan co nhieu loi giai
Cac bai toan co nhieu loi giai
Cac bai toan co nhieu loi giai
Cac bai toan co nhieu loi giai

More Related Content

What's hot

Tuyen tap cac bai toan va phuong phap giai pt va bpt vo ty
Tuyen tap cac bai toan va phuong phap giai  pt va bpt vo ty Tuyen tap cac bai toan va phuong phap giai  pt va bpt vo ty
Tuyen tap cac bai toan va phuong phap giai pt va bpt vo ty Huynh ICT
 
Tuyển tập 100 hệ phương trình thường gặp (2015-2016) - Megabook.vn
Tuyển tập 100 hệ phương trình thường gặp (2015-2016) - Megabook.vnTuyển tập 100 hệ phương trình thường gặp (2015-2016) - Megabook.vn
Tuyển tập 100 hệ phương trình thường gặp (2015-2016) - Megabook.vnMegabook
 
Bai giang phuong tring luong giac
Bai giang phuong tring luong giacBai giang phuong tring luong giac
Bai giang phuong tring luong giactotoanms
 
Tích phân từng phần
Tích phân từng phầnTích phân từng phần
Tích phân từng phầnroggerbob
 
Pt co ban bttl phan 6 ct0, ct1, ct2
Pt co ban bttl phan 6 ct0, ct1, ct2Pt co ban bttl phan 6 ct0, ct1, ct2
Pt co ban bttl phan 6 ct0, ct1, ct2Hồng Quang
 
01 cac phep bien doi lg p1
01 cac phep bien doi lg p101 cac phep bien doi lg p1
01 cac phep bien doi lg p1Huynh ICT
 
[ Www.nguoithay.com ] hon 250 bai phuong trinh va he phuong trinh 2012
[ Www.nguoithay.com ] hon 250 bai phuong trinh va he phuong trinh 2012[ Www.nguoithay.com ] hon 250 bai phuong trinh va he phuong trinh 2012
[ Www.nguoithay.com ] hon 250 bai phuong trinh va he phuong trinh 2012Thuy Trang
 
10 ptvt bien doi lopluyenthi.vn
10 ptvt bien doi lopluyenthi.vn10 ptvt bien doi lopluyenthi.vn
10 ptvt bien doi lopluyenthi.vnHồng Quang
 
200 Bài Tập Tích Phân Hay Và Khó
200 Bài Tập Tích Phân Hay Và Khó200 Bài Tập Tích Phân Hay Và Khó
200 Bài Tập Tích Phân Hay Và KhóAnh Thư
 
10 ptvt lien hop lopluyenthi.vn
10 ptvt lien hop lopluyenthi.vn10 ptvt lien hop lopluyenthi.vn
10 ptvt lien hop lopluyenthi.vnHồng Quang
 
Tich phan %28 nguyen duy khoi%29
Tich phan %28 nguyen duy khoi%29Tich phan %28 nguyen duy khoi%29
Tich phan %28 nguyen duy khoi%29trongphuckhtn
 
Bt toan a2
Bt toan   a2Bt toan   a2
Bt toan a2Duy Duy
 
[Vietmaths.net] bi quyet giai ptlg tm han
[Vietmaths.net] bi quyet giai ptlg tm han[Vietmaths.net] bi quyet giai ptlg tm han
[Vietmaths.net] bi quyet giai ptlg tm hanNam Le
 
Bai tap-dinh-thuc
Bai tap-dinh-thucBai tap-dinh-thuc
Bai tap-dinh-thucthuyvan1991
 
9 pt vo ti (co ban) htq
9 pt vo ti (co ban) htq9 pt vo ti (co ban) htq
9 pt vo ti (co ban) htqHồng Quang
 
Ba dạng hệ phương trình cơ bản
Ba dạng hệ phương trình cơ bảnBa dạng hệ phương trình cơ bản
Ba dạng hệ phương trình cơ bảnHồng Quang
 
Bai tap a2 c2
Bai tap a2   c2Bai tap a2   c2
Bai tap a2 c2Duy Duy
 

What's hot (17)

Tuyen tap cac bai toan va phuong phap giai pt va bpt vo ty
Tuyen tap cac bai toan va phuong phap giai  pt va bpt vo ty Tuyen tap cac bai toan va phuong phap giai  pt va bpt vo ty
Tuyen tap cac bai toan va phuong phap giai pt va bpt vo ty
 
Tuyển tập 100 hệ phương trình thường gặp (2015-2016) - Megabook.vn
Tuyển tập 100 hệ phương trình thường gặp (2015-2016) - Megabook.vnTuyển tập 100 hệ phương trình thường gặp (2015-2016) - Megabook.vn
Tuyển tập 100 hệ phương trình thường gặp (2015-2016) - Megabook.vn
 
Bai giang phuong tring luong giac
Bai giang phuong tring luong giacBai giang phuong tring luong giac
Bai giang phuong tring luong giac
 
Tích phân từng phần
Tích phân từng phầnTích phân từng phần
Tích phân từng phần
 
Pt co ban bttl phan 6 ct0, ct1, ct2
Pt co ban bttl phan 6 ct0, ct1, ct2Pt co ban bttl phan 6 ct0, ct1, ct2
Pt co ban bttl phan 6 ct0, ct1, ct2
 
01 cac phep bien doi lg p1
01 cac phep bien doi lg p101 cac phep bien doi lg p1
01 cac phep bien doi lg p1
 
[ Www.nguoithay.com ] hon 250 bai phuong trinh va he phuong trinh 2012
[ Www.nguoithay.com ] hon 250 bai phuong trinh va he phuong trinh 2012[ Www.nguoithay.com ] hon 250 bai phuong trinh va he phuong trinh 2012
[ Www.nguoithay.com ] hon 250 bai phuong trinh va he phuong trinh 2012
 
10 ptvt bien doi lopluyenthi.vn
10 ptvt bien doi lopluyenthi.vn10 ptvt bien doi lopluyenthi.vn
10 ptvt bien doi lopluyenthi.vn
 
200 Bài Tập Tích Phân Hay Và Khó
200 Bài Tập Tích Phân Hay Và Khó200 Bài Tập Tích Phân Hay Và Khó
200 Bài Tập Tích Phân Hay Và Khó
 
10 ptvt lien hop lopluyenthi.vn
10 ptvt lien hop lopluyenthi.vn10 ptvt lien hop lopluyenthi.vn
10 ptvt lien hop lopluyenthi.vn
 
Tich phan %28 nguyen duy khoi%29
Tich phan %28 nguyen duy khoi%29Tich phan %28 nguyen duy khoi%29
Tich phan %28 nguyen duy khoi%29
 
Bt toan a2
Bt toan   a2Bt toan   a2
Bt toan a2
 
[Vietmaths.net] bi quyet giai ptlg tm han
[Vietmaths.net] bi quyet giai ptlg tm han[Vietmaths.net] bi quyet giai ptlg tm han
[Vietmaths.net] bi quyet giai ptlg tm han
 
Bai tap-dinh-thuc
Bai tap-dinh-thucBai tap-dinh-thuc
Bai tap-dinh-thuc
 
9 pt vo ti (co ban) htq
9 pt vo ti (co ban) htq9 pt vo ti (co ban) htq
9 pt vo ti (co ban) htq
 
Ba dạng hệ phương trình cơ bản
Ba dạng hệ phương trình cơ bảnBa dạng hệ phương trình cơ bản
Ba dạng hệ phương trình cơ bản
 
Bai tap a2 c2
Bai tap a2   c2Bai tap a2   c2
Bai tap a2 c2
 

Similar to Cac bai toan co nhieu loi giai

Bđt weitzenbock, bđt hadwinger finsler và những mở rộng
Bđt weitzenbock, bđt hadwinger   finsler và những mở rộngBđt weitzenbock, bđt hadwinger   finsler và những mở rộng
Bđt weitzenbock, bđt hadwinger finsler và những mở rộngĐình Huy
 
Một số bài toán bồi dưỡng HSG môn Toán lớp 6 - 7 - 8 - Phần Đại Số
Một số bài toán bồi dưỡng HSG môn Toán lớp 6 - 7 - 8 - Phần Đại SốMột số bài toán bồi dưỡng HSG môn Toán lớp 6 - 7 - 8 - Phần Đại Số
Một số bài toán bồi dưỡng HSG môn Toán lớp 6 - 7 - 8 - Phần Đại SốBồi dưỡng Toán lớp 6
 
bất đẳng thức và các ứng dụng-đã nén.pdf
bất đẳng thức và các ứng dụng-đã nén.pdfbất đẳng thức và các ứng dụng-đã nén.pdf
bất đẳng thức và các ứng dụng-đã nén.pdfNguynHuy323399
 
15 CHUYÊN ĐÊ NÂNG CAO TOÁN LỚP 8 NĂM 2023 (700 TRANG) HỆ THỐNG BÀI TẬP HSG TỈ...
15 CHUYÊN ĐÊ NÂNG CAO TOÁN LỚP 8 NĂM 2023 (700 TRANG) HỆ THỐNG BÀI TẬP HSG TỈ...15 CHUYÊN ĐÊ NÂNG CAO TOÁN LỚP 8 NĂM 2023 (700 TRANG) HỆ THỐNG BÀI TẬP HSG TỈ...
15 CHUYÊN ĐÊ NÂNG CAO TOÁN LỚP 8 NĂM 2023 (700 TRANG) HỆ THỐNG BÀI TẬP HSG TỈ...Nguyen Thanh Tu Collection
 
SƠ LƯỢC VỀ PHƯƠNG PHÁP ĐÁNH GIÁ
SƠ LƯỢC VỀ PHƯƠNG PHÁP ĐÁNH GIÁSƠ LƯỢC VỀ PHƯƠNG PHÁP ĐÁNH GIÁ
SƠ LƯỢC VỀ PHƯƠNG PHÁP ĐÁNH GIÁDANAMATH
 
12 Cách giải cho 1 bài bất đẳng thức
12 Cách giải cho 1 bài bất đẳng thức12 Cách giải cho 1 bài bất đẳng thức
12 Cách giải cho 1 bài bất đẳng thứcSirô Tiny
 
Tìm trị riêng bằng pp qr
Tìm trị riêng bằng pp qrTìm trị riêng bằng pp qr
Tìm trị riêng bằng pp qrToàn Phan
 
Bat dang thuc amgm
Bat dang thuc amgmBat dang thuc amgm
Bat dang thuc amgmHùng Sỹ
 
Dap an-de-thi-dai-hoc-mon-toan-khoi-a-a1-2014
Dap an-de-thi-dai-hoc-mon-toan-khoi-a-a1-2014Dap an-de-thi-dai-hoc-mon-toan-khoi-a-a1-2014
Dap an-de-thi-dai-hoc-mon-toan-khoi-a-a1-2014webdethi
 
Chinh phục 100 hệ phương trình hay thường gặp 2015-2016 - Megabook.vn
Chinh phục 100 hệ phương trình hay thường gặp 2015-2016 - Megabook.vnChinh phục 100 hệ phương trình hay thường gặp 2015-2016 - Megabook.vn
Chinh phục 100 hệ phương trình hay thường gặp 2015-2016 - Megabook.vnMegabook
 
LIÊN HỢP NGƯỢC DẤU & TỶ SỐ TRONG OXY
LIÊN HỢP NGƯỢC DẤU & TỶ SỐ TRONG OXYLIÊN HỢP NGƯỢC DẤU & TỶ SỐ TRONG OXY
LIÊN HỢP NGƯỢC DẤU & TỶ SỐ TRONG OXYDANAMATH
 
De tsl10 toan hai duong(chuyen ng trai) 13 14(giai)
De tsl10 toan hai duong(chuyen ng trai) 13 14(giai)De tsl10 toan hai duong(chuyen ng trai) 13 14(giai)
De tsl10 toan hai duong(chuyen ng trai) 13 14(giai)Toan Isi
 
Toan pt.de004.2011
Toan pt.de004.2011Toan pt.de004.2011
Toan pt.de004.2011BẢO Hí
 
Dap an-de-thi-toan-vao-10-2013-ha-noi
Dap an-de-thi-toan-vao-10-2013-ha-noiDap an-de-thi-toan-vao-10-2013-ha-noi
Dap an-de-thi-toan-vao-10-2013-ha-noiwebdethi
 
Bất đẳng thức bồi dưỡng học sinh giỏi thcs
Bất đẳng thức bồi dưỡng học sinh giỏi thcsBất đẳng thức bồi dưỡng học sinh giỏi thcs
Bất đẳng thức bồi dưỡng học sinh giỏi thcsmaytinh_5p
 
Toan pt.de012.2011
Toan pt.de012.2011Toan pt.de012.2011
Toan pt.de012.2011BẢO Hí
 

Similar to Cac bai toan co nhieu loi giai (20)

Bđt weitzenbock, bđt hadwinger finsler và những mở rộng
Bđt weitzenbock, bđt hadwinger   finsler và những mở rộngBđt weitzenbock, bđt hadwinger   finsler và những mở rộng
Bđt weitzenbock, bđt hadwinger finsler và những mở rộng
 
Mot bo de hay CM BDT
Mot bo de hay CM BDTMot bo de hay CM BDT
Mot bo de hay CM BDT
 
Một số bài toán bồi dưỡng HSG môn Toán lớp 6 - 7 - 8 - Phần Đại Số
Một số bài toán bồi dưỡng HSG môn Toán lớp 6 - 7 - 8 - Phần Đại SốMột số bài toán bồi dưỡng HSG môn Toán lớp 6 - 7 - 8 - Phần Đại Số
Một số bài toán bồi dưỡng HSG môn Toán lớp 6 - 7 - 8 - Phần Đại Số
 
100 de toan 6
100 de toan 6100 de toan 6
100 de toan 6
 
Lời giả3 (1)
Lời giả3 (1)Lời giả3 (1)
Lời giả3 (1)
 
bất đẳng thức và các ứng dụng-đã nén.pdf
bất đẳng thức và các ứng dụng-đã nén.pdfbất đẳng thức và các ứng dụng-đã nén.pdf
bất đẳng thức và các ứng dụng-đã nén.pdf
 
15 CHUYÊN ĐÊ NÂNG CAO TOÁN LỚP 8 NĂM 2023 (700 TRANG) HỆ THỐNG BÀI TẬP HSG TỈ...
15 CHUYÊN ĐÊ NÂNG CAO TOÁN LỚP 8 NĂM 2023 (700 TRANG) HỆ THỐNG BÀI TẬP HSG TỈ...15 CHUYÊN ĐÊ NÂNG CAO TOÁN LỚP 8 NĂM 2023 (700 TRANG) HỆ THỐNG BÀI TẬP HSG TỈ...
15 CHUYÊN ĐÊ NÂNG CAO TOÁN LỚP 8 NĂM 2023 (700 TRANG) HỆ THỐNG BÀI TẬP HSG TỈ...
 
SƠ LƯỢC VỀ PHƯƠNG PHÁP ĐÁNH GIÁ
SƠ LƯỢC VỀ PHƯƠNG PHÁP ĐÁNH GIÁSƠ LƯỢC VỀ PHƯƠNG PHÁP ĐÁNH GIÁ
SƠ LƯỢC VỀ PHƯƠNG PHÁP ĐÁNH GIÁ
 
12 Cách giải cho 1 bài bất đẳng thức
12 Cách giải cho 1 bài bất đẳng thức12 Cách giải cho 1 bài bất đẳng thức
12 Cách giải cho 1 bài bất đẳng thức
 
Tìm trị riêng bằng pp qr
Tìm trị riêng bằng pp qrTìm trị riêng bằng pp qr
Tìm trị riêng bằng pp qr
 
Bat dang thuc amgm
Bat dang thuc amgmBat dang thuc amgm
Bat dang thuc amgm
 
Dap an-de-thi-dai-hoc-mon-toan-khoi-a-a1-2014
Dap an-de-thi-dai-hoc-mon-toan-khoi-a-a1-2014Dap an-de-thi-dai-hoc-mon-toan-khoi-a-a1-2014
Dap an-de-thi-dai-hoc-mon-toan-khoi-a-a1-2014
 
Chinh phục 100 hệ phương trình hay thường gặp 2015-2016 - Megabook.vn
Chinh phục 100 hệ phương trình hay thường gặp 2015-2016 - Megabook.vnChinh phục 100 hệ phương trình hay thường gặp 2015-2016 - Megabook.vn
Chinh phục 100 hệ phương trình hay thường gặp 2015-2016 - Megabook.vn
 
LIÊN HỢP NGƯỢC DẤU & TỶ SỐ TRONG OXY
LIÊN HỢP NGƯỢC DẤU & TỶ SỐ TRONG OXYLIÊN HỢP NGƯỢC DẤU & TỶ SỐ TRONG OXY
LIÊN HỢP NGƯỢC DẤU & TỶ SỐ TRONG OXY
 
De tsl10 toan hai duong(chuyen ng trai) 13 14(giai)
De tsl10 toan hai duong(chuyen ng trai) 13 14(giai)De tsl10 toan hai duong(chuyen ng trai) 13 14(giai)
De tsl10 toan hai duong(chuyen ng trai) 13 14(giai)
 
Toan pt.de004.2011
Toan pt.de004.2011Toan pt.de004.2011
Toan pt.de004.2011
 
Dap an-de-thi-toan-vao-10-2013-ha-noi
Dap an-de-thi-toan-vao-10-2013-ha-noiDap an-de-thi-toan-vao-10-2013-ha-noi
Dap an-de-thi-toan-vao-10-2013-ha-noi
 
Bất đẳng thức bồi dưỡng học sinh giỏi thcs
Bất đẳng thức bồi dưỡng học sinh giỏi thcsBất đẳng thức bồi dưỡng học sinh giỏi thcs
Bất đẳng thức bồi dưỡng học sinh giỏi thcs
 
BĐT
BĐTBĐT
BĐT
 
Toan pt.de012.2011
Toan pt.de012.2011Toan pt.de012.2011
Toan pt.de012.2011
 

Recently uploaded

GIÁO TRÌNH KHỐI NGUỒN CÁC LOẠI - ĐIỆN LẠNH BÁCH KHOA HÀ NỘI
GIÁO TRÌNH  KHỐI NGUỒN CÁC LOẠI - ĐIỆN LẠNH BÁCH KHOA HÀ NỘIGIÁO TRÌNH  KHỐI NGUỒN CÁC LOẠI - ĐIỆN LẠNH BÁCH KHOA HÀ NỘI
GIÁO TRÌNH KHỐI NGUỒN CÁC LOẠI - ĐIỆN LẠNH BÁCH KHOA HÀ NỘIĐiện Lạnh Bách Khoa Hà Nội
 
TÀI LIỆU BỒI DƯỠNG HỌC SINH GIỎI LÝ LUẬN VĂN HỌC NĂM HỌC 2023-2024 - MÔN NGỮ ...
TÀI LIỆU BỒI DƯỠNG HỌC SINH GIỎI LÝ LUẬN VĂN HỌC NĂM HỌC 2023-2024 - MÔN NGỮ ...TÀI LIỆU BỒI DƯỠNG HỌC SINH GIỎI LÝ LUẬN VĂN HỌC NĂM HỌC 2023-2024 - MÔN NGỮ ...
TÀI LIỆU BỒI DƯỠNG HỌC SINH GIỎI LÝ LUẬN VĂN HỌC NĂM HỌC 2023-2024 - MÔN NGỮ ...Nguyen Thanh Tu Collection
 
Các điều kiện bảo hiểm trong bảo hiểm hàng hoá
Các điều kiện bảo hiểm trong bảo hiểm hàng hoáCác điều kiện bảo hiểm trong bảo hiểm hàng hoá
Các điều kiện bảo hiểm trong bảo hiểm hàng hoámyvh40253
 
GNHH và KBHQ - giao nhận hàng hoá và khai báo hải quan
GNHH và KBHQ - giao nhận hàng hoá và khai báo hải quanGNHH và KBHQ - giao nhận hàng hoá và khai báo hải quan
GNHH và KBHQ - giao nhận hàng hoá và khai báo hải quanmyvh40253
 
30 ĐỀ PHÁT TRIỂN THEO CẤU TRÚC ĐỀ MINH HỌA BGD NGÀY 22-3-2024 KỲ THI TỐT NGHI...
30 ĐỀ PHÁT TRIỂN THEO CẤU TRÚC ĐỀ MINH HỌA BGD NGÀY 22-3-2024 KỲ THI TỐT NGHI...30 ĐỀ PHÁT TRIỂN THEO CẤU TRÚC ĐỀ MINH HỌA BGD NGÀY 22-3-2024 KỲ THI TỐT NGHI...
30 ĐỀ PHÁT TRIỂN THEO CẤU TRÚC ĐỀ MINH HỌA BGD NGÀY 22-3-2024 KỲ THI TỐT NGHI...Nguyen Thanh Tu Collection
 
ĐỀ CHÍNH THỨC KỲ THI TUYỂN SINH VÀO LỚP 10 THPT CÁC TỈNH THÀNH NĂM HỌC 2020 –...
ĐỀ CHÍNH THỨC KỲ THI TUYỂN SINH VÀO LỚP 10 THPT CÁC TỈNH THÀNH NĂM HỌC 2020 –...ĐỀ CHÍNH THỨC KỲ THI TUYỂN SINH VÀO LỚP 10 THPT CÁC TỈNH THÀNH NĂM HỌC 2020 –...
ĐỀ CHÍNH THỨC KỲ THI TUYỂN SINH VÀO LỚP 10 THPT CÁC TỈNH THÀNH NĂM HỌC 2020 –...Nguyen Thanh Tu Collection
 
Giới thiệu Dự án Sản Phụ Khoa - Y Học Cộng Đồng
Giới thiệu Dự án Sản Phụ Khoa - Y Học Cộng ĐồngGiới thiệu Dự án Sản Phụ Khoa - Y Học Cộng Đồng
Giới thiệu Dự án Sản Phụ Khoa - Y Học Cộng ĐồngYhoccongdong.com
 
30 ĐỀ PHÁT TRIỂN THEO CẤU TRÚC ĐỀ MINH HỌA BGD NGÀY 22-3-2024 KỲ THI TỐT NGHI...
30 ĐỀ PHÁT TRIỂN THEO CẤU TRÚC ĐỀ MINH HỌA BGD NGÀY 22-3-2024 KỲ THI TỐT NGHI...30 ĐỀ PHÁT TRIỂN THEO CẤU TRÚC ĐỀ MINH HỌA BGD NGÀY 22-3-2024 KỲ THI TỐT NGHI...
30 ĐỀ PHÁT TRIỂN THEO CẤU TRÚC ĐỀ MINH HỌA BGD NGÀY 22-3-2024 KỲ THI TỐT NGHI...Nguyen Thanh Tu Collection
 
GIÁO ÁN DẠY THÊM (KẾ HOẠCH BÀI DẠY BUỔI 2) - TIẾNG ANH 7 GLOBAL SUCCESS (2 CỘ...
GIÁO ÁN DẠY THÊM (KẾ HOẠCH BÀI DẠY BUỔI 2) - TIẾNG ANH 7 GLOBAL SUCCESS (2 CỘ...GIÁO ÁN DẠY THÊM (KẾ HOẠCH BÀI DẠY BUỔI 2) - TIẾNG ANH 7 GLOBAL SUCCESS (2 CỘ...
GIÁO ÁN DẠY THÊM (KẾ HOẠCH BÀI DẠY BUỔI 2) - TIẾNG ANH 7 GLOBAL SUCCESS (2 CỘ...Nguyen Thanh Tu Collection
 
TỔNG HỢP ĐỀ THI CHÍNH THỨC KỲ THI TUYỂN SINH VÀO LỚP 10 THPT MÔN NGỮ VĂN NĂM ...
TỔNG HỢP ĐỀ THI CHÍNH THỨC KỲ THI TUYỂN SINH VÀO LỚP 10 THPT MÔN NGỮ VĂN NĂM ...TỔNG HỢP ĐỀ THI CHÍNH THỨC KỲ THI TUYỂN SINH VÀO LỚP 10 THPT MÔN NGỮ VĂN NĂM ...
TỔNG HỢP ĐỀ THI CHÍNH THỨC KỲ THI TUYỂN SINH VÀO LỚP 10 THPT MÔN NGỮ VĂN NĂM ...Nguyen Thanh Tu Collection
 
Campbell _2011_ - Sinh học - Tế bào - Ref.pdf
Campbell _2011_ - Sinh học - Tế bào - Ref.pdfCampbell _2011_ - Sinh học - Tế bào - Ref.pdf
Campbell _2011_ - Sinh học - Tế bào - Ref.pdfTrnHoa46
 
SÁNG KIẾN ÁP DỤNG CLT (COMMUNICATIVE LANGUAGE TEACHING) VÀO QUÁ TRÌNH DẠY - H...
SÁNG KIẾN ÁP DỤNG CLT (COMMUNICATIVE LANGUAGE TEACHING) VÀO QUÁ TRÌNH DẠY - H...SÁNG KIẾN ÁP DỤNG CLT (COMMUNICATIVE LANGUAGE TEACHING) VÀO QUÁ TRÌNH DẠY - H...
SÁNG KIẾN ÁP DỤNG CLT (COMMUNICATIVE LANGUAGE TEACHING) VÀO QUÁ TRÌNH DẠY - H...Nguyen Thanh Tu Collection
 
1.DOANNGOCPHUONGTHAO-APDUNGSTEMTHIETKEBTHHHGIUPHSHOCHIEUQUA (1).docx
1.DOANNGOCPHUONGTHAO-APDUNGSTEMTHIETKEBTHHHGIUPHSHOCHIEUQUA (1).docx1.DOANNGOCPHUONGTHAO-APDUNGSTEMTHIETKEBTHHHGIUPHSHOCHIEUQUA (1).docx
1.DOANNGOCPHUONGTHAO-APDUNGSTEMTHIETKEBTHHHGIUPHSHOCHIEUQUA (1).docxTHAO316680
 
BỘ LUYỆN NGHE VÀO 10 TIẾNG ANH DẠNG TRẮC NGHIỆM 4 CÂU TRẢ LỜI - CÓ FILE NGHE.pdf
BỘ LUYỆN NGHE VÀO 10 TIẾNG ANH DẠNG TRẮC NGHIỆM 4 CÂU TRẢ LỜI - CÓ FILE NGHE.pdfBỘ LUYỆN NGHE VÀO 10 TIẾNG ANH DẠNG TRẮC NGHIỆM 4 CÂU TRẢ LỜI - CÓ FILE NGHE.pdf
BỘ LUYỆN NGHE VÀO 10 TIẾNG ANH DẠNG TRẮC NGHIỆM 4 CÂU TRẢ LỜI - CÓ FILE NGHE.pdfNguyen Thanh Tu Collection
 
TÀI LIỆU BỒI DƯỠNG HỌC SINH GIỎI KỸ NĂNG VIẾT ĐOẠN VĂN NGHỊ LUẬN XÃ HỘI 200 C...
TÀI LIỆU BỒI DƯỠNG HỌC SINH GIỎI KỸ NĂNG VIẾT ĐOẠN VĂN NGHỊ LUẬN XÃ HỘI 200 C...TÀI LIỆU BỒI DƯỠNG HỌC SINH GIỎI KỸ NĂNG VIẾT ĐOẠN VĂN NGHỊ LUẬN XÃ HỘI 200 C...
TÀI LIỆU BỒI DƯỠNG HỌC SINH GIỎI KỸ NĂNG VIẾT ĐOẠN VĂN NGHỊ LUẬN XÃ HỘI 200 C...Nguyen Thanh Tu Collection
 
3-BẢNG MÃ LỖI CỦA CÁC HÃNG ĐIỀU HÒA .pdf - ĐIỆN LẠNH BÁCH KHOA HÀ NỘI
3-BẢNG MÃ LỖI CỦA CÁC HÃNG ĐIỀU HÒA .pdf - ĐIỆN LẠNH BÁCH KHOA HÀ NỘI3-BẢNG MÃ LỖI CỦA CÁC HÃNG ĐIỀU HÒA .pdf - ĐIỆN LẠNH BÁCH KHOA HÀ NỘI
3-BẢNG MÃ LỖI CỦA CÁC HÃNG ĐIỀU HÒA .pdf - ĐIỆN LẠNH BÁCH KHOA HÀ NỘIĐiện Lạnh Bách Khoa Hà Nội
 
sách sinh học đại cương - Textbook.pdf
sách sinh học đại cương   -   Textbook.pdfsách sinh học đại cương   -   Textbook.pdf
sách sinh học đại cương - Textbook.pdfTrnHoa46
 
Nhiễm khuẩn tiêu hóa-Tiêu chảy do vi khuẩn.pptx
Nhiễm khuẩn tiêu hóa-Tiêu chảy do vi khuẩn.pptxNhiễm khuẩn tiêu hóa-Tiêu chảy do vi khuẩn.pptx
Nhiễm khuẩn tiêu hóa-Tiêu chảy do vi khuẩn.pptxhoangvubaongoc112011
 
30 ĐỀ PHÁT TRIỂN THEO CẤU TRÚC ĐỀ MINH HỌA BGD NGÀY 22-3-2024 KỲ THI TỐT NGHI...
30 ĐỀ PHÁT TRIỂN THEO CẤU TRÚC ĐỀ MINH HỌA BGD NGÀY 22-3-2024 KỲ THI TỐT NGHI...30 ĐỀ PHÁT TRIỂN THEO CẤU TRÚC ĐỀ MINH HỌA BGD NGÀY 22-3-2024 KỲ THI TỐT NGHI...
30 ĐỀ PHÁT TRIỂN THEO CẤU TRÚC ĐỀ MINH HỌA BGD NGÀY 22-3-2024 KỲ THI TỐT NGHI...Nguyen Thanh Tu Collection
 

Recently uploaded (20)

GIÁO TRÌNH KHỐI NGUỒN CÁC LOẠI - ĐIỆN LẠNH BÁCH KHOA HÀ NỘI
GIÁO TRÌNH  KHỐI NGUỒN CÁC LOẠI - ĐIỆN LẠNH BÁCH KHOA HÀ NỘIGIÁO TRÌNH  KHỐI NGUỒN CÁC LOẠI - ĐIỆN LẠNH BÁCH KHOA HÀ NỘI
GIÁO TRÌNH KHỐI NGUỒN CÁC LOẠI - ĐIỆN LẠNH BÁCH KHOA HÀ NỘI
 
TÀI LIỆU BỒI DƯỠNG HỌC SINH GIỎI LÝ LUẬN VĂN HỌC NĂM HỌC 2023-2024 - MÔN NGỮ ...
TÀI LIỆU BỒI DƯỠNG HỌC SINH GIỎI LÝ LUẬN VĂN HỌC NĂM HỌC 2023-2024 - MÔN NGỮ ...TÀI LIỆU BỒI DƯỠNG HỌC SINH GIỎI LÝ LUẬN VĂN HỌC NĂM HỌC 2023-2024 - MÔN NGỮ ...
TÀI LIỆU BỒI DƯỠNG HỌC SINH GIỎI LÝ LUẬN VĂN HỌC NĂM HỌC 2023-2024 - MÔN NGỮ ...
 
Các điều kiện bảo hiểm trong bảo hiểm hàng hoá
Các điều kiện bảo hiểm trong bảo hiểm hàng hoáCác điều kiện bảo hiểm trong bảo hiểm hàng hoá
Các điều kiện bảo hiểm trong bảo hiểm hàng hoá
 
1 - MÃ LỖI SỬA CHỮA BOARD MẠCH BẾP TỪ.pdf
1 - MÃ LỖI SỬA CHỮA BOARD MẠCH BẾP TỪ.pdf1 - MÃ LỖI SỬA CHỮA BOARD MẠCH BẾP TỪ.pdf
1 - MÃ LỖI SỬA CHỮA BOARD MẠCH BẾP TỪ.pdf
 
GNHH và KBHQ - giao nhận hàng hoá và khai báo hải quan
GNHH và KBHQ - giao nhận hàng hoá và khai báo hải quanGNHH và KBHQ - giao nhận hàng hoá và khai báo hải quan
GNHH và KBHQ - giao nhận hàng hoá và khai báo hải quan
 
30 ĐỀ PHÁT TRIỂN THEO CẤU TRÚC ĐỀ MINH HỌA BGD NGÀY 22-3-2024 KỲ THI TỐT NGHI...
30 ĐỀ PHÁT TRIỂN THEO CẤU TRÚC ĐỀ MINH HỌA BGD NGÀY 22-3-2024 KỲ THI TỐT NGHI...30 ĐỀ PHÁT TRIỂN THEO CẤU TRÚC ĐỀ MINH HỌA BGD NGÀY 22-3-2024 KỲ THI TỐT NGHI...
30 ĐỀ PHÁT TRIỂN THEO CẤU TRÚC ĐỀ MINH HỌA BGD NGÀY 22-3-2024 KỲ THI TỐT NGHI...
 
ĐỀ CHÍNH THỨC KỲ THI TUYỂN SINH VÀO LỚP 10 THPT CÁC TỈNH THÀNH NĂM HỌC 2020 –...
ĐỀ CHÍNH THỨC KỲ THI TUYỂN SINH VÀO LỚP 10 THPT CÁC TỈNH THÀNH NĂM HỌC 2020 –...ĐỀ CHÍNH THỨC KỲ THI TUYỂN SINH VÀO LỚP 10 THPT CÁC TỈNH THÀNH NĂM HỌC 2020 –...
ĐỀ CHÍNH THỨC KỲ THI TUYỂN SINH VÀO LỚP 10 THPT CÁC TỈNH THÀNH NĂM HỌC 2020 –...
 
Giới thiệu Dự án Sản Phụ Khoa - Y Học Cộng Đồng
Giới thiệu Dự án Sản Phụ Khoa - Y Học Cộng ĐồngGiới thiệu Dự án Sản Phụ Khoa - Y Học Cộng Đồng
Giới thiệu Dự án Sản Phụ Khoa - Y Học Cộng Đồng
 
30 ĐỀ PHÁT TRIỂN THEO CẤU TRÚC ĐỀ MINH HỌA BGD NGÀY 22-3-2024 KỲ THI TỐT NGHI...
30 ĐỀ PHÁT TRIỂN THEO CẤU TRÚC ĐỀ MINH HỌA BGD NGÀY 22-3-2024 KỲ THI TỐT NGHI...30 ĐỀ PHÁT TRIỂN THEO CẤU TRÚC ĐỀ MINH HỌA BGD NGÀY 22-3-2024 KỲ THI TỐT NGHI...
30 ĐỀ PHÁT TRIỂN THEO CẤU TRÚC ĐỀ MINH HỌA BGD NGÀY 22-3-2024 KỲ THI TỐT NGHI...
 
GIÁO ÁN DẠY THÊM (KẾ HOẠCH BÀI DẠY BUỔI 2) - TIẾNG ANH 7 GLOBAL SUCCESS (2 CỘ...
GIÁO ÁN DẠY THÊM (KẾ HOẠCH BÀI DẠY BUỔI 2) - TIẾNG ANH 7 GLOBAL SUCCESS (2 CỘ...GIÁO ÁN DẠY THÊM (KẾ HOẠCH BÀI DẠY BUỔI 2) - TIẾNG ANH 7 GLOBAL SUCCESS (2 CỘ...
GIÁO ÁN DẠY THÊM (KẾ HOẠCH BÀI DẠY BUỔI 2) - TIẾNG ANH 7 GLOBAL SUCCESS (2 CỘ...
 
TỔNG HỢP ĐỀ THI CHÍNH THỨC KỲ THI TUYỂN SINH VÀO LỚP 10 THPT MÔN NGỮ VĂN NĂM ...
TỔNG HỢP ĐỀ THI CHÍNH THỨC KỲ THI TUYỂN SINH VÀO LỚP 10 THPT MÔN NGỮ VĂN NĂM ...TỔNG HỢP ĐỀ THI CHÍNH THỨC KỲ THI TUYỂN SINH VÀO LỚP 10 THPT MÔN NGỮ VĂN NĂM ...
TỔNG HỢP ĐỀ THI CHÍNH THỨC KỲ THI TUYỂN SINH VÀO LỚP 10 THPT MÔN NGỮ VĂN NĂM ...
 
Campbell _2011_ - Sinh học - Tế bào - Ref.pdf
Campbell _2011_ - Sinh học - Tế bào - Ref.pdfCampbell _2011_ - Sinh học - Tế bào - Ref.pdf
Campbell _2011_ - Sinh học - Tế bào - Ref.pdf
 
SÁNG KIẾN ÁP DỤNG CLT (COMMUNICATIVE LANGUAGE TEACHING) VÀO QUÁ TRÌNH DẠY - H...
SÁNG KIẾN ÁP DỤNG CLT (COMMUNICATIVE LANGUAGE TEACHING) VÀO QUÁ TRÌNH DẠY - H...SÁNG KIẾN ÁP DỤNG CLT (COMMUNICATIVE LANGUAGE TEACHING) VÀO QUÁ TRÌNH DẠY - H...
SÁNG KIẾN ÁP DỤNG CLT (COMMUNICATIVE LANGUAGE TEACHING) VÀO QUÁ TRÌNH DẠY - H...
 
1.DOANNGOCPHUONGTHAO-APDUNGSTEMTHIETKEBTHHHGIUPHSHOCHIEUQUA (1).docx
1.DOANNGOCPHUONGTHAO-APDUNGSTEMTHIETKEBTHHHGIUPHSHOCHIEUQUA (1).docx1.DOANNGOCPHUONGTHAO-APDUNGSTEMTHIETKEBTHHHGIUPHSHOCHIEUQUA (1).docx
1.DOANNGOCPHUONGTHAO-APDUNGSTEMTHIETKEBTHHHGIUPHSHOCHIEUQUA (1).docx
 
BỘ LUYỆN NGHE VÀO 10 TIẾNG ANH DẠNG TRẮC NGHIỆM 4 CÂU TRẢ LỜI - CÓ FILE NGHE.pdf
BỘ LUYỆN NGHE VÀO 10 TIẾNG ANH DẠNG TRẮC NGHIỆM 4 CÂU TRẢ LỜI - CÓ FILE NGHE.pdfBỘ LUYỆN NGHE VÀO 10 TIẾNG ANH DẠNG TRẮC NGHIỆM 4 CÂU TRẢ LỜI - CÓ FILE NGHE.pdf
BỘ LUYỆN NGHE VÀO 10 TIẾNG ANH DẠNG TRẮC NGHIỆM 4 CÂU TRẢ LỜI - CÓ FILE NGHE.pdf
 
TÀI LIỆU BỒI DƯỠNG HỌC SINH GIỎI KỸ NĂNG VIẾT ĐOẠN VĂN NGHỊ LUẬN XÃ HỘI 200 C...
TÀI LIỆU BỒI DƯỠNG HỌC SINH GIỎI KỸ NĂNG VIẾT ĐOẠN VĂN NGHỊ LUẬN XÃ HỘI 200 C...TÀI LIỆU BỒI DƯỠNG HỌC SINH GIỎI KỸ NĂNG VIẾT ĐOẠN VĂN NGHỊ LUẬN XÃ HỘI 200 C...
TÀI LIỆU BỒI DƯỠNG HỌC SINH GIỎI KỸ NĂNG VIẾT ĐOẠN VĂN NGHỊ LUẬN XÃ HỘI 200 C...
 
3-BẢNG MÃ LỖI CỦA CÁC HÃNG ĐIỀU HÒA .pdf - ĐIỆN LẠNH BÁCH KHOA HÀ NỘI
3-BẢNG MÃ LỖI CỦA CÁC HÃNG ĐIỀU HÒA .pdf - ĐIỆN LẠNH BÁCH KHOA HÀ NỘI3-BẢNG MÃ LỖI CỦA CÁC HÃNG ĐIỀU HÒA .pdf - ĐIỆN LẠNH BÁCH KHOA HÀ NỘI
3-BẢNG MÃ LỖI CỦA CÁC HÃNG ĐIỀU HÒA .pdf - ĐIỆN LẠNH BÁCH KHOA HÀ NỘI
 
sách sinh học đại cương - Textbook.pdf
sách sinh học đại cương   -   Textbook.pdfsách sinh học đại cương   -   Textbook.pdf
sách sinh học đại cương - Textbook.pdf
 
Nhiễm khuẩn tiêu hóa-Tiêu chảy do vi khuẩn.pptx
Nhiễm khuẩn tiêu hóa-Tiêu chảy do vi khuẩn.pptxNhiễm khuẩn tiêu hóa-Tiêu chảy do vi khuẩn.pptx
Nhiễm khuẩn tiêu hóa-Tiêu chảy do vi khuẩn.pptx
 
30 ĐỀ PHÁT TRIỂN THEO CẤU TRÚC ĐỀ MINH HỌA BGD NGÀY 22-3-2024 KỲ THI TỐT NGHI...
30 ĐỀ PHÁT TRIỂN THEO CẤU TRÚC ĐỀ MINH HỌA BGD NGÀY 22-3-2024 KỲ THI TỐT NGHI...30 ĐỀ PHÁT TRIỂN THEO CẤU TRÚC ĐỀ MINH HỌA BGD NGÀY 22-3-2024 KỲ THI TỐT NGHI...
30 ĐỀ PHÁT TRIỂN THEO CẤU TRÚC ĐỀ MINH HỌA BGD NGÀY 22-3-2024 KỲ THI TỐT NGHI...
 

Cac bai toan co nhieu loi giai

  • 1. 1 CÁC BÀI TOÁN CÓ NHIỀU LỜI GIẢI Võ Quốc Bá Cẩn Bài 1. (Chọn đội tuyển Rumani 1999) Cho 1 2 ,, , n aa a là các số thực dương thỏa mãn 1 2 1. n a aa Chứng minh rằng 1 2 1 1 1 1 1 1. a n a n an n 1 Chứng minh. Cách 1. Nhân cả hai vế của bất đẳng thức cho 1 n và để ý rằng 1 1 , 1 1 i i i n a a n a n ta viết được nó dưới dạng 1 2 1 2 1. 1 1 1 n n a a a a n a n a n Tới đây, sử dụng bất đẳng thức Cauchy-Schwarz 1 2 1 2 2 1 2 1 2 1 1 1 , ( 1) ( 1) ( 1) n n n n a a a a n a n a n a a a n a n a a n ta đưa được bài toán về chứng minh 2 1 2 1 2 ( 1). n n a a a a a a n n Do 2 1 2 1 1 2 n n i j i j n i i a a a a a a nên bất đẳng thức trên tương đương với 1 ( 1) . i j 2 i j n n n a a Bất đẳng thức này đúng theo bất đẳng thức AM-GM
  • 2. 2 ( 1) 2 1 1 ( 1) ( 1) . 2 2 n n i j i j i j n i j n n n n n a a a a Bài toán được chứng minh xong. Đẳng thức xảy ra khi và chỉ khi 1 2 1. n a a a Cách 2. Giả sử bất đẳng thức đã cho sai. Khi đó tồn tại các số thực dương 1 2 ,, , n aa a thỏa mãn 1 2 1 n aa a sao cho 1 2 1 1 1 1 1 1, 1 n a n a n a n hay 1 2 1 2 1. 1 1 1 n n a a a a n a n a n Từ đây sử dụng bất đẳng thức AM-GM, ta có 1 2 1 2 3 2 3 3 1 1 1 1 1 1 2 3 1 1 1 1 1 1 ( 1) ( 1)( 1) ( 1) 1 ( 1) . ( 1 1 ) n n n n n n n i i a a n a a a n a a a a n a n a n n a n a n a n a n n a n a n a Suy ra 1 1 1 1 ( 1 1. ( 1 ) ) i n n i a a a n n Tương tự ta cũng có 2 3 1 1 1 2 3 1 1, 1 1, 1 1. ( 1) ( ( 1) 1) ( 1 ( 1) ( , ) 1) n n n n n n n i i i i i i n a a a a n a n a n a n a n a n Nhân các bất đẳng thức này lại theo vế ta thu được 1 1 (vô lý). Mâu thuẫn này chứng tỏ điều giả sử ở trên là sai, hay nói một cách khác, bài toán được chứng minh. Cách 3. Thực hiện biến đổi tương tự như cách 1, ta phải chứng minh
  • 3. 3 1 2 1 2 1. 1 1 1 n n a a a a n a n a n Sử dụng bất đẳng thức AM-GM, ta có 1 1 1 1 1 1 1 1 2 1 2 3 1 1 1 1 1 1 1 1 2 3 1 1 1 1 1 1 1 2 3 1 ( 1) ( 1) ( 1) . n n n n n n n n n n n n n n n n n n n n n n n n n n n n n n n a a a a n a n a a a n a a a a n a a a a a a a a a a Tương tự 1 2 2 1 1 1 2 1 2 1 3 3 1 1 1 3 1 2 1 1 1 1 1 2 , 1 , . 1 , 1 n n n n n n n n n n n n n n n n n n n n n n n n n n n n n n a a a n a a a a a a n a a a a a a n a a a Cộng n bất đẳng thức trên lại theo vế, ta có ngay điều phải chứng minh. Cách 4. Ta sẽ chứng minh mệnh đề tổng quát như sau: Nếu 1 2 , , , n a a a là các số thực dương thỏa mãn 1 2 1 n a a a và 1, n m n thì 1 2 1 1 1 . 1 n n n n n a m a m n a m m Kết quả bài toán ứng với trường hợp 1. n m n Chứng minh. Ta sử dụng phương pháp quy nạp Toán học.
  • 4. 4 Với 2, n bất đẳng thức trở thành 1 2 2 2 2 1 1 , 1 n a m a m m đúng vì 1 1 2 2 2 2 1 2 2 1 2 2 1 2 1 2 2 1 2 1 1 2 1 2 1 1 1 ( 1) ( 1) 0. ( )(1 )(1 ) a a m a m m a m m a m a m a m m a m Giả sử bất đẳng thức đúng với 2.nk Ta sẽ chứng minh nó cũng đúng với 1,nk tức là: Nếu 1 2 1 , , , , k k a a a a là các số thực dương thỏa mãn 1 2 1 1 k k a a a a và 1 , k m k thì 1 1 2 1 1 1 1 1 1 1 1 1 1 . 1 k k k k k k k k a m a m a m a m m Không mất tính tổng quát giả sử 1 2 1, k k a a a a suy ra 1 1. k a Đặt 1 1 2 1 1 1, 2, , , 1 k , i , k . k i k k k a i k a m b b a a b m a b Khi đó ta có 1 2 1 k bb b và 1 1 k k m m k k nên theo giả thiết quy nạp, ta sẽ có 1 2 1 1 . 1 1 k k k k k b m b m k b m m Thay i i a b b và k 1 k m m b vào rồi rút gọn, ta thu được ngay 1 1 2 1 1 1 1 1 . 1 k k k k k k a m a m a m b m Từ đánh giá này suy ra ta chỉ cần chứng minh (chú ý 1 1 k k a b ) 1 1 1 1 . 1 1 k k k k k k b k b m m b m Bất đẳng thức này tương đương với
  • 5. 5 1 1 1 1 1 1 1 1 1 (1 ) (1 ) 1 , 1 1 (1 ) , 1 (1 ) ( )(1 ) (1 ) 0, k k k k k k k k k k k k k k b m k m k m b b m b k b m b b m b b m b b k m b 1 2 1 2 1 1 1 (1 ) (1 ) 0, (1 ) (1 2 ) ( 1) 0. k k k k k k k b m b b kb b b b k b m b kb k k b b Bất đẳng thức cuối cùng đúng do 1 1 1 1 1 ( 1) (1 ) (1 2 ) (1 2 ). k k k k k k k b b k b b k b kb m b kb Như vậy, ta đã chứng minh được bất đẳng thức cũng đúng với n k 1. Theo nguyên lý quy nạp, ta có nó đúng với mọi 2. n Bài 2. Cho a, b, c, d là các số thực dương thỏa mãn abcd 1. Chứng minh bất đẳng thức sau 2 2 2 2 1 1 1 1 1. (1 a) (1 b) (1 c) (1 d) Chứng minh. Cách 1. Trước hết ta chứng minh bổ đề sau. Bổ đề. Nếu , xy là các số thực dương, thì 2 2 1 1 1 . (1 x) (1 y) 1 xy Chứng minh. Cách 1. Sử dụng bất đẳng thức Cauchy-Schwarz, ta có 2 ( )(1 ) (1 ) 1 (1 ) , x x y xy x xy y y suy ra 2 1 . (1 ) ( )(1 ) y x x y xy Tương tự ta cũng có
  • 6. 6 2 1 . (1 ) ( )(1 ) x y x y xy Cộng hai bất đẳng thức này lại, ta được 2 2 1 1 1 . (1 ) (1 ) ( )(1 ) 1 y x x y x y xy xy Bổ đề được chứng minh. Cách 2. Tiến hành biến đổi trực tiếp, ta có 2 2 2 2 2 2 2 2 2 2 2 2 1 1 1 ( ) 2 1 (1 ) (1 ) 1 (1 ) (1 ) (1 ) ( ) ( 1) 0. (1 ) (1 ) (1 ) xy x y x y xy x y xy x y xy xy x y xy x y xy Trở lại bài toán. Sử dụng bổ đề trên ta được 2 2 2 2 1 1 1 , (1 ) (1 ) 1 1 1 1 1 . (1 ) (1 ) 1 1 1 1 a b ab ab c d cd ab ab Từ đó suy ra 2 2 2 2 1 1 1 1 1 1. (1 ) (1 ) (1 ) (1 ) 1 ab a b c d ab Đẳng thức xảy ra khi và chỉ khi a b c d 1. Cách 2. Đặt 5 4 2 3 5 3 2 5 2 4 1 1 1 x , y , z a b c ab c a bc và 1, t ta dễ dàng kiểm tra được 2 2 2 2 , , , . yz zt tx xy a b c d x y z t Thay bất đẳng thức đã cho, ta viết được nó lại thành 4 4 4 4 2 2 2 2 2 2 2 2 1. ( ) ( ) ( ) ( ) x y z t x yz y zt z tx t xy Sử dụng bất đẳng thức Cauchy-Schwarz, ta được
  • 7. 7 4 4 4 4 2 2 2 2 2 2 2 2 2 2 2 2 2 2 2 2 2 2 2 2 2 ( ) ( ) ( ) ( ) ( ) . ( ) ( ) ( ) ( ) x y z t x yz y zt z tx t xy x y z t x yz y zt z tx t xy Mặt khác 2 2 2 2 2 2 2 2 2 2 2 2 2 2 2 2 2 2 2 2 2 2 2 2 2 2 2 2 2 2 2 2 2 2 2 2 ( ) ( ) ( )( ) ( )( ) ( )( ), ( ) ( ) ( )( ) ( )( ) ( )( ). x yz z tx x y x z z t z x x y z t x z y zt t xy y z y t t x t y x y z t y t Kết hợp với trên, ta suy ra 4 4 4 4 2 2 2 2 2 2 2 2 2 2 2 2 2 2 2 2 2 2 2 2 2 2 2 2 2 ( ) ( ) ( ) ( ) ( ) 1. ( )( ) ( )( ) x y z t x yz y zt z tx t xy x y z t x y z t x z x y z t y t Cách 3. Đặt x 1, y a, z ab và t abc, ta dễ thấy , , y z a b x y t c z và . x d t Thay vào bất đẳng thức đã cho, ta được 2 2 2 2 2 2 2 2 1. ( ) ( ) ( ) ( ) x y z t x y y z z t t x Gọi P là vế trái của bất đẳng thức trên. Sử dụng lần lượt các bất đẳng thức Cauchy-Schwarz và AM-GM, ta có 2 2 2 2 2 2 2 2 2 2 2 2 2 2 2 2 2 2 2 2 2 2 2 2 ( ) ( ) ( ) ( ) ( ) ( ) ( ) ( ) ( ) ( ) ( ) ( ) ( ) ( ) ( ) ( ) ( ) ( ) ( ) ( ) ( ) 4 ( ) ( ) x x t y y x z z y t t z P x y x t y z y x z t z y t x t z x y z t xy yz zt tx x y z t x t y z x y z t x t y z x y z t 2 2 2 1. (x t) (y z)
  • 8. 8 Cách 4. Đặt 4 4 4 4 1 1 1 1 a , b , c , d x y z t ( , , , 0) x y z t thì ta cũng có xyzt 1. Khi đó 8 8 6 2 4 2 4 2 3 2 1 . (1 ) ( 1) ( ) ( ) x x x a x x xyzt x yzt Suy ra bất đẳng thức cần chứng minh có thể viết lại thành 6 6 6 6 3 2 3 2 3 2 3 2 1. ( ) ( ) ( ) ( ) x y z t x yzt y ztx z txy t xyz Tới đây, sử dụng bất đẳng thức Cauchy-Schwarz, ta đưa được bài toán về chứng minh (x3 y3 z3 t3)2 (x3 yzt)2 (y3 ztx)2 (z3 txy)2 (t3 xyz)2. Thực hiện khai triển và rút gọn, ta được 2 3 3 2 3 2 2 2. sym x y x yzt y z t Bất đẳng thức này đúng vì theo bất đẳng thức AM-GM, ta có 3 3 3 3 3 3 3 3 3 3 3 3 3 2 2 2 3 3 4 2 2 , 3 3 2 . 3 3 sym sym y z t x yzt x x y y z z t t y y z t x y Phép chứng minh hoàn tất. Bài 3. (Olympic Toán Ukraine 2001) Chứng minh rằng với mọi số thực dương a, b, c, x, y, z, bất đẳng thức sau đây luôn được thỏa mãn (ay az bz bx cx cy)2 4(ab bc ca)(xy yz zx). Chứng minh. Cách 1. Áp dụng bất đẳng thức AM-GM, ta có 2 ( ) ( ) 4 ( ) ( ) ( ) ( ) ( ) ( ) . bc y z yz b c VP a y z x b c y z y z bc y z yz b c a y z x b c y z y z Do đó, để chứng minh bất đẳng thức đã cho, ta chỉ cần chứng minh được
  • 9. 9 ( ) ( ) ( ) ( ) . bc y z yz b c ay az bz bx cx cy a y z x b c y z y z Bất đẳng thức này tương đương với ( ) ( ) , bc y z yz b c bz cy b c y z 2 2 ( ) . b z c y yz b c b c y z Tới đây, sử dụng bất đẳng thức Cauchy-Schwarz, ta được 2 2 2 1 1 (b z c y) (b c) . z y Từ đó suy ra 2 2 ( ) . 1 1 b z c y b c yz b c b c y z z y Phép chứng minh được hoàn tất. Cách 2. Lấy căn bậc hai của hai vế và chú ý rằng ay az bz bx cx cy (a b c)(x y z) (ax by cz), ta có thể viết lại bất đẳng thức cần chứng minh dưới dạng ax by cz 2 (ab bc ca)(xy yz zx) (a b c)(x y z). Đến đây, sử dụng bất đẳng thức Cauchy-Schwarz, ta có 2 2 2 2 2 2 2 2 2 2 2 2 ( )( ) 2( ) 2( ) [ 2( )][ 2( )] ( )( ) . VT a b c x y z ab bc ca xy yz zx a b c ab bc ca x y z xy yz zx a b c x y z VP Cách 3. Tương tự như cách 2, ta phải chứng minh rằng ax by cz 2 (ab bc ca)(xy yz zx) (a b c)(x y z). Để ý rằng đây là một bất đẳng thức thuần nhất cho các bộ số (a, b, c) và (x, y, z). Vì thế, không giảm đi tính tổng quát, ta hoàn toàn có thể giả sử rằng a b c x y z 1. Lúc này, áp dụng bất đẳng thức AM-GM, ta có
  • 10. 10 2 2 2 2 2 2 2 2 2 2 2 2 2 2 ( ) ( ) 2 2 2 2( ) 2( ) 2 2 ( ) ( ) 1 ( )( ). 2 2 a x b y c z VT ab bc ca xy yz zx a b c ab bc ca x y z xy yz zx a b c x y z a b c x y z Bài 4. (IMO 2008) Cho ,, 1 x y z và 1. xyz Chứng minh rằng 2 2 2 1. 1 1 1 x y z x y z Chứng minh. Cách 1. Đặt , , , 1 1 1 x y z a b c x y z khi đó ta có , , . 1 1 1 a b c x y z a b c Do giả thiết 1xyz nên ( 1)( 1)( 1), abc a b c suy ra a b c ab bc ca 1. Bất đẳng thức cần chứng minh được đưa về a2 b2 c2 1. Để ý rằng 1 2 1 2(a b c ab bc ca) 1, ta có thể viết lại bất đẳng thức trên dưới dạng a2 b2 c2 2(a b c ab bc ca) 1. Không mấy khó khăn, ta có thể phân tích bất đẳng thức cuối thành (a b c 1)2 0 là một bất đẳng thức hiển nhiên đúng. Chứng minh hoàn tất. Cách 2. Do x, y, z 1 và xyz 1 nên tồn tại các số thực a, b, c sao cho 2 2 2 , , bc ca ab x y z a b c (chẳng hạn 3 3 3 1 1 1 a ,b ,c x y z ). Khi đó, bất đẳng thức đã cho được viết lại dưới dạng thuần nhất là 4 4 4 2 2 2 2 2 2 1. ( ) ( ) ( ) a b c a bc b ca c ab Áp dụng bất đẳng thức Cauchy-Schwarz, ta có
  • 11. 11 2 2 2 2 2 2 2 2 2 2 ( ) . ( ) ( ) ( ) a b c VT a bc b ca c ab Lại có (a2 b2 c2)2 (a2 bc)2 (b2 ca)2 (c2 ab)2 (ab bc ca)2 0, nên kết hợp với trên, ta dễ dàng suy ra được điều phải chứng minh. Cách 3. Dễ thấy tồn tại các số thực phân biệt ,,a b c sao cho , a x b ,. bc yz ca Thay vào, bất đẳng thức cần chứng minh trở thành 2 2 2 2 2 2 1. ( ) ( ) ( ) a b c a b b c c a Áp dụng bất đẳng thức Cauchy-Schwarz, ta có 2 2 2 VT (a b) (a c) a(a c) b(b a) c(c b) . Đến đây, với chú ý ở đẳng thức xy yz zx 0 trong đó x (a b)(a c), y (b c)(b a), z (c a)(c b), ta thấy 2 2 2 2 2 2 2 2 2 ( ) ( ) 2( ) ( ) , a b a c x y z x y z xy yz zx x y z mà x y z (a b)(a c) a(a c) b(b a) c(c b), nên từ trên, ta có ngay điều phải chứng minh. Bài 5. Cho A, B, C là ba góc của một tam giác nhọn. Chứng minh rằng cos2 cos2 cos2 1 . cos 1 cos 1 cos 1 2 A B C A B C Chứng minh. Cách 1. Đặt x cotA, y cotB và z cotC thì ta có x, y, z 0 và xy yz zx 1. Sử dụng bất đẳng thức AM-GM, ta có
  • 12. 12 2 2 2 2 2 2 2 2 2 2 3 3 2 2 2 3 3 2 cos 1 1 cos 1 1 1 1 1 1 1 ( )( ) . 2( ) 2( ) x A x x x x x A x x x x x x x x x x x x y x z x x x x y x z Tương tự ta cũng có 2 3 3 2 2 3 3 2 cos , cos 1 2( ) 2( ) cos . cos 1 2( ) 2( ) B y y y B y z y x C z z z C z x z y Từ đây suy ra 2 3 3 3 3 3 3 2 2 2 2 2 2 2 2 2 2 2 2 cos cos 1 2( ) 2( ) 2( ) 2 2 2 1 . 2 2 A x y y z z x x y z A x y y z z x x xy y y yz z z zx x x y z xy yz zx Bài toán được chứng minh xong. Đẳng thức xảy ra khi và chỉ khi tam giác đã cho là tam giác đều. Cách 2. Đặt x cosA, y cosB và z cosC thì ta có x, y, z 0 và x2 y2 z2 2xyz 1. Bất đẳng thức cần chứng minh trở thành 2 2 2 2 2 2 1. 1 1 1 x y z x y z Thay 1 x2 y2 z2 2xyz và để ý rằng 2 2 2 2 (1 ) , 1 1 x x x x x x ta có thể viết lại bất đẳng thức trên dưới dạng 2(1 ) 2(1 ) 2(1 ) 2 . 1 1 1 x x y y z z xyz x y z Áp dụng bất đẳng thức AM-GM, ta có
  • 13. 13 2 2 2 2 2 2 3 (1 ) (1 ) (1 ) (1 )(1 )(1 ) 3 . 1 1 1 (1 )(1 )(1 ) x x y y z z x y z x y z x y z x y z Sử dụng các kết quả quen thuộc trong tam giác nhọn (1 cos )(A 1 cosB)(1 cosC) cosAcosB cosC, 3 cos cos cos , 2 A B C ta thu được (1 x)(1 y)(1 z) xyz và 3 . 2 x y z Từ đó dẫn đến 2 2 2 2 2 2 3 3 (1 ) (1 ) (1 ) (1 )(1 )(1 ) 3 1 1 1 (1 )(1 )(1 ) 3 (1 )(1 )(1 ) 9 9 2 . 3 3 3 2 x x y y z z x y z x y z x y z x y z xyz x y z xyz xyz xyz x y z Cách 3. Thay 2 2 2 cos 2 b c a A bc vào, ta được 2 2 2 2 2 2 2 2 2 2 2 2 2 2 cos 2 ( ) . cos 1 2[ ( ) ] 1 2 b c a A bc b c a A b c a bc b c a bc bc Do đó, bất đẳng thức cần chứng minh tương đương với 2 2 2 2 2 2 2 2 2 2 2 2 2 2 2 2 2 2 ( ) ( ) ( ) 1. ( ) ( ) ( ) b c a c a b a b c bc b c a bc ca c a b ca ab a b c ab Áp dụng bất đẳng thức Cauchy-Schwarz, ta được 2 2 2 2 2 2 2 ( ) . ( ) ( ) ( ) ( ) a b c VT bc b c ca c a ab a b abc a b c Mà (a2 b2 c2)2 bc(b c)2 abc(a b c) a2(a b)(a c) 0 theo bất đẳng thức Schur bậc 4 nên kết hợp với trên, ta có ngay điều phải chứng minh.
  • 14. 14 Bài 6. Chứng minh rằng với mọi số thực dương , , , a b c ta luôn có 3 3 1 1 1 3 . a(b 1) b(c 1) c(a 1) abc 1 abc Chứng minh. Cách 1. Áp dụng kết quả quen thuộc (x y z)2 3(xy yz zx), ta có thể đưa bài toán về chứng minh 2 3 2 2 2 3 1 3 , ( 1)( 1) 1 ab b c a b c abc hay là 2 3 2 2 2 3 3 . ( 1)( 1)( 1) 1 a b c ab bc ca abc a b c a b c abc Bây giờ, để ý rằng (a 1)(b 1)(c 1) (abc 1) (a b c ab bc ca), a b c ab bc ca 33 abc 33 a2b2c2 và hàm số ( ) t f t t k luôn tăng với mọi t, k 0. Do đó, ta dễ dàng thu được 3 3 2 2 2 3 3 2 2 2 2 3 2 2 2 3 3 3 ( 1)( 1)( 1) ( 1) 3 3 3 . 1 a b c ab bc ca abc a b c abc a b c abc abc abc a b c a b c abc Phép chứng minh hoàn tất. Đẳng thức xảy ra khi và chỉ khi a b c. Cách 2. Để ý rằng 1 1 ( 1) 1 , ( 1) ( 1) 1 abc a b c a b a b a ta có 3 3 1 1 1 1 ( 1) 3 ( 1) ( 1) ( 1) ( 1) 1 3 3 . abc abc abc a b c a b b c c a a b b abc abc
  • 15. 15 Từ đó, ta được 3 3 3 3 3 3 3 1 1 1 3 . ( 1) ( 1) ( 1) 1 1 abc abc a b b c c a abc abc abc Cách 3. Đặt 3, abc k khi đó ta dễ dàng chứng minh được tồn tại các số dương ,, x y z sao cho , , . ky kz kx a b c x y z Với phép đặt ẩn này, bất đẳng thức của ta được đưa về 1 1 1 3 , ( 1) 1 1 1 ky kz kz kx kx ky k k x y y z z x tương đương 3 . 1 x y z y kz z kx x ky k Áp dụng bất đẳng thức Cauchy-Schwarz, ta có 2 2 ( ) ( ) ( ) ( ) ( ) 3 . ( 1)( ) 1 x y z x y z y kz z kx x ky x y kz y z kx z x ky x y z k xy yz zx k Bất đẳng thức được chứng minh xong. Bài 7. Cho a, b, c, d là các số thực thỏa mãn ad bc 1. Chứng minh a2 b2 c2 d2 ac bd 3. Chứng minh. Cách 1. Thay 3 3(ad bc) vào, ta viết được bất đẳng thức dưới dạng a2 b2 c2 d2 ac bd 3(ad bc), tương đương a2 b2 c2 d2 a 3d c b 3c d . Sử dụng bất đẳng thức AM-GM, ta có
  • 16. 16 2 2 2 2 3 3 3 , 3 . 4 4 d c c d a d c a b c d b Từ đó suy ra 2 2 2 2 2 2 2 2 3 3 3 3 4 . d c c d a d c b c d a b a b c d Chứng minh hoàn tất. Đẳng thức xảy ra khi và chỉ khi 1 2 3 . 2 3 ad bc a d c b c d Cách 2. Để ý rằng 2 2 2 2 2 2 2 2 3 1 ( ) ( ) , 4 4 1 3 ( ) ( ) . 4 4 a c ac a c a c b d bd d b d b Do đó bất đẳng thức cần chứng minh có thể viết lại thành 3(a c)2 (d b)2 (a c)2 3(d b)2 4 3. Sử dụng bất đẳng thức AM-GM, ta được 2 2 2 2 3( ) ( ) 2 3( )( ), ( ) 3( ) 2 3( )( ). a c d b a c d b a c d b a c d b Từ đây suy ra VT 2 3 (a c)(d b) (a c)(d b) 4 3(ad bc) 4 3. Cách 3. Sử dụng giả thiết ad bc 1, ta có (a2 b2)(c2 d2) (ac bd)2 (ad bc)2 (ac bd)2 1. Áp dụng bất đẳng thức AM-GM, suy ra 2 2 2 2 2 2 2 2 2 2 2 2 2 ( ) ( ) 2 ( )( ) 2 ( ) 1. a b c d a b c d a b c d ac bd Do đó ta ta chỉ cần chứng minh 2 1 x2 x 3 với x ac bd.
  • 17. 17 Rõ ràng vế trái của bất đẳng thức này luôn dương, vì thế ta có thể lấy bình phương hai vế để thu được bất đẳng thức tương đương 2 2 1 x2 x 3. Ta có 2 2 2 2 2 2 2 2 2 2 2 1 4(1 ) 4 1 (1 ) 4 1 4 3 1 2 3 3. x x x x x x x x x x x x Bài toán được chứng minh xong. Bài 8. (Chọn đội tuyển Iran 2009) Cho ,,a b c là các số thực dương thỏa mãn a b c 3. Chứng minh rằng 2 2 2 2 2 2 1 1 1 3 . a b 2 b c 2 c a 2 4 Chứng minh. Cách 1. Bất đẳng thức cần chứng minh tương đương với 2 2 2 2 2 2 2 2 2 2 2 2 3 . 2 2 2 2 a b b c c a a b b c c a Sử dụng bất đẳng thức Cauchy-Schwarz, ta được 2 2 2 2 2 2 2 2 2 2 2 2 2 2 2 2 2 2 2 2 2 2 2 ( 2) ( 2) ( 2) ( )( ) . 3 a b b c c a VT a b b c c a a b c a b a c a b c Do đó ta chỉ cần chứng minh 2(a2 b2 c2) 2 (a2 b2)(a2 c2) 3(a2 b2 c2 3). Bất đẳng thức này tương đương với 2 2 2 2 2 2 2 2 2 2 2 2 2 2 2 2 2 2 2 2 2 2 2 ( )( ) 9, 2 ( )( ) ( ) , ( )( ) . a b a c a b c a b a c a b c a b c a b a c a b c ab bc ca
  • 18. 18 Bất đẳng thức cuối đúng vì theo bất đẳng thức Cauchy-Schwarz, ta có (a2 b2)(a2 c2) (a2 bc) a2 b2 c2 ab bc ca. Bài toán được chứng minh xong. Đẳng thức xảy ra khi và chỉ khi a b c 1. Cách 2. Tương tự như cách 1, ta phải chứng minh 2 2 2 2 2 2 2 2 2 2 2 2 3 , 2 2 2 2 a b b c c a a b b c c a tương đương 2 2 2 2 2 2 2 2 2 2 2 2 2 2 2 ( ) ( ) ( ) 3 . 2( ) 2( ) 2( ) 2 ( ) ( ) ( ) a b b c c a a b b c c a a b b c c a a b b c c a Áp dụng bất đẳng thức Cauchy-Schwarz, ta được 2 2 2 2 2 2 2 2 2 2 2 2 2 4( ) . 2( ) 2( ) 2( ) ( ) ( ) ( ) a b c VT a b b c c a a b b c c a a b b c c a Như vậy, bất đẳng thức đã cho sẽ được chứng minh nếu ta có 2 2 2 2 2 2 2 2 2 2 2 2 2( ) 2( ) 2( ) ( ) ( ) ( ) 24. a b b c c a a b b c c a a b b c c a Đến đây, ta thấy 2 2 2 2 4 1 12 ( ) ( ) ( ) ( ) 3 3 a b a b c a b a b và 2 2 2 2 2 2 2 2 2 2 2 2 2 2 2 2 2 2 2( ) 2( ) 2( ) 2( ) 2( ) 2( ) 12 a b b c c a a b b c c a a b b c c a a b b c c a nên bất đẳng thức này tương đương với 2 2 2 2 2 2 2 2 2 6 6 6 (a b) 1 (b c) 1 (c a) 1 0. a b b c c a Với giả thiết a b c, ta thấy ngay bất đẳng thức cuối hiển nhiên đúng nếu a2 b2 6, và vì thế bài toán của ta cũng được chứng minh trong trường hợp này. Xét trường hợp ngược lại a2 b2 6, khi đó ta có 2 2 1 1 a b 2 8 và
  • 19. 19 do 2 2 2 2 2 2 2 2 2 1 1 1 1 1 1 2 2 2 2 8 2 1 1 ( 0 6). 8 2 a c b c a b b b b Điều này dẫn đến 2 2 2 2 2 2 1 1 1 1 1 1 3 , a b 2 b c 2 c a 2 8 8 2 4 và bài toán của ta được giải quyết hoàn toàn. Cách 3. Trước hết, ta chứng minh kết quả sau: Nếu ,,x y z là các số thực dương, thì 1 1 1 3 . 2( ) x y z y z z x x y xy yz zx x y z Nhân hai vế của bất đẳng thức này cho xy yz zx và để ý rằng , xy yz zx yz x y z y z ta viết được nó dưới dạng 3( ) . 2( ) yz zx xy xy yz zx y z z x x y x y z Nhân tiếp hai vế cho x y z rồi sử dụng bất đẳng thức AM-GM, ta có ( ) ( ) , 4 ( ) ( ) , 4 ( ) ( ) . 4 yz x y z xyz x y z yz yz y z y z zx x y z yzx y z x zx zx z x z x xy x y z zxy z x y xy xy x y x y Cộng ba bất đẳng thức này lại, ta được 3 ( ) ( ). 2 xy yz zx x y z xy yz zx x y y z z x Từ đây ta thu được kết quả như trên. Bây giờ, áp dụng bất đẳng thức trên với x a2 1, y b2 1, z c2 1, ta được
  • 20. 20 2 2 2 2 2 2 2 2 2 2 2 2 2 2 2 2 2 2 2 2 2 2 2 2 2 2 2 2 2 2 2 1 3 3 2 ( 1)( 1) 2( 3) 3 3 2( ) 3 2( 3) 3 3 . 2( 3) 2( ) 3 3 a b c a b a b a b c a b c a b a b c a b c a b c ab a b c a b c Vậy ta chỉ cần chứng minh 2 2 2 2 2 2 2 2 2 2 3 3 3 . 2( 3) 4 2( ) 3 3 a b c ab a b c a b c Đặt q ab bc ca, ta có a2 b2 c2 9 2q. Thay vào, bất đẳng thức trên được viết lại thành 2 3 2 9 2 3 3 3 , 2(9 2 3) 4 2(9 2 ) 3 3 3( 3) 0. 4( 12 63)(6 ) q q q q q q q q Bất đẳng thức cuối đúng do ( )2 3. 3 a b c q ab bc ca Bài 9. Cho a, b, c, d là các số thực thỏa mãn (a2 1)(b2 1)(c2 1)(d2 1) 16. Chứng minh rằng 3 ab ac ad bc bd cd abcd 5. Chứng minh. Cách 1. Để ý rằng 2 2 2 2 2 2 2 ( 1)( 1) (1 ) ( ) , ( 1)( 1) ( 1) ( ). a b ab a b c d cd c d Từ đó, sử dụng bất đẳng thức Cauchy-Schwarz, ta được
  • 21. 21 2 2 16 (1 )( 1) ( )( ) ( 1) . ab cd a b c d ab ac ad bc bd cd abcd Từ đây ta suy ra 3 ab ac ad bc bd cd abcd 5 là điều phải chứng minh. Cách 2. Do a2 1 a2 i2 (a i)(a i) nên 16 (a i)(a i) (a i) (a i) . Ta có 2 3 4 2 3 4 ( ) 1 1 , ( ) 1 1 . sym sym sym sym sym sym a i abcd i abc i ab i a i abcd i abc ab i a abcd ab i a abc a i abcd i abc i ab i a i abcd i abc ab i a abcd ab i a abc Do đó tích (a i) (a i) có dạng (A Bi)(A Bi) A2 B2, từ đây suy ra 2 2 2 16 1 1 , sym sym abcd ab a abc abcd ab suy ra 3 ab ac ad bc bd cd abcd 5. Bài toán được chứng minh xong. Cách 3. Bất đẳng thức cần chứng minh tương đương với 2 4 1 4, ( ) 1 4, ( ) 1 16. ab ac ad bc bd cd abcd a b c d bcd bc bd cd a b c d bcd bc bd cd Áp dụng bất đẳng thức Cauchy-Schwarz, ta được
  • 22. 22 2 2 2 2 2 2 2 ( 1) ( ) ( 1) ( 1)( 1)( 1)( 1) 16. VT a b c d bcd bc bd cd a b c d Suy ra ta chỉ cần chứng minh (b c d bcd)2 (bc bd cd 1)2 (b2 1)(c2 1)(c2 1). Thực hiện khai triển và rút gọn, ta được ngay một hằng đẳng thức đúng. Bài 10. (Olympic Toán Belarus 1998) Cho ,,a b c là các số thực dương. Chứng minh rằng 1. a b c a b b c b c a b c a b Chứng minh. Cách 1. Bất đẳng thức cần chứng minh tương đương với ( 2 )2 1 2 . ( )( ) a b c a b b c a b c b c a b c a b a b b c Sử dụng bất đẳng thức Cauchy-Schwarz, ta có 2 2 2 2 2 2 2 2 ( ) 1 ( 2 ) . ( )( ) a b c a b c b a b c b b c a ab bc ca b ab bc ca b a b c a b b c Bài toán được chứng minh xong. Đẳng thức xảy ra khi và chỉ khi a b c. Cách 2. Nhân cả hai vế của bất đẳng thức cho , bc ta được 2 2 2 2 2 2 2 ( ) ( ) ( ) ( ) 2 , ( ) , ( ) 2 2 0. a a b b b c c b c b c a b c b c a a b ac bc c b b c b c b a a c a b ac bc b c b c c c b b b a c a a b Bất đẳng thức cuối cùng đúng do 2 2 2 2 2 2 2 2 , 2 2 , ( ) . ac bc ac bc b b c c c b b a b a c c c c b c b b a a b a b
  • 23. 23 Cách 3. Bất đẳng thức cần chứng minh tương đương với 2 1, 2 . ( ) ( ) ( ) a a b b c c b b b c c b c a a b a b ac b bc a b b b c c b c a a b a b Áp dụng bất đẳng thức Cauchy-Schwarz, ta có 2 2 ( ) ( ) ( ) ( ) ( ) ( ) . ac b bc a b c b c a b b b c c b c a a b b ac ab bc b c a Bài toán được quy về chứng minh 2 2 ( ) ( ) ( ) . ab bc ca a b a b c b c a b c a b a b b Bất đẳng thức này tương đương với ( 2 )(2 ) 2 2 2 2 4 2 , ab bc ca a b b c ac a b c a b c c a b b b 2 ab bc b c a (đúng theo AM-GM). Bài toán được chứng minh xong. Cách 4. Đặt , , a c x y b b ta có 1 1 , , . 1 1 c y a b x b c y a x b c y a b x Bất đẳng thức cần chứng minh có thể viết lại thành 1 1 1 1. 1 1 y x y x y x y x Sau khi khai triển và rút gọn, ta được x3y2 x2 x y3 y2 x2y 2xy 2xy2. Bất đẳng thức cuối này đúng vì theo AM-GM, ta có 3 2 3 2 3 3 2 , 2 2, 2 2 2 . 2 2 x y x x y x y y x y xy x y xy Cách 5. Bất đẳng thức cần chứng minh tương đương với
  • 24. 24 2 2 2 2 ( ) 3 ( ) 2 , ( )( ) 2 2 2 . ( )( ) a b c a b b c a b c a b c b c a b c a b a b c ab bc ca a b c a c a b c b c a c a b a b b c Do 2( )2 2( )2 2( )2 0 2 ab bc ca a b c b c a c a b a b c c a b abc và a2 b2 c2 (a b)2 (b c)2 (c a)2 a b c b c a b c a nên ta chỉ cần chứng minh ( )2 ( )2 ( )2 ( )( )2 ( )( ). a b b c c a a b c a c b c a a b b c Sử dụng bất đẳng thức Cauchy-Schwarz, ta có 2 ( )2 ( )2 ( ) ( ) ( )2 . a b b c a b b c a c b c b c b c Suy ra ta chỉ cần chứng minh ( )2 ( )2 ( )( )2 . ( )( ) a c a c a b c a c b c a a b b c Bất đẳng thức cuối này đúng vì 1 1 ( ) 0. ( )( ) ( )( ) a b c b a b c a b c a b b c a a b b c Bài 11. Chứng minh rằng với mọi a, b, c dương, ta đều có 4 2 2 4 2 2 4 2 2 2 2 2 2 2 2 . 2 2 2 a b c b c a c a b a b c a bc b ca c ab Chứng minh. Cách 1. Sử dụng tính thuần nhất, ta có thể chuẩn hóa cho abc 1. Bất đẳng thức cần chứng minh trở thành 6 6 6 3 3 3 2 2 2 .. ( 2) ( 2) ( 2) a b c a b c a a b b c c Áp dụng bất đẳng thức Holder, ta có
  • 25. 25 3 (a6 1 1)(a2 a 1)(a2 1 a) 3 a10 3 a 3 a a(a3 2)3, từ đó suy ra 6 3 3 2 2 2 2 3 1. ( 2) 1 1 a a a a a a a a a Bằng cách thiết lập hai bất đẳng thức tương tự cho hai biểu thức còn lại, ta có thể đưa bài toán về chứng minh 2 2 2 1 1 1 1. a a 1 b b 1 c c 1 Đặt 2 2 2 , , yz zx xy a b c x y z với x, y, z 0, ta được 4 4 4 4 2 2 2 4 2 2 2 4 2 2 2 1. x y z x x yz y z y y zx z x z z xy x y Tới đây, sử dụng bất đẳng thức Cauchy-Schwarz kết hợp với kết quả cơ bản x2y2 y2z2 z2x2 xyz(x y z), ta có 2 2 2 2 4 4 4 2 2 2 2 2 2 2 2 2 2 4 4 4 2 2 2 2 2 2 ( ) ( ) ( ) ( ) ( ) 1. ( ) 2( ) x y z VT x y z xyz x y z y z z x x y x y z x y z y z z x x y Bài toán được chứng minh xong. Đẳng thức xảy ra khi và chỉ khi a b c. Cách 2. Áp dụng bất đẳng thức Holder, ta có (a4 b2c2 b2c2)(a b c)(a c b) (a2 2bc)3, suy ra 4 2 2 2 2 2 2 . 2 a b c a bc a bc a b c Từ đó, ta được 2 2 2 2 2 2 . a bc b ca c ab VT a b c a b c a b c a b c Cách 3. Trước hết, ta sẽ chứng minh 2 2 2 . 2 2 2 a b c a b c a bc b ca c ab ab bc ca
  • 26. 26 Thật vậy, do 2 2 ( )( ) 2 ( 2 )( ) a a a a b a c ab bc ca a bc a bc ab bc ca nên bất đẳng thức này tương đương với 2 2 2 ( )( ) ( )( ) ( )( ) 0. 2 2 2 a a b a c b b c b a c c a c b a bc b ca c ab Không mất tính tổng quát, giả sử . a b c Khi đó 2 ( )( ) 0 2 c c a c b c ab nên ta chỉ cần chứng minh 2 2 ( )( ) ( )( ) 0. 2 2 a a b a c b b c b a a bc b ca Do ( ) 0 a a c b c b và a b 0 nên 2 2 2 2 2 2 2 2 2 2 ( )( ) ( )( ) ( )( ) ( )( ) 2 2 ( 2 ) 2 2 ( ) ( )( ) 0. ( 2 )( 2 ) a a b a c b b c b a a a b b c b b c b a a bc b ca b a bc b ca c a b b c a ab b b a bc b ca Quay trở lại bài toán chính. Bình phương hai vế, bất đẳng thức cần chứng minh tương đương với 4 2 2 4 2 2 4 2 2 2 2 2 2 2 ( 2 )( 2 ) 2 2 . 2 ( 2 )( 2 ) a b c a b c b c a a ab a bc a bc b ca Áp dụng kết quả vừa chứng minh ở trên, ta có 4 2 2 2 2 2 2 2 2 2 2 2 2 2 2 2 2 3 2 2 3 2 3 ( ) ( ) . 2( ) a b c a bc a bc a bc a bc a a ab abc a bc abc a b c a ab ab bc ca a b c a a b c ab bc ca Vậy ta chỉ cần chứng minh
  • 27. 27 4 2 2 4 2 2 2 2 ( 2 )( 2 ) . ( 2 )( 2 ) a b c b c a ab bc ca a bc b ca Sử dụng bất đẳng thức AM-GM, ta có 4 2 2 4 2 2 4 2 2 3 2 2 2 ( 2 )( 2 )( 2 ) 3 . ( 2 )( 2 )( 2 ) a b c b c a c a b VT a bc b ca c ab Mặt khác, ta lại có 2 2 4 2 2 2 2 2 2 ( 2 ) 2 ( ) ( ) ( ) 3 a bc a b c a bc bc nên VT 3 (a2 2bc)(b2 2ca)(c2 2ab). Cuối cùng ta chỉ cần chứng minh (a2 2bc)(b2 2ca)(c2 2ab) (ab bc ca)3. Khai triển và rút gọn, ta được a3b3 3a2b2c2 4abc a3 3abc ab(a b). Bất đẳng thức cuối này đúng vì ta có a3b3 3a2b2c2 abc ab(a b) (bất đẳng thức Schur bậc 3 áp dụng cho ab, bc, ca ) và 4abc a3 2abc ab(a b) (theo AM-GM). Bài toán được chứng minh xong. Bài 12. Chứng minh rằng với các số không âm ,,a b c thỏa mãn không có hai số nào đồng thời bằng 0, ta luôn có 2 2 2 2 2 2 ( ) ( ) ( ) 2. a b c b c a c a b b bc c c ca a a ab b Chứng minh. Cách 1. Áp dụng bất đẳng thức AM-GM, ta có 2 2 2 2 2 2 ( ) 4 ( )( ) 4 ( ) , ( ) ( )( ) a b c a b c ab bc ca a ab bc ca b bc c b bc c ab bc ca b c a b c nên để chứng minh bất đẳng thức đã cho, ta chỉ cần có 2 2 2 4 ( ) 4 ( ) 4 ( ) 2, ( )( ) ( )( ) ( )( ) a ab bc ca b ab bc ca c ab bc ca b c a b c c a a b c a b a b c tức là
  • 28. 28 ( )2 . 2( ) a b c a b c b c c a a b ab bc ca Nhưng bất đẳng thức này hiển nhiên đúng theo Cauchy-Schwarz nên phép chứng minh của ta vì thế được hoàn tất. Dễ thấy đẳng thức xảy ra khi và chỉ khi a b c hoặc ,0 a b c cùng hoán vị. Cách 2. Sử dụng bất đẳng thức Cauchy-Schwarz, ta có 2 2 2 2 2 ( ) ( ) ( ) . a b c a b bc c a b c b bc c b c Do đó, bất đẳng thức cần chứng minh được đưa về 2 2 2 ( ) ( ) 2 . a b bc c a b c b c Mặt khác, ta lại có Cauchy - Schwarz ( 2 2) 1 2 2 ( ) 2 9 2 ( ) ( ) 9 4( ) , a b bc c a b c abc b c b c abc a b c a b c abc ab bc ca a b c nên ta chỉ cần chứng minh được 2 9 ( ) 4( ) , abc a b c ab bc ca a b c tức là 2 2 2 9 2( ). abc a b c ab bc ca a b c Đây chính là bất đẳng thức Schur bậc 3 ở dạng phân thức nên nó hiển nhiên đúng. Và vì thế, bất đẳng thức của ta được chứng minh xong. Cách 3. Nhân cả hai vế của bất đẳng thức đã cho với ab bc ca và thực hiên phép tách 2(ab bc ca) a(b c) b(c a) c(a b), ta thấy 2 2 2 2 2 2 2 2 ( )( ) ( )[ ( ) ( )] ( ) ( )( ) ( )( ) . a b c ab bc ca a b c b a b c a c a b c b bc c b bc c ab a b b c ca c a b c b bc c b bc c Do đó
  • 29. 29 2 2 2 2 2 2 2 2 2 2 2 2 2 2 2 ( )( ) 2( ) ( )( ) ( )( ) ( )( ) ( )( ) ( ) ( ) , ( )( ) a b c ab bc ca ab bc ca b bc c ab a b b c ca c a b c b bc c b bc c ab a b b c ab a b a c b bc c a ac c ab a b ab bc ca a ac c b bc c và ta tìm được 2 2 2 2 2 2 2 ( ) ( ) 2 . ( )( ) a b c ab a b b bc c a ac c b bc c Do ,,a b c là các số không âm nên hiển nhiên vế phải không thể nào bé hơn 0, từ đó, ta dễ dàng suy ra được bất đẳng thức cần chứng minh. Cách 4. Bài toán sẽ được chứng minh xong nếu ta có 2 2 2 2 1 1 2( ) . ( ) ( ) ( ) a b a ac c b bc c ab a b bc b c ca c a (1) Thật vậy, giả sử bất đẳng thức trên đúng, khi đó 2 2 2 2 2 2 2 2 2 2 2 2 2 2 ( ) 1 1 2( ) 2. ( ) ( ) ( ) a b c ab ca b bc c b bc c b bc c ab ab b bc c c ca a ab a ac c b bc c a b ab ab a b bc b c ca c a Bây giờ ta sẽ đi chứng minh (1). Áp dụng bất đẳng thức AM-GM, ta có 2 2 2 2 2 2 2 2 1 1 2 . a ac c b bc c (a ac c )(b bc c ) Suy ra ta chỉ cần chứng minh 2 2 2 2 ( ) ( ) ( ) ( )( ) . ab a b bc b c ca c a a ac c b bc c a b Theo bất đẳng thức Cauchy-Schwarz
  • 30. 30 2 2 2 2 2 2 2 2 2 ( )( ) ( )( ) ( )( ) ( ) , ac bc a ac c b bc c ac a ac c bc b bc c a c b c ab a b c c từ đó suy ra 2 2 2 2 2 2 2 2 2 ( )( ) 2 ( ) ( ) ( ) . a ac c b bc c ab a b ab c c ab ab a b c c a b a b ab c c a b ab a b bc b c ca c a a b Bài toán được chứng minh xong. Bài 13. Cho ,,x y z là các số dương thỏa mãn 2 4 7 2 . x y z xyz Tìm giá trị nhỏ nhất của biểu thức P x y z. Lời giải. Cách 1. Áp dụng bất đẳng thức AM-GM, ta có 1 6 10 14 30 1 1 7 5 3 15 2 2 2 4 7 6 10 14 30 3 5 2 3 5 2 2 30 3 5 2 x y z x y z x y z x y z và 1 6 5 4 15 2 1 4 5 3 15 2 2 2( ) 6 5 4 15 3 5 2 3 5 2 2 15 . 3 5 2 x y z x y z x y z x y z Từ đây ta có
  • 31. 31 2 2 2 2 1 1 7 2 1 4 5 3 15 5 3 15 2 (2 4 7 ) 2( ) ( ) 8 2 2 30 15 2 3 5 2 3 5 2 30 15 125 3 5 2 , 8 8 4 x y z x y z P x y z xyz x y z x y z x y z xyz xyz suy ra 15 . 2 P Mặt khác, dễ thấy bộ 5 ( , , ) 3, , 2 2 x y z thỏa mãn điều kiện của đề bài và 15 . 2 P Do đó 15 min . 2 P Cách 2. Từ giả thiết suy ra 2 4 2 7 x y z xy và 2xy 7 0. Do đó 2 4 . 2 7 x y P x y xy Sử dụng bất đẳng thức AM-GM, ta có 2 2 2 4 11 7 2 4 2 2 7 2 2 2 7 11 2 7 2( 7) 11 2 7 . 2 2 (2 7) 2 x y x y P x y x y xy x x xy x xy x x x x x x x xy x x Đẳng thức xảy ra khi 2 7 2( 2 7) , 2 2 7 xy x xy hay 7 2 2 7 . 2 x y x Mặt khác, theo bất đẳng thức Cauchy-Schwarz thì 2 2 ( 7)(9 7) 3 7 7 4 4 x x x (Đẳng thức xảy ra khi x 3 ). Suy ra 11 3 7 9 3 3 15 6 2 2 2 2 2 x P x x x x x (Đẳng thức xảy ra khi x 3 ). Vậy 15 min 2 P đạt được khi
  • 32. 32 2 2 4 2 7 3 7 2 7 5 . 2 2 3 2 x y z xy x x y y x x z Cách 3. Ta đã biết tính chất sau: Nếu ()fx khả vi bậc hai và lồi trên ( , ) ab thì với mọi , , ( ), ax b y ta có f (x) f (y) f (y)(x y). Dưới đây, ta sẽ sử dụng tính chất này để giải bài toán đã cho. Đặt 7 7 , 2 x a y b và 2 7 7 z c thì ta có a b c abc và 7 (14 7 4 ). 14 P a b c Do a, b, c 0 và a b c abc nên tồn tại , , 0, 2 A B C thỏa mãn A B C và a tanA, b tanB, c tanC, suy ra 7 (14 tan 7 tan 4 tan ). 14 P A B C Do f (x) tanx khả vi bậc hai và lồi trên 0, 2 nên theo tính chất trên, với mọi 0, , 2 x, y ta có tanx tany (tany) (x y) tany (tan2 y 1)(x y). Thay (x, y) lần lượt bởi 3 5 , arctan , , arctan , , arctan 7 , 7 7 A B C ta được
  • 33. 33 2 2 2 3 3 3 3 16 3 tan 1 arctan arctan , 7 7 7 7 7 7 5 5 5 5 32 5 tan 1 arctan arctan , 7 7 7 7 7 7 tan 7 7 1 a A A A B B B C C rctan 7 7 8 C arctan 7 . Từ đó suy ra (chú ý rằng 3 5 arctan arctan arctan 7 7 7 ) 7 3 5 15 7 32 arctan arctan arctan 7 14 7 7 7 15 15 7 32( ) . 14 2 P A B C A B C Đẳng thức xảy ra khi và chỉ khi 5 3, , 2. 2 x y z Bài 14. Cho ,,a b c là các số thực không âm thỏa mãn không có hai số nào đồng thời bằng 0. Chứng minh rằng 2 2 2 1 1 1 2 2 . a bc b ca c ab ab bc ca Chứng minh. Cách 1. Không mất tính tổng quát, giả sử a max{a, b, c}. Khi đó, sử dụng bất đẳng thức Holder, ta có VT2 (b c)3(a2 bc) a3(b2 ca) a3(c2 ab) (2a b c)3. Bài toán được đưa về chứng minh (2a b c)3(ab bc ca) 8(b c)3(a2 bc) 8a3(b2 c2 ab ac). Đến đây, sử dụng tính thuần nhất, ta chuẩn hóa cho b c 1. Bất đẳng thức trên được lại thành (2a 1)3(a bc) 8(a2 bc) 8a3(1 2bc a), hay bc[(2a 1)3 16a3 8] a(2a 1)3 8a2 8a3(a 1) 0.
  • 34. 34 Ta có (2a 1)3 (b c 1)3 8 và a(2a 1)3 8a2 8a3(a 1) a(a 1)2 3a3 0 nên bất đẳng thức cuối hiển nhiên đúng. Phép chứng minh được hoàn tất. Cách 2. Không mất tính tổng quát, ta có thể giả sử a max{a, b, c}. Khi đó có hai trường hợp xảy ra. + Trường hợp 1. . b c a Dễ thấy a2 b2 c2 2(ab bc ca). Do đó, áp dụng bất đẳng thức Cauchy-Schwarz, ta được 2 2 2 2 2 2 2 1 9 3 3 3 . a bc a bc b ca c ab a b c ab bc ca ab bc ca + Trường hợp 2. a b c. Khi đó, cũng sử dụng bất đẳng thức Cauchy -Schwarz, ta có 2 2 2 2 2 2 1 1 4 2 2 . b ca c ab b ca c ab b c a(b c) Do đó, ta chỉ cần chứng minh được 2 2 2 2( ) 2 2 2. ( ) ab bc ca ab bc ca a bc b c a b c Ta có 2 2 2 ( ) ( ) 1 1 ab bc ca a b c a a b c a b c a bc a bc a a và 2 2 2 ( ) , ( ) ( ) ( ) ab bc ca a b c a b c a b c b c a b c a b c nên bất đẳng thức trên được suy ra từ 2 2 2 2. b c a a a b c Từ đây, bằng cách đặt 1, b c t a ta có thể đưa bài toán về chứng minh một bất đẳng thức một biến và việc chứng minh không mấy khó khăn (chỉ cần AM-GM). Xin được dành lại cho bạn đọc.
  • 35. 35 Cách 3. Nếu có một số bằng 0 trong ,,a b c thì bất đẳng thức hiển nhiên đúng. Vì vậy ta chỉ cần xét 0. abc Khi đó, đặt 11 , xy ab và 1 z , c ta có thể viết lại bất đẳng thức cần chứng minh dưới dạng 2 2 2 ( ) ( ) ( ) 2 2. x x y z y x y z z x y z x yz y zx z xy Theo bất đẳng thức AM-GM thì 2 2 ( ) 2 2 ( ) . ( ) 2( ) x x y z x x y z x yz x x y z x yz Do đó, ta chỉ cần chứng minh được 2 ( ) 1. 3 2 x x y z x xy xz yz Đến đây, thực hiện khai triển và rút gọn, ta thu được bất đẳng thức đúng 2(x y)2(y z)2(z x)2 12xyz(x y)(y z)(z x) 2x2y2z2 0. Bài toán được chứng minh xong. Bài 15. Cho ,,a b c là các số thực dương. Chứng minh rằng 3 ( 1) ( 1) ( 1) ( 1)( 1)( 1). 2 a b b c c a a b c Chứng minh. Cách 1. Chia hai vế của bất đẳng thức cho (a 1)(b 1)(c 1), ta được 1 1 1 3 . 1 1 1 1 1 1 2 a b c a c b a c b Sử dụng bất đẳng thức AM-GM, ta có 1 1 1 1 1 1 , , 1 1 2 1 1 1 1 2 1 1 1 1 1 . 1 1 2 1 1 a a b b a c a c b a b a c c c b c b Cộng ba bất đẳng thức này lại theo vế, ta được 1 1 1 1 1 3 . 1 1 2 1 1 1 2 a a b c a c a b c
  • 36. 36 Bài toán được chứng minh xong. Đẳng thức xảy ra khi và chỉ khi 1. a b c Cách 2. Áp dụng bất đẳng thức Cauchy-Schwarz, ta có VT2 3 a(b 1) b(c 1) c(a 1) . Như vậy, bất đẳng thức của ta sẽ được chứng minh nếu ta có 4 a(b 1) b(c 1) c(a 1) 3(a 1)(b 1)(c 1), tức là a b c ab bc ca 3abc 3. Xét trường hợp ngược lại a b c ab bc ca 3abc 3. Khi đó, lấy bình phương hai vế, ta viết lại bất đẳng thức cần chứng minh dưới dạng 9 2 ( 1)( 1) ( 1)( 1)( 1). 4 a ab ab b c a b c Theo bất đẳng thức AM-GM thì 2 ( 1)( 1) ( 1) ( 1) 3 3. ab b c ab c b a b c ab bc ca abc Do đó, ta chỉ cần chứng minh được 9 2( ) 3 3 ( 1)( 1)( 1), 4 a b c ab bc ca abc a b c tương đương a b c ab bc ca 3abc 3 (đúng). Bài toán được chứng minh xong. Cách 3. Sử dụng bất đẳng thức Cauchy-Schwarz, ta được ( 1) ( 1) ( 1) ( 1) ( 1) ( 1)( 2 1). a b b c a b b c a bc b Từ đó bài toán được quy về chứng minh 3 ( 1)( 2 1) ( 1) ( 1)( 1)( 1). 2 a bc b c a a b c Đơn giản cả hai vế cho a 1, ta được 3 2 1 ( 1)( 1). 2 bc b c b c Đến đây, sử dụng tiếp bất đẳng thức Cauchy-Schwarz và kết hợp với bất đẳng thức AM-GM, ta có
  • 37. 37 2 ( 2)(2 1) ( 2 1) ( 1) 1 ( 1) 1 1 ( 2) (2 1) 3 ( 1) ( 1)( 1). 4( 1) 2 c c c VT bc b c b c c c c b b c c Bài 16. Cho các số thực không âm ,,x y z thỏa mãn không có hai số nào đồng thời bằng 0. Chứng minh bất đẳng thức sau 2 1 . ( )( )( ) x y z xyz y z z x x y x y y z z x Chứng minh. Cách 1. Không mất tính tổng quát, giả sử z min{x, y, z}. Khi đó, ta sẽ chứng minh 2 . 2 x y x y y z z x x y z (1) Thật vậy, theo bất đẳng thức Holder, ta có 2 2( ) 2( ) ( )2. x y x y z y z x x y y z z x Mặt khác, do z min{x, y, z} nên 2 2 2 2 2 2 ( ) ( ) ( 2 ) ( ) ( ) ( 2 ) ( ) 4 ( ) ( 2 ) . 4 x y z y z x xy x y z z x y x y x y z z x y x y x y z Kết hợp với trên, ta có ngay 3 2 2 ( ) 2 . ( ) ( ) 2 x y x y x y y z z x x y z y z x x y z Bây giờ, sử dụng (1) kết hợp với đánh giá 2 2 2 1 1 ( ) , ( )( ) ( ) 4 ( ) ( 2 ) 1 1 ( ) xy x y x z y z z x y z z x y z x y z xy x y
  • 38. 38 ta đưa được bài toán về chứng minh 2 ( ) 2 2 1 . 2 ( 2 ) x y z z x y x y z x y x y z Đặt 2 1, z t xy ta phải chứng minh 2 2 2 1 1 2 2( 1) t t t t hay 2 4 2 2 2 4 . 1 2 1 ( 1) t t t t t t Nếu 0 t thì bất đẳng thức trở thành đẳng thức. Xét trường hợp 0, t bất đẳng thức này có thể được thu gọn thành 2 2 2 (2 3) 2 2 1 ( 1) t t t t t t hay là 2 1 4 2(2 3) . 2 2 ( 1) ( 1) t t t t Theo bất đẳng thức AM-GM thì 1 2 . 2t(t 1) 3t 1 Lại có 2 2 2 1 8 2(2 3) (3 5)( 1) 0, 2 3 1 ( 1) (3 1)( 1) t t t t t t t nên bất đẳng thức trên hiển nhiên đúng. Bài toán được chứng minh xong. Đẳng thức xảy ra khi và chỉ khi x y z hoặc x y, z 0 cùng các hoán vị tương ứng. Cách 2. Đặt a x, b y, c z, ta phải chứng minh 2 2 2 2 2 2 2 2 2 2 2 2 2 2 2 2 2 2 2 1 . ( )( )( ) a b c a b c b c c a a b a b b c c a Áp dụng bất đẳng thức Holder, ta có VT2 a(b2 c2) b(c2 a2) c(a2 b2) (a b c)3. Do đó, ta chỉ cần chứng minh được 3 2 2 2 2 2 2 2 2 2 ( ) 4 4 . ( ) ( ) ( ) ( )( )( ) a b c a b c ab a b bc b c ca c a a b b c c a Do (a b c)3 4 ab(a b) a(a b)(a c) 3abc 3abc và
  • 39. 39 2 2 2 2 2 2 2 2 2 2 2 2 2 2 2 4 4 9 ( ) 8 2 9 9 , 2 a b c a b c abc abc a a b a a b a a a b abc a a nên ta chỉ cần chứng minh 2 2 2 3 9 , ( ) ( ) ( ) 2( )( ) abc abc ab a b bc b c ca c a a b c a b c tương đương abc (a b)(a b)2 (b c)(b c)2 (c a)(c a)2 0 (đúng). Bài toán được chứng minh xong. Cách 3. Bình phương hai vế, bất đẳng thức đã cho có thế được viết lại thành 4 2 4 . ( )( ) ( )( )( ) x xy xyz y z x z y z x y y z z x Sử dụng các bất đẳng thức Cauchy-Schwarz và AM-GM, ta được ( )( ) ( )( ) ( )( ) ( )( ) ( ) ( ) ( )( )( ) ( ) 2 ( ) 6 ( )( )( ) ( )( )( ) 4 1 . ( )( )( ) xy xy x z y z xy xy z x z y z x z y z x z y z xy x y z xy x y x z y z x y xy x y xyz xy x y xyz x z y z x y x y y z z x xyz x y y z z x Do đó, ta chỉ cần chứng minh được 4 2. ( )( )( ) x y z xyz y z z x x y x y y z z x Đây chính là bất đẳng thức Schur bậc 3 ở dạng phân thức, vì thế nó hiển nhiên đúng. Và như vậy, ta có điều phải chứng minh. Cách 4. Sử dụng bất đẳng thức Holder, ta có
  • 40. 40 2 3 2 3 3 ( )( ) ( )( )( ) ( ) . ( )( )( ) x x x y z x y x z y z x y x z x y z x y y z z x Từ đó với chú ý ( )( ) 1 , ( )( )( ) ( )( )( ) xyz x y z xy yz zx x y y z z x x y y z z x ta đưa được bài toán về chứng minh ( )2 2 4 . ( )( ) x y z x xy yz zx x y x z Không mất tính tổng quát, giả sử z min{x, y, z}. Khi đó ta có 2 2 2 2 2 2 2 2 2 2 2 ( ) 3 3 4 2 2 2 . ( )( ) x y z x y z xy yz zx xy yz zx xy yz zx x y z xy yz zx xy yz zx z x y z xy yz zx x z y z Vậy chỉ cần chứng minh (x y)(x2 y2 4z 2 2xy 2yz 2zx) 4 x2(y z). Sau khi khai triển và rút gọn, ta thu được bất đẳng thức hiển nhiên đúng (x y)2(x y 2z) 0. Bài 17. Cho ,,a b c là các số thực dương. Chứng minh rằng a3 b3 c3 3abc ab 2(a2 b2) bc 2(b2 c2) ca 2(c2 a2). Chứng minh. Cách 1. Không mất tính tổng quát, giả sử c min{a, b, c}. Áp dụng bất đẳng thức AM-GM, ta có 2 2 2 2 2 2 2 2 2 2 2 2 2 2 2 2( ) 2( ) 2 2 3 3 2 , 2 b b c a c a b b c a c a a b c từ đó suy ra
  • 41. 41 2 2 2 2 3 2 2 3 2( ) 2( ) ( ). 2 bc b c ca c a c c a b Bài toán được quy về chứng minh 3 3 3 2 2 3 2 2 3 3 2( ) ( ), 2 a b c abc ab a b c c a b hay 2a3 2b3 2ab 2(a2 b2) 3c(a b)2. Ta có 3 3 2 2 2 2 3 3 2 2 2 2 2 2 2 2 2 2 2 2 2( ) 2( ) ( )( ) ( ) 2( ) 4 2 ( )( ) ( ) 2( ) 4 . 2 a b ab a b a b a b a b a b a b a b ab a b a b a b a b a b ab Do 2 2 ( )( )2 2 ( )2, 2 a b a b a b c a b c và (a b) 2(a2 b2) 4ab (a b)2 4ab (a b)2 0, nên hiển nhiên 2 2 ( )( )2 ( ) 2( 2 2) 4 3 ( )2. 2 a b a b a b a b a b ab c a b Bài toán được chứng minh xong. Đẳng thức xảy ra khi và chỉ khi a b c. Cách 2. Không mất tính tổng quát, giả sử a b c. Sử dụng bất đẳng thức AM-GM, ta có 2 2 2 2 2 2 2 2 2 2 2 2 2 2 2 2( ) ( ) 2 , 2 2( ) ( ) 2 , ( ) 2 2( ) 2 . ab a b a a b ab bc b c c b c cb ca c a ca c a abc b Cộng ba bất đẳng thức trên lại theo vế, ta thu được 2 2 2 2 3 3 2 ( ) 2 2( ) 3 ( ) 2 . ca c a ab a b a c b a c abc b Bài toán quy về chứng minh 2 2 3 3 3 3 3 2 ( ) 2 2 2 6 3 ( ) 2 , ca c a a b c abc a c b a c abc b
  • 42. 42 hay tương đương với 2 2 3 3 3 2 ( ) 2 4 3 ( ) . ca c a a c b abc b a c b Bằng một vài tính toán đơn giản, ta có thể viết lại nó trở thành 2 2 ( )( ) 2 0, a c a b b c b a c b đúng vì (a b)(b c) 0, 2 2 2 2 2 0. a c a b a c a b b b Cách 3. Áp dụng bất đẳng thức AM-GM, ta có 2 2 2 2 2 2 2 2 2 2 2 2 2 2 2 2 2( ) ( ) 2 2( ) 2 ( ) 2 ( ) ( ) , 2( ) ( ) 2 2( ) 2 ( ) 2 ( ) ( ) , 2( ) 2 2( ) 2 ( ) a b ab a b ab a b ab a b ab a b a b a b ab a b b a b b c bc b c bc b c bc b c bc b c b c b c bc b c c b c c a c ca c a ca c a ca c a c a 2 2 ( ) 2 ( ) ( ) . a c a c a ca c a c c a Cộng ba bất đẳng thức trên lại theo vế, ta thu được 2 2 2 2 2 2 2 2 3 3 2 2( ) 2 ( ) ( ) ( ) ( ) 3 3 3 2 . ab a b ab a b b a b c b c c a c a b a c b c b c Do đó, ta chỉ cần chứng minh 2a3 2b3 2c3 6abc 3a2b 3a2c 3b2c b3 2c3. Sau khi thu gọn, ta thu được bất đẳng thức hiển nhiên đúng (a b)2(2a b 3c) 0. Cách 4. Ta viết lại bất đẳng thức như sau 3 3 3 2 2 2 2 2 2 3 ( ) 2( ) , 1 ( ) ( ) ( ) . 2 2( ) a b c abc bc b c bc b c b c bc b c b c b c a b c b c Do 2(b2 c2) b c 2(b c) nên ta chỉ cần chứng minh
  • 43. 43 ( )2 ( )2 ( )2 0, a b c S b c S c a S a b trong đó 2 , a bc b S b c a c a b c b c , b S c S tương tự. Không mất tính tổng quát, giả sử . a b c Ta có 2 2 0, 0. b c c a S a b S b c c a a b Từ đó suy ra (chú ý rằng (a c)2 (b c)2 ) ( )2 ( )2 ( )2 ( )2( ). a a b a b S b c S b c S c a b c S S Lại có 2 2 2 1 1 0, a b bc ca b a S S c c b c c a b c c a c b c c a nên kết hợp với trên, ta suy ra điều phải chứng minh. Bài 18. Cho ,,ab c là các số dương thỏa mãn 1. abc Chứng minh rằng 2 2 2 1 1 1 2 1. (1 a) (1 b) (1 c) (1 a)(1 b)(1 c) Chứng minh. Cách 1. Đặt 2 2 , yz zx a b x y và 2 xy c z với x, y, z 0. Khi đó, bất đẳng thức cần chứng minh có thể viết lại thành 4 4 4 2 2 2 2 2 2 2 2 2 2 2 2 2 1. ( ) ( ) ( ) ( )( )( ) x y z x y z x yz y zx z xy x yz y zx z xy Áp dụng bất đẳng thức Cauchy-Schwarz, ta thu được 4 4 4 2 2 2 2 2 2 2 2 2 2 2 2 2 2 2 2 2 2 2 2 2 2 2 ( ) ( ) ( )( ) ( )( )( ) 2 1 . ( )( )( ) x x x y z x yz x y x z x y y z z x x y z x y y z z x Do đó, bài toán quy về chứng minh (x2 y2)(y2 z2)(z2 x2) (x2 yz)(y2 zx)(z2 xy).
  • 44. 44 Bất đẳng thức cuối này đúng vì ta có x2 yz (x2 y2)(x2 z2) theo bất đẳng thức Cauchy-Schwarz. Bài toán được chứng minh xong. Đẳng thức xảy ra khi và chỉ khi a b c 1. Cách 2. The nguyên lý Dirichlet, ta biết rằng trong ba số ,,a b c luôn có hai số hoặc cùng 1 hoặc cùng 1. Không mất tính tổng quát, ta giả sử hai số đó là a và ,b thế thì (1 )(1 ) 0. a b Sử dụng bất đẳng thức Cauchy- Schwarz, ta được 2 2 1 1 1 1 1 . (1 ) (1 ) 1 (1 ) 1 (1 ) 1 a b a b ab ab ab b a (1) Mặt khác ta lại có (1 a)(1 b) 2(1 ab) (1 a)(1 b) 2(1 ab). (2) Từ (1) và (2) suy ra ta chỉ cần chứng minh 2 1 1 1 1. 1 ab (1 c) (1 ab)(1 c) Thay 1 ab c ta thu được một hằng đẳng thức đúng. Cách 3. Đặt 1 1 1 1 , 1 2 1 2 x y a b và 1 1 . 1 2 z c Ta có 1 1 1 1 , , 1, , , . 1 1 1 x y z x y z a b c x y z Do abc 1 nên (1 x)(1 y)(1 z) (1 x)(1 y)(1 z), suy ra x y z xyz 0. Như vậy, ta phải chứng minh (1 x)2 (1 y)2 (1 z)2 (1 x)(1 y)(1 z) 4 với 1 x, y, z 1 và x y z xyz 0. Bất đẳng thức này tương đương với x2 y2 z2 2(x y z) xy yz zx 0. Do ( )2 2 2 2 2 x y z x y z xy yz zx nên bất đẳng thức trên có thể viết lại thành x2 y2 z2 4(x y z) (x y z)2 0.
  • 45. 45 Thay x y z xyz vào, ta được x2 y2 z2 x2y2z2 4xyz. Bất đẳng thức cuối này đúng theo AM-GM x2 y2 z2 x2y2z2 44 x4y4z 4 4 xyz 4xyz. Bài 19. Cho , , , , , a b c x y z là các số thực dương thỏa mãn ax by cz xyz. Chứng minh rằng x y z a b b c c a. Chứng minh. Cách 1. Giả thiết đã cho có thể được viết lại dưới dạng 1, a b c yz zx xy do đó 1, 1, 1. a b b c a c yz zx zx xy yz xy Từ đây ta suy ra , , . b c a c a b x y z z y z x y x Sử dụng kết quả này kết hợp với bất đẳng thức am, ta được 2( ) 2 2 2 . b c a c a b x y z x y z z y z x y x a b b c c a z x y z x y a b b c c a Từ đây ta có điều phải chứng minh. Cách 2. Bất đẳng thức đã cho có thể dược viết dưới dạng thuần nhất như sau ( )2( ) 2 . x y z ax by cz a b b c c a xyz Không mất tính tổng quát, giả sử x max{x, y, z}. Bây giờ, sử dụng lần lượt các bất đẳng thức AM-GM và Cauchy-Schwarz, ta có
  • 46. 46 2 4 ( )( ) 2( )[ ( ) 2 2 ] 2 2 2( ) 2 2 2 . x y z ax by cz y z a y z by cz VT xyz yz a c a b y z a c a b y z Bài toán được quy về chứng minh 2 2 2 a 2c a 2b a b b c c a . Bất đẳng thức này đúng, vì theo bất đẳng thức Cauchy-Schwarz ta có 2( 2 ) 2( 2 ). a b b c c a a b b a c c a b a c Cách 3. Giả thiết đã cho được có thể được dưới dạng 1. a b c yz zx xy Đặt a yzu, b zxv và c xyw. Ta quy được bài toán về chứng minh z(yu xv) x(zv yw) y(zu xw) x y z với u v w 1. Áp dụng bất đẳng thức Cauchy-Schwarz VT2 (x y z) (yu xv) (zv yw) (zu xw) , do vậy ta chỉ cần chứng minh x(v w) y(w u) z(u v) (x y z)(u v w). Bất đẳng thức này tương đương với xu yv zw 0, hiển nhiên đúng. Bài 20. (Chọn đội tuyển Moldova 2006) Cho ,,a b c là độ dài ba cạnh của một tam giác. Chứng minh rằng 2 1 2 1 2 1 0. b c a a b c c a b Chứng minh. Cách 1. Bất đẳng thức cần chứng minh tương đương với 2 2 2 2 2 2. a b b c c a a b c c a b Sử dụng bất đẳng thức Cauchy-Schwarz, ta có
  • 47. 47 2 2 2 2 2 2 2 2 2 2 2 2 2 2 ( ) ( ) . ( ) a b b c c a a b b c c a a b b c c a c a b a bc b ca c ab abc a b c Do đó, ta chỉ cần chứng minh được (a2b b2c c2a)2 abc(a b c)(a2 b2 c2). Không mất tính tổng quát, giả sử b là số hạng nằm giữa a và c. Khi đó, áp dụng bất đẳng thức AM-GM, ta được 2 2 2 2 2 2 2 [ ( ) ( )] ( )( ) . 4 ac a b c b a b c abc a b c a b c Ta cần chứng minh 2(a2b b2c c2a) ac(a b c) b(a2 b2 c2). Thu gọn và phân tích nhân tử, ta được (a b)(b c)(a b c) , . hiển nhiên đúng do ( )( ) 0 a b b c (theo giả thiết của b ) và a b c (theo giả thiết của đề bài). Vậy ta có điều phải chứng minh. Đẳng thức xảy ra khi và chỉ khi tam giác đã cho là tam giác đều. Cách 2. Nhân hai vế của bất đẳng thức với , abc ta viết được nó dưới dạng a3b2 b3c2 c3a2 abc(a2 b2 c2). Ta có 3 2 3 3 2 2 2 3 3 3 2 2 3 2 2 2 2 ( 2 ) ( ) ( ) ( ) ( ) ( ). a b a bc a b c bc a b c a b c a b c a b c a b c Mà ,,ab c là ba cạnh của một tam giác nên hiển nhiên , , a b c b c a c a b 0, từ đó suy ra a2(b c)2(a b c) 0, tức là a3b2 b3c2 c3a2 abc(a2 b2 c2). Cách 3. Không mất tính tổng quát, giả sử c max{a, b, c}. Khi đó, sử dụng bất đẳng thức AM-GM, ta có 2 2 1 2 2 2. c b c b ca ca b bc ca b a a Và như thế, ta chỉ cần chứng minh ( ) 2 2 2 2 1 0. b c a f a a bc ca b c c b
  • 48. 48 Ta thấy f (a) là một tam thức bậc hai và hệ số cao nhất của nó không dương. Vì vậy ()fa sẽ đạt cực tiểu tại biên. Mà c b a c nên f (a) min{f (c b), f (c)}. Tính toán trực tiếp, ta dễ thấy ( )3 ( )4 ( ) 0, ( ) 0. c b b c f c f c b b bc Từ đó suy ra ( ) 0. fa Bài toán được chứng minh xong. Cách 4. Đặt 11 , ab xy và 1 c . z Bất đẳng thức cần trở thành 2 2 2 1 1 1 1 1 1 0, z x y x y y z z x tương đương E(x, y, z) yz2(z y) zx2(x z) xy2(y x) 0. Không mất tính tổng quát, giả sử a min{x, y, z}, khi đó từ phép đặt ẩn suy ra x max{x, y, z}. Ta sẽ chứng minh E(x, y, z) E(y, y, z). Ta có 3 3 2 2 2 3 2 2 2 2 ( , , ) ( , , ) ( ) ( ) ( ) ( ) ( )( )( ). E x y z E y y z z x y z x y y x y y x y x y x z xz yz y Do (x y)(x z) 0 và 2 2 2 2 ( ) ( ) (2 ) 0 b c b a a b c xz yz y y z y b c b c nên hiển nhiên E(x, y, z) E(y, y, z). Mặt khác, ta lại có E(y, y, z) yz(y z)2 0, nên bài toán được chứng minh xong. Bài 21. Chứng minh rằng với mọi số dương a, b, c, ta luôn có 1 1 1 1 1 1 1 1 1 1 1 1 3. a b b c b c c a c a a b
  • 49. 49 Chứng minh. Cách 1. Đặt 1 1 1 x a 1, y b 1, z c 1 b c a thì bất đẳng thức của ta trở thành xy yz zx 3. Theo nguyên lý Dirichlet, ta thấy rằng tồn tại ít nhất hai số trong ba số ,,x y z sao cho hiệu của chúng với 1 cùng dấu. Giả sử hai số đó là x và ,y khi đó ta có ( 1)( 1) 0, x y suy ra 1.xyx y Vì vậy, ta chỉ cần chứng minh được (x y 1) z(x y) 3 hay 4 . 1 x y z Bất đẳng thức này hiển nhiên đúng vì 1 1 1 1 1 4 4 2 . 1 1 1 a c c a x y a b a c b c z c c a a Phép chứng minh của ta được hoàn tất. Đẳng thức xảy ra khi và chỉ khi x y z 1. Cách 2. Bằng phép khai triển trực tiếp, ta có thể dễ dàng viết lại bất đẳng thức cần chứng minh dưới dạng 1 2 2 1 3 0. b a a ab abc abc Đặt 3 k abc 0, ta thấy tồn tại các số dương x, y, z sao cho , kx a y , . ky kz b c z x Với phép đổi biến này, bất đẳng thức trên trở thành 2 2 2 1 2 2 3 0, x x y k k yz k y k x tương đương 3 2 2 2 2 2 1 f (k) x k x y 2k xy 3xyz 0. k k
  • 50. 50 Ta có 3 2 2 3 2( 1) ( ) , k f k k x y xy k phương trình ( ) 0 fk chỉ có một nghiệm dương duy nhất là 2 0 2 . xy k xy Ngoài ra, ta thấy rằng qua 0 k thì ()fk đổi dấu từ âm sang dương nên đây cũng chính là giá trị cực tiểu của ( ),fk tức là 0 ( ) ( ) f k f k với mọi 0.k Vì vậy, để chứng minh bất đẳng thức đã cho, ta chỉ cần chứng minh được 2 2 0, xy f x y tương đương 2 2 2 2 2 2 2 2 3 3 3 2 2 2 2 2 2 ( ) ( ) 3 . x y y z z x xy yz zx x y z xyz xy yz zx x y y z z x Để ý rằng 2 2 2 1 1 ( ) ( ) m n m n m n n m m n với mọi m, n 0 và x 3 3xyz xy(x y) (x y z)(x y)2 z(x z)(y z), ta có thể viết lại bất đẳng thức cần chứng minh như sau 2 2 2 2 2 2 ( )( ) ( )( ) 1 1 ( ) ( ) ( ) . x y z x y z x z y z x y y z x z x y xy Không mất tính tổng quát, giả sử z min{x, y, z}. Khi đó, ta có 2 2 2 2 2 2 2 2 2 ( ) ( ) ( ) ( )( ) 0, , ( ) 1 1 1 1 , x z y z xy x z x y z x z y z x y z x z y x y x y xy x y xy nên 2 2 2 2 2 2 1 1 ( ) ( ) ( )( ) . x y x y z VP x y x y z x y VT x y xy x y Bất đẳng thức được chứng minh xong. Cách 3. Không mất tính tổng quát, ta có thể giả sử c là số hạng nằm giữa a và b. Khi đó, ta có đánh giá sau
  • 51. 51 1 1 1 1 ( )( ) 1 1 1 1 0. a c c b b c c b c a c a ac Do đó, bất đẳng thức đã cho sẽ được chứng minh nếu bất đẳng thức mạnh hơn đúng 1 1 1 1 1 1 1 1 1 1 1 1 3, a b c a b c a b c b c a tức là 1 1 1 1 1 a 1 b 1 c 1 a b 2 3. b a c a b Theo bất đẳng thức AM-GM, 1 c 1 1, c 1 1 a b 2 0, a b suy ra 1 1 1 1 1 c 1 a b 2 a b 2. c a b a b Vì vậy, ta chỉ cần chứng minh được 1 1 1 1 a 1 b 1 a b 2 3 b a a b hay 1 1 a b 4. b a Theo bất đẳng thức AM-GM, dễ thấy bất đẳng thức cuối hiển nhiên đúng. Từ đó, ta có điều phải chứng minh. Bài 22. Cho a, b, c, x, y, z là các số thực dương thỏa mãn a b c x y z. Chứng minh rằng ax2 by2 cz2 xyz 4abc. Chứng minh. Cách 1. Đặt , 2 2 z x x y p b q c và . 2 y z r a Dễ thấy trong ba số này luôn có hai số cùng dấu với nhau. Không mất tính tổng quát, giả sử pq 0. Ta có , , . 2 2 2 z x x y y z b p c q a x y z b c p q Thay vào bất đẳng thức đã cho, ta được
  • 52. 52 2 2 2 2 2 2 2 2 2 4 2 2 4 2 2 2 2 4 ( ) 2 ( ) 2 ( ) 4 4 2 y z x z ax by cz xyz abc p q x p y x y y z x z x y q z xyz p q p q pq p q p x y q x z pqx x z q p 4 2 4 4( 2 2 ) 0. 2 x y p q pqx q b p c pqx Bài toán được chứng minh xong. Ta dễ thấy đẳng thức xảy ra khi và chỉ khi p q r 0, tức , , . 2 2 2 y z z x x y a b c Cách 2. Ta thấy nếu x2 4bc thì ax2 4abc, suy ra ax2 by2 cz2 xyz ax2 4abc. Bất đẳng thức hiển nhiên đúng. Vì vậy, không mất tính tổng quát, ta chỉ cần xét x2 4bc. Đặt u a b c x y z. Thay z u x y và a u b c vào, sau khi thu gọn, ta viết được bất đẳng thức dưới dạng 2 2 2 2 2 ( ) ( 4 ) 2 ( ) ( )( 4 ) ( ) ( ) 0. f u cu x bc c x y xy u b c x bc by c x y xy x y Ta thấy ()fu là 1 tam thức bậc hai của u với hệ số cao nhất dương. Lại có ( 2 4 )(2 )2 0, f x bc c x y nên hiển nhiên f (u) 0. Chứng minh hoàn tất. Cách 3. Dễ thấy kết quả bài toán được suy ra từ hằng đẳng thức sau 2 2 2 2 2 2 2( )( 4 ) ( ) ( ) ( ) 2 ( ), yzu zxv xyw ax by cz xyz abc xu v w yv w u zw u v uvw x y z a b c trong đó u 2ax yz, v 2by zx và w 2cz xy. Cách 4. Giả sử bất đẳng thức đã cho sai, tức ax2 by2 cz2 xyz 4abc. Đặt ax2 by2 cz2 xyz 4kabc với 0 k 1. Dễ thấy x2 4kbc, y2 4kca, z2 4kab,
  • 53. 53 suy ra tồn tại , , 0, 2 A B C sao cho cos , cos , cos . 2 2 2 x y z A B C kbc kca kab Thay vào ax2 by2 cz2 xyz 4kabc, ta được cos2 A cos2 B cos2C 2cosAcosBcosC 1, suy ra ,,A B C là ba góc của một tam giác. Đến đây, sử dụng bất đẳng thức cơ bản trong tam giác: Nếu ,,A B C là ba góc của một tam giác và ,,m n p là các số thực, thì 2np cosA 2pmcosB 2mn cosC m2 n2 p2, ta được 2 cos 2 cos 2 cos ( ) . x y z k bc A ca B ab C k a b c a b c Điều này mâu thuẫn với giả thiết của bài toán. Vậy điều giả sử ở trên là sai. Ta có điều phải chứng minh. Cách 5. Ta có (b c x) (c a y) (a b z) a b c 0. Do vậy, trong các số b c x, c a y, a b z phải có ít nhất một số dương. Không mất tính tổng quát, ta giả sử b c x 0. Bây giờ, ta thấy rằng trong trường hợp x2 4bc thì kết quả bài toán là hiển nhiên nên ở đây ta sẽ xét 4bc x2. Ta có bất đẳng thức đã cho tương đương với 2ax2 2by2 2cz 2 x (y z)2 y2 z 2 8abc, 2ax2 (2b x)y2 (2c x)z2 x(y z)2 8abc. Với mọi số thực p, q, u, v thỏa mãn p q 0, ta có 2 2 2 2 ( ) ( ) 0. pq u v pu qv pu qv p q p q Sử dụng kết quả này với chú ý rằng (2b x) (2c x) 2(b c x) 0, ta được 2 2 2 (2 )(2 )( ) (2 ) (2 ) . 2( ) b x c x y z b x y c x z b c x Như vậy ta chỉ còn phải chứng minh
  • 54. 54 2 2 2 (2 )(2 )( ) 2 ( ) 8 . 2( ) b x c x y z ax x y z abc b c x Bất đẳng thức này tương đương với 2 2 (2 )(2 ) ( ) 2 (4 ), 2( ) b x c x y z x a bc x b c x 2 2 2 ( ) (4 ) 2 (4 ). 2( ) y z bc x a bc x b c x Do 2 40bc x nên ta có thể viết lại bất đẳng thức cuối thành (y z)2 4a(b c x), đúng theo vì AM-GM, (y z)2 (a b c x)2 4a(b c x). Bài 23. Cho ,,a b c là các số thực không âm. Chứng minh rằng 2 2 2 3 ( ) . 2 a b c a bc b ca c ab Chứng minh. Cách 1. Không mất tính tổng quát, giả sử a b c. Nếu b c 0 thì bất đẳng thức hiển nhiên đúng. Vì vậy ta chỉ cần xét trường hợp 0 b là đủ. Khi đó, sử dụng bất đẳng thức AM-GM, ta có 2 2 2 2 2 2 2 , 2 , 2 . a bc b ca a bc a c b ca b c a c b c c ab c ab b c b c Suy ra, ta chỉ cần chứng minh được 2 2 2 ( ) 3( ) 2 3 , a bc b c a b c a b c a b c a c b c hay là 2 2 2 . a bc b c a b a c b c Bất đẳng thức này hiển nhiên đúng do a2 bc a2 ac a a c a c và 2 2 2 . b c b bc b b c b c
  • 55. 55 Vây ta có điều phải chứng minh. Với giả thiết a b c, ta có đẳng thức xảy ra khi và chỉ khi , 0. a b c Cách 2. Giả sử a b c. Ta dễ thấy 2 2 2 c a bc a ac a và Cauchy - Schwarz b2 ca c2 ab 2(b2 c2 ab ac) ( ), nên bất đẳng thức của ta được suy ra từ 2 2 3 2 2( ) . 2 a b c b c ab ac Bình phương hai vế, ta có thể viết lại bất đẳng thức này dưới dạng a2 b2 4c2 2ab 12bc 4ca 0, (a b 2c)2 8c(b c) 0 (đúng). Bất đẳng thức được chứng minh xong. Cách 3. Giả sử a b c. Khi đó, bất đẳng thức Cauchy-Schwarz cho ta b2 ca c2 ab 2(b2 c2 ab ac). Do đó, ta chỉ cần chứng minh được 2 2 2 3 2( ) ( ). 2 a bc b c ab ac a b c Đặt ( ) 2 b c s s a và , p bc bất đẳng thức trên được viết lại thành 2 a2 p 4 2s2 p as 3(a 2s), tương đương 4 2s2 p as 3(a 2s) 2 a2 p. Bình phương hai vế và thu gọn, ta thu được bất đẳng thức tương đương là 12(a 2s) a2 p 13a2 20as 4s2 20p, hay 12(a 2s) a2 p a (a 2s)2 20p. Do (a 2s)2 0, 2 2 2 p p a p a a p a a nên ta chỉ cần chứng minh được
  • 56. 56 6( 2 ) 20 a s p p a hay (6 7 ) 0. p s a Bất đẳng thức cuối này hiển nhiên đúng do . as Cách 4. Bằng cách sử dụng phép bình phương, ta có thể viết lại bất đẳng thức cần chứng minh dưới dạng 5 a2 14 ab 8 (a2 bc)(b2 ca). Áp dụng bất đẳng thức Cauchy Schwarz, ta có 2 (a2 bc)(b2 ca) 3 (a2 bc)(b2 ca). Do đó, ta chỉ cần chứng minh 2 5 a2 14 ab 192 (a2 bc)(b2 ca), hay là 25 a4 54 a2b2 340abc a 52 ab(a2 b2) 0. Để ý rằng 2 a2 2 ab a4 4 ab(a2 b2) 6 a2b2 4abc a, do đó 2 a4 a2 2 ab 4 ab(a2 b2) 6 a2b2 4abc a. Ta viết được bất đẳng thức trên lại thành 2 25 a2 2 ab 48 ab(a2 b2) 96 a2b2 240abc a 0, 2 25 a2 2 ab 48 ab(a b)2 240abc a 0 (đúng). Bất đẳng thức được chứng minh xong. Bài 24. Cho a, b, c là các số dương thỏa mãn a b c 1. Chứng minh a (b c)2 b (c a)2 c (a b)2 3. Chứng minh. Cách 1. Để ý rằng ( ) 3 3, 2( ) a b c ab bc ca nên ta chỉ cần chứng minh mỗi hạng tử của a (b c)2 không nhỏ hơn mỗi hạng tử của tổng trên là được, tức là
  • 57. 57 2 3 ( ) ( ) . 2( ) a b c a b c ab bc ca Ta thấy bất đẳng thức này tương đương với 2 2 2 2 2 ( ) 3 ( ) . ( ) 4( ) a b c a b c a b c a b c ab bc ca Đây là một bất đẳng thức đồng bậc cho , , , a b c nên ta có thể bỏ qua giả thiết 1 a b c để chuẩn hóa cho 1. bc Bây giờ với phép chuẩn hóa này và bằng phép đặt , x bc ta có thể viết lại bất đẳng thức trên dưới dạng 2 2 2 1 4 3 ( ) 0. 1 ( 1) 4( ) a x a f x a a a x Dễ thấy f (x) là một hàm lõm trên (do đạo hàm cấp hai của nó âm), nên nó có tính chất: Với mọi x [a, b] thì f (x) min{f (a), f (b)}. Mặt khác, từ phép chuẩn hóa và cách đặt trên, ta dễ dàng kiểm tra được 1 0 , 4 x vì thế 1 ( ) min (0), . 4 f x f f Mà 2 2 ( 1) (0) 0 4( 1) a f a và 2 2 1 (2 1) 0 4 ( 1)(4 1) a a f a a nên hiển nhiên ta có f (x) 0. Và như thế, bất đẳng thức của ta được chứng minh xong. Dễ thấy đẳng thức xảy ra khi và chỉ khi 1 . 3 a b c Cách 2. Không mất tính tổng quát, ta có thể giả sử c min{a, b, c}. Do a b c 1 nên 1 0 . 3 c Ta có c (a b)2 c và a (b c)2 b (a c)2 2(a b) (a b 2c)2 (tương đương (a b)2(3 8c) 0 đúng do 1 0 3 c ). Từ hai đánh giá này, ta đưa được bài toán về chứng minh
  • 58. 58 c 2(1 c) (1 3c)2 3 . Bằng một số phép tính đơn giản, ta thấy bất đẳng thức này tương đương với c(1 3c)2(4 3c) 0 (hiển nhiên đúng). Cách 3. Bình phương hai vế và thu gọn (với chú ý a b c 1), ta có thể viết lại bất đẳng thức dưới dạng a (b c)2 b (a c)2 3(ab bc ca). Giả sử , a b c khi đó áp dụng bất đẳng thức Cauchy-Schwarz, ta có 2 2 2 2 2 ( )( ) ( )( ) ( )( ) ( )( ) 1 ( ) ( )( ) 1 ( ) ( )( ) 1 ( ) ( )( ) . VT ab a c b c ab bc ca a c b c a b b c a b a c a c a b b c ab bc ca a b c a c a b b c a c a b b c a c a b b c a c Như thế, ta cần chứng minh 2 1 (a c)2 (a b)(b c) a c 3(ab bc ca). Do (a b c)2 3(ab bc ca) (a c)2 (a b)(b c) nên bất đẳng thức trên tương đương với 2 2(a c)2 a c 0 hay 2 2 a c 1 0. Ta có 2 2 a c 1 2(a c) (a b c) (a b) c 0, do đó bất đẳng thức trên hiển nhiên . Phép chứng minh hoàn tất. Bài 25. Cho các số thực không âm a, b, c, d. Chứng minh rằng a4 b4 c4 d4 2abcd a2b2 a2c2 a2d2 b2c2 b2d2 c2d2. Chứng minh. Cách 1. Không mất tính tổng quát, giả sử a b c d. Xét hàm số sau ( ) 4 4 4 4 2 2 2, sym f a a b c d abcd a b Tính đạo hàm đến cấp hai của f (a), ta có
  • 59. 59 f (a) 4a3 2bcd 2a(b2 c2 d2), f (a) 12a2 2(b2 c2 d2) 0. Từ f (a) 0, ta suy ra f (a) đồng biến và như thế, ta được 3 2 2 2 2 2 ( ) ( ) 4 2 2 ( ) 2 ( ) ( ) 0, f a f b b bcd b b c d b b c d c d Vậy ()fa là hàm đồng biến với mọi ab nên 4 4 4 2 2 2 2 2 2 2 2 2 2 2 ( ) ( ) 2 2( ) ( ) (2 )( ) 0. f a f b b c d b cd c d c d b b c d b cd d c d Bất đẳng thức của ta được chứng minh xong. Với giả thiết a b c d 0, ta dễ thấy đẳng thức xảy ra khi và chỉ khi a b c d hoặc a b c, d 0. Cách 2. Do tính đối xứng nên ta có thể giả sử a b c d, khi đó với chú ý rằng a4 b4 c4 d4 4abcd (a2 b2)2 (c2 d2)2 2(ab cd)2, và 3 4 2 2 2 ( 2 2)2, sym sym a a b a b ta có thể viết lại bất đẳng thức cần chứng minh như sau (a2 c2)2 (b2 c2)² (a2 d2)2 (b2 d2)2 2(ab cd)2. Bây giờ, áp dụng các bất đẳng thức Cauchy-Schwarz và AM-GM, ta có 2 2 2 2 2 2 2 2 2 2 2 2 2 2 ( 2 ) (2 2 ) ( ) ( ) 2 2 (2 2 ) 2( ) , 2 a b d ab d a d b d ab cd ab cd nên bất đẳng thức trên là hiển nhiên. Phép chứng minh của ta được hoàn tất. Cách 3. Giả sử a b c d. Bất đẳng thức đã cho có thể viết lại thành a4 b4 c4 a2b2 b2c2 c2a2 d d(a2 b2 c2 d2) 2abc , hay tương đương (a2 b2)2 (b2 c2)2 (c2 a2)2 2d d(a2 b2 c2 d2) 2abc . Do 2 2 2 2 , , , , ( ) 2 ( )( ) a b c a b c a b d a b a b nên ta chỉ cần chứng minh
  • 60. 60 2 2 2 2 2 2 2 ( )( ) ( )( ) ( )( ) ( ) 2 , a b a b b c b c c a c a d a b c d abc hay 3 3 3 2 2 2 2 2( ) ( ) ( ) ( ) ( ). a b c abc ab a b bc b c ca c a d a b c d Áp dụng bất đẳng thức Schur bậc 3, ta có ab(a b) bc(b c) ca(c a) a3 b3 c3 3abc, nên để chứng minh bất đẳng thức cuối, ta chỉ cần chứng minh a3 b3 c3 abc d(a2 b2 c2 d2). Bây giờ, sử dụng bất đẳng thức Chebyshev, ta thu được 3 3 3 2 2 2 1 ( )( ). 3 a b c a b c a b c Do đó, ta cần chứng minh 2 2 2 2 2 2 2 1 ( )( ) ( ), 3 a b c a b c abc d a b c d hay là (a2 b2 c2)(a b c 3d) 3(abc d3). Áp dụng bất đẳng thức AM-GM, ta có 3 3 2 2 2 3 2 3 2 2 2 3 2 2 2 2 2 2 3 9 3( ) ( )( 3 ), 3 VP abc d a b c d abc d a b c abc d a b c a b c d a b c a b c d nên bất đẳng thức trên là hiển nhiên đúng. Bài toán được chứng minh xong. Cách 4. Không mất tính tổng quát, giả sử a b c d. Khi đó, đặt 4 2 2 2 2 2 2 , , , , , , 2 2 2 2 , , , , , , 4 2 ( )( ), ( )( ), 2 ( ), a b c a b c a b c a b c a b c a b c A a a b a b a c B d ab d a d a b a c C d abcd d ab bc ca ta dễ thấy
  • 61. 61 4 4 4 4 2 2 2 2 2 2 2 2 2 2 2 2 2 2 2 2 2 , , , , 2 2 , , , , 2 ( ) ( )( ) ( )( ) ( )( )( ) ( ) ( )( ) . a b c a b c a b c a b c a b c d abcd a b a c a d b c b d c d A B C a b a c d a b a c C a d a b a c ab bc ca a b a c C Vì a2 d2 b2 d2 c2 d2 0 nên 2 2 , , 2 2 2 2 2 2 2 2 2 2 2 ( )( )( ) ( )( )( ) ( )( )( ) ( )( )( ) ( )( )( ) ( )( ) 0. a b c a d a b a c a d a b a c b d b c b a b d a b a c b d b c b a b d a b Lại có 2 2 2 , , ( )( ) 0, a b c a b a c a b c ab bc ca nên để chứng minh bất đẳng thức đã cho, ta chỉ cần chứng minh được 0, C tức là d3 2abc d(ab bc ca) 0. Ta có 3 3 3 2 ( ) (2 ) (2 ) ( )( ) 0, d abc d ab bc ca d dab c ab ad bd d dab d ab ad bd d d a d b nên bất đẳng thức 0 C là hiển nhiên. Phép chứng minh hoàn tất. Bài 26. (Olympic Toán Hoa Kỳ 2003) Cho ,,a b c là các số thực dương. Chứng minh rằng 2 2 2 2 2 2 2 2 2 (2 ) (2 ) (2 ) 8. 2 ( ) 2 ( ) 2 ( ) a b c b c a c a b a b c b c a c a b Chứng minh. Cách 1. Bất đẳng thức cần chứng minh tương đương với 2 2 2 2 2 2 2 2 2 2 2 2 2 2 2 2 2 2 (2 ) (2 ) (2 ) 3 3 3 1, 2 ( ) 2 ( ) 2 ( ) 2( ) 2( ) 2( ) 1. 2 ( ) 2 ( ) 2 ( ) a b c a b c a b c a b c a b c a b c b c a c a b a b c a b c b c a c a b Sử dụng bất đẳng thức Cauchy-Schwarz, ta có (b c)2 2(b2 c2), suy ra
  • 62. 62 2 2 2 2 2 2 2 2 2 2 2 2( ) 2( ) ( ) . 2 ( ) 2 2( ) b c a b c a b c a a b c a b c a b c Tương tự, ta cũng có 2 2 2 2 2 2 2 2 2 2 2 2 2 2 2( ) ( ) 2( ) ( ) , . 2 ( ) 2 ( ) c a b c a b a b c a b c b c a a b c c a b a b c Sử dụng các đánh giá này, ta đưa được bài toán về chứng minh (b c a)2 (c a b)2 (a b c)2 a2 b2 c2. Sau khi khai triển và rút gọn, ta thu được bất đẳng thức hiển nhiên đúng a2 b2 c2 ab bc ca. Bài toán được chứng minh xong. Đẳng thức xảy ra khi và chỉ khi a b c. Cách 2. Không mất tính tổng quát, ta có thể giả sử a b c 3. Khi đó, bất đẳng thức cần chứng minh được viết lại thành 2 2 2 2 2 2 2 2 2 ( 3) ( 3) ( 3) 8. 2 (3 ) 2 (3 ) 2 (3 ) a b c a a b b c c Ta có 2 2 2 2 2 2 2 ( 3) 6 9 1 8 6 2 (3 ) 3 6 9 3 3( 2 3) 1 8 6 1 8 6 4 4 . 3 3 ( 1) 2 3 3 2 3 3 a a a a a a a a a a a a a a Tương tự 2 2 2 2 2 2 ( 3) 4 4 ( 3) 4 4 , . 2 (3 ) 3 3 2 (3 ) 3 3 b c b c b b c c Cộng ba bất đẳng thức này lại theo vế, ta được 2 2 2 2 2 2 2 2 2 ( 3) ( 3) ( 3) 4 ( ) 4 8. 2 (3 ) 2 (3 ) 2 (3 ) 3 a b c a b c a a b b c c Bài toán được chứng minh xong. Cách 3. Tương tự như cách 1, ta phải chứng minh 2 2 2 2 2 2 2 2 2 ( ) ( ) ( ) 1 . 2 ( ) 2 ( ) 2 ( ) 2 b c a c a b a b c a b c b c a c a b Sử dụng bất đẳng thức Cauchy-Schwarz, ta có
  • 63. 63 2 2 2 2 2 2 2 2 2 ( ) 2 ( ) ( ) . 2 ( ) b c a b a b c b b c a a a b c Do đó ta chỉ cần chứng minh 2(a2 b2 c2)2 b2 2a2 (b c)2 . Bất đẳng thức này tương đương với a4 b4 c4 a2b2 b2c2 c2a2 2(a3b b3c c3a), hiển nhiên đúng vì a4 a2b2 2a3b, b4 b2c2 2b3c, c4 c2a2 2c3a. Cách 4. Cũng như hai cách 1 và 3, ta biến đổi bất đẳng thức về dạng 2 2 2 2 2 2 2 2 2 ( ) ( ) ( ) 1 . 2 ( ) 2 ( ) 2 ( ) 2 b c a c a b a b c a b c b c a c a b Đặt , b c c a x y a b và . a b z c Bất đẳng thức trên trở thành 2 2 2 2 2 2 ( 1) ( 1) ( 1) 1 . 2 2 2 2 x y z x y z Áp dụng bất đẳng thức Cauchy-Schwarz, ta được 2 2 2 2 2 2 2 2 2 2 ( 1) ( 1) ( 1) ( 3) . 2 2 2 6 x y z x y z x y z x y z Bài toán được quy về chứng minh 2(x y z 3)2 x2 y2 z2 6. Bất đẳng thức này tương đương với 2(x2 y2 z2 2xy 2yz 2zx 6x 6y 6z 9) x2 6, hay là (x y z)2 2(xy yz zx) 12(x y z) 12 0. Do xy yz zx 33 x2y2z2 12 nên ta chỉ cần chứng minh (x y z)2 24 12(x y z) 12 0. Bất đẳng thức này tương đương với bất đẳng thức hiển nhiên đúng (x y z 6)2 0. Bài toán được chứng minh xong. Bài 27. Chứng minh rằng với mọi a, b, c dương, ta có
  • 64. 64 2 2 2 3. a b c a b b c c a Chứng minh. Cách 1. Sử dụng bất đẳng thức Cauchy-Schwarz, ta có 2 2 2( ) ( )( ) 8( )( ) . ( )( )( ) a a a c a b a b a c a b c ab bc ca a b b c c a Từ đó, bài toán được quy về chứng minh 8(a b c)(ab bc ca) 9(a b)(b c)(c a). Để ý rằng (a b c)(ab bc ca) (a b)(b c)(c a) abc, do đó bất đẳng thức trên trên có thể viết lại thành 8abc (a b)(b c)(c a) (đúng theo AM-GM). Bài toán được chứng minh xong. Đẳng thức xảy ra khi và chỉ khi a b c. Cách 2. Áp dụng bất đẳng thức AM-GM, ta có 2 3 ( ) 2( ) 4( ) 3( )( ) 3 ( ) 3( ) 1 4( ) 3 ( )( ) 3 4( )( ) 3 4 4 2 3( )( )( ) 2 3 3( )( )( ) 3 4 4 3. 2 3 3 2 2 2 a a b c ab bc ca a b ab bc ca a b b c a b c ab bc ca ab bc ca a b b c ab bc ca a b c abc a b b c c a a b b c c a abc ab bc ca Cách 3. Đặt 2, 2, 2 b c a x y z a b c với x, y, z 0. Từ phép đặt, dễ thấy xyz 1. Bất đẳng thức cần chứng minh trở thành 2 2 2 2 2 2 3. 1 x 1 y 1 z Không mất tính tổng quát, giả sử x max{x, y, z}, suy ra yz 1 x. Ta sẽ chứng minh
  • 65. 65 2 2 2 2 2 2 2 2 . 1 1 1 1 x y z yz x Thật vậy, theo bất đẳng thức Cauchy-Schwarz, ta có 2 2 2 2 2 2 2 2 2 2 1 1 2 2 . 1 y 1 z 1 y 1 z 1 y 1 z Mặt khác, 2 2 2 2 2 1 1 2 ( 1)( ) 0 1 1 1 (1 )(1 )(1 ) yz y z y z yz y z yz nên kết hợp với trên, ta suy ra điều khẳng định. Bây giờ, từ khẳng định vừa chứng minh, ta đưa được bài toán về chứng minh 2 2 2 2 3. 1 1 x x x Do 2 2 2 1 x 1 x (theo Cauchy-Schwarz) nên ta chỉ cần chứng minh 2 2 2 3 1 1 x x x hay 2 2x(1 x) 1 3x. Bất đẳng thức cuối đúng vì theo AM-GM, ta có 2 2x(1 x) 2x (1 x) 1 3x. Cách 4. Đặt 1 1 1 , , . 2 2 2 a x b y c z a b b c c a Dễ thấy 1 1 1 1 , , 1, , , . 1 1 1 b x c y a z x y z a x b y c z Do 1 b c a a b c nên (1 x)(1 y)(1 z) (1 x)(1 y)(1 z), suy ra x y z xyz 0. Vậy ta phải chứng minh 1 x 1 y 1 z 3 với 1 x, y, z 1 và x y z xyz 0. Nếu x y z 0, thì theo bất đẳng thức Cauchy-Schwarz ta có 1 x 1 y 1 z 3(3 x y z) 3.
  • 66. 66 Xét trường hợp x y z 0. Lúc này xyz 0, suy ra tồn tại một số âm trong ba số , , . x y z Giả sử 0,z khi đó dễ thấy (0). , ,1xy Sử dụng bất đẳng thức Cauchy-Schwarz, ta được 1 x 1 y 2(2 x y). Bài toán được quy về chứng minh 2(2 x y) 1 z 3. Bất đẳng thức này tương đương với 2x 2y 4 2 1 1 z hay 2( ) . 2 2 2 4 1 1 x y z x y z Do x y z(1 xy) nên bất đẳng thức trên có thể viết lại thành 2 (1 ) , 2 2 2 4 1 1 z x z x y z tương đương 2xy 2(1 xy) z 1 2x 2y 4. Do (1 )(1 ) 1 1 x y z xy nên ta có (1 )(1 )(1 ) 1 . 2 x y xy x y xy Bất đẳng thức cuối đúng theo bất đẳng thức Cauchy-Schwarz 2 2 (1 )(1 )(1 ) 1 (1 )(1 ) (1 ) (1 )(1 ) 1 ( 1) 1 1 . 2 xy x y xy x xy x y xy x x xy y xy x y y x Bài 28. (Olympic Toán Anh 1986) Cho x, y, z là các số thực thỏa mãn đồng thời các điều kiện x y z 0 và x2 y2 z2 6. Tìm giá trị lớn nhất và giá trị nhỏ nhất của biểu thức P x2y y2z z2x. Lời giải. Cách 1. Từ giả thiết, dễ dàng suy ra xy yz zx 3. Từ đó suy ra
  • 67. 67 x2y2 y2z2 z2x2 (xy yz zx)2 2xyz(x y z) 9. Bây giờ, ta xét biểu thức Q (x xy 1)2 (y yz 1)2 (z zx 1)2. Hiển nhiên 0. Q Khai triển ,Q ta được 3 2 2 2 2 2 2 2 3 6 9 2 ( 3) 2 0 2 12 2 . Q x x y xy x x y P P Do 0 Q nên ta có 6. P Lại thấy bộ số 2 4 8 2 cos , 2 cos , 2 cos 9 , 9 ( ) 9 x y, z thỏa mãn đồng thời x y z 0, x2 y2 z2 x2y y2z z2x 6. Vì vậy ta có kết luận maxP 6. Tiếp theo ta sẽ đi tìm min. P Dễ thấy bộ số ( , , ) ( , , ) a b c x y z thỏa mãn điều kiện bài toán, tức là a b c 0 và a2 b2 c2 6. Do đó, từ kết quả của max , P ta suy ra a2b b2c c2a 6. Mà a2b b2c c2a ( x)2( y) ( y)2( z) ( z)2( x) P. Do vậy, ta có P 6. Dấu đẳng thức xảy ra khi 2 4 8 2 cos , 2 cos , 2 cos , 9 9 9 (a, b, c) tức 2 4 8 ( , , ) 2 cos , 2 cos , 2 cos . 9 9 9 x y z Vậy minP 6. Cách 2. Tương tự như cách 1, ta chỉ cần chứng minh được bất đẳng thức x2y y2z z2x 6 là bài toán được giải quyết xong. Đặt p x y z 0, q xy yz zx 3 và r xyz. Khi đó, dễ dàng tính được 2 2 2 2 2 3 2 2 3 2 2 ( ) ( ) ( ) 4 2(9 2 4( 3) 27 27 ) 2 (4 ) x y y z z x p q q q p p 7 r r r r (x2y y2z z2x) (xy2 yz2 zx2) pq 3r 3r. Do đó
  • 68. 68 2 2 2 2 2 2 2 2 2 2 2 2 2 3 ( ) 3 ( ) 3 3 3(4 ) 3 1 3 ( ) 4 12. x y x y xy x y xy r x y r x y r r r r Từ đây ta suy ra x2y y2z z2x 6. Cách 3. Cũng giống như hai lời giải trước, ở đây ta cũng sẽ chứng minh x2y y2z z2x 6 hay 3(x2y y2z z2x) 18. Sử dụng bất đẳng thức Cauchy-Schwarz, ta có 2 2 2 2 2 2 2 2 2 2 4 2 2 2 2 3 (2 ) (2 ) 6 (2 ) 6 (4 4 ) 6 2 4 18. x y x xy z x xy z xy z x y xyz z x x y xyz x Bài toán được chứng minh xong. Cách 4. Dưới đây chúng ta sẽ cùng đến với một kỹ thuật khác để chứng minh 6.P Thay , z x y ta có x2 y2 ( x y)2 6, suy ra x 2 y2 xy 3 hay 2 2 3 3. 2 4 y y x Từ đây ta thấy tồn tại số thực a [ , ] sao cho 3 sin 2 , 3 3 cos 2 y x a y a hay 3 sin cos . 2 cos x a a y a Thay vào biểu thức , P ta được
  • 69. 69 2 2 2 3 2 3 3 2 3 3 2 2 3 3 2 2 3 3 3 ( ) ( ) 3 3 sin cos 6 cos 3 sin cos 8 cos 3 3 sin 9 sin cos 9 3 sin cos 3 cos 3 3 sin 9(1 cos )cos 9 3 sin (1 sin ) 3 cos 3 3(4 sin sin ) 3(3 cos 4 cos ) 3 3 sin 3 3 cos P x y y x y x y x x x y y a a a a a a a a a a a a a a a a a a a a a a a 3a. Theo bất đẳng thức Cauchy-Schwarz, ta có 2 3 3 sin 3a 3 cos 3a 3 3 ( 1)2 (sin2 3a cos2 3a) 6. Do đó 6. P Bài 29. Giả sử ,,x y z là các số thực thỏa x2 y2 z2 2. Chứng minh bất đẳng thức sau x y z xyz 2. Chứng minh. Cách 1. Ta viết lại bất đẳng thức dưới dạng x(1 yz) (y z) 2. Sử dụng bất đẳng thức Cauchy-Schwarz, ta được do 2 2 2 2 2 2 2 2 2 2 2 ( ) (1 ) 1 (2 2 )(2 2 ) 4 2 (1 ) 4 ( 1). 2 VT x y z yz yz yz y z y z y z yz yz Từ đây ta có điều phải chứng minh. Đẳng thức xảy ra khi và chỉ khi (x, y, z) là một hoán vị của bộ (1, 1, 0). Cách 2. Không mất tính tổng quát, giả sử x y z. Ta xét hai trường hợp. + Trường hợp 1. x 0. Viết lại bất đẳng thức dưới dạng x(1 yz) 2 y z 0, ta thấy nó hiển nhiên đúng vì 2 2 0, 1 1 0 2 y z x yz và 2 y z 2 2(y2 z2) 2 2 2 2. + Trường hợp 2. 0 x y z. Dễ thấy 1 . 3 z Đặt t x y, ta có
  • 70. 70 2 2 2 2 2 2 . 2 2 t x y t z xy Do đó, bất đẳng thức cần chứng minh tương đương với 2 2 2 2 2 t z z t z hay f (t) zt2 2t z3 4z 4 0. Ta thấy ()ft là một tam thức bậc hai của , t hệ số cao nhất của nó dương. Lại có 3 2 2 2 1 ( 4 4) ( 2 1)(1 ) 1 2 1 (1 ) 0. 3 3 f z z z z z z z Do đó hiển nhiên ta có f (t) 0, t . Bài toán được chứng minh xong. Cách 3. Bằng lập luận tương tự như cách 2 ở trên, ta cũng đưa được bài toán về xét trong trường hợp x, y, z 0. Không mất tính tổng quát, ta có thể giả sử 0 . x y z Nếu 1,z thì ta có 2 xyz x y z (1 z)(1 xy) (1 x)(1 y) 0. Xét trường hợp 1. z Khi đó z (x y) 2 z2 (x y)2 2 1 xy 1 (1 xy) 2 xyz. Bất đẳng thức được chứng minh. Bài 30. (Chọn đội tuyển Việt Nam 1996) Chứng minh rằng với mọi số thực a, b, c, ta luôn có 4 4 4 4 4 4 4 ( ) ( ) ( ) ( ). 7 b c c a a b a b c Chứng minh. Cách 1. Đặt a b 2z, c a 2y, b c 2x. Bất đẳng thức cần chứng minh được viết lại dưới dạng P (y z x)4 (z x y)4 (x y z)4 28(x4 y4 z4). Ta có
  • 71. 71 2 2 2 2 2 2 2 2 2 2 2 2 2 4 2 2 2 2 2 2 2 2 2 2 3 4 ( ) 4 ( ) 3 4 16 4 4 16 4 16 . P x yz xy xz x x yz xy xz xy xz yz x x xy x y xy x x y x x x y Do đó ta chỉ cần chứng minh được 2 x2 4 x2y2 7 x 4. Bất đẳng thức cuối này đúng do 2 x2 3 x 4 và x2y2 x 4. Cách 2. Để ý rằng a(a b c) b(a b c) c(a b c) (a b c)2 0. Do đó trong các số a(a b c), b(a b c), c(a b c) phải có ít nhất một số không âm. Không mất tính tổng quát, giả sử a(a b c) 0. Đặt 4 4 4 4 4 4 4 ( , , ) ( ) ( ) ( ) ( ). 7 P a b c b c c a a b a b c Ta sẽ chứng minh ( , , ) , , . 2 2 b c b c P a b c P a Thật vậy, do 4 4 3 2 2 2 3 3 4 4 3 ( , , ) ( ) 4 ( ) 6 ( ) 4 ( ) 7 4 ( ) , 7 P a b c b c a b c a b c a b c b c a nên ta có 4 4 4 2 2 2 2 3 3 3 2 2 2 2 2 2 2 2 2 2 3 ( ) ( , , ) , , 2 2 7 8 3 2( ) ( ) 4( ) ( ) 3 (7 10 7 )( ) 3 ( ) 3 ( )( ) 56 3 (7 10 7 )( ) 3 ( )( ) 0. 56 b c b c b c P a b c P a b c a b c b c a b c b c b bc c b c a b c a b c b c b bc c b c a a b c b c Vậy ta chỉ cần chứng minh được P(a, t, t) 0, với . 2 b c t Bất đẳng thức này tương đương với
  • 72. 72 4 4 4 4 4 2( ) 16 ( 2 ). 7 a t t a t Sau khi khai triển và rút gọn, ta được Q 5a4 28a3t 42a2t2 28at3 59t4 0. Bất đẳng thức cuối này đúng vì Q 2(a 2t)4 3(a 3t)2(a t)2 0. Cách 3. Đặt d a b c. Ta sẽ chứng minh bất đẳng thức chặt hơn là 4 4 4 4 4 4 4 4 ( ) ( ) ( ) ( ). 7 b c c a a b a b c d Theo bất đẳng thức Cauchy-Schwarz thì 2 2 2 2 2 2 2 2 2 2 2 2 2 2 ( ) ( ) ( ) ( ) 3 3 ( ) . 3 b c c a a b a b c a b c VT a b c d Do đó ta chỉ cần chứng minh được 2 2 2 2 2 4 4 4 4 7 ( ) . 12 a b c d a b c d Ta sẽ chứng minh bất đẳng thức này đúng với mọi a, b, c, d thỏa mãn a b c d 0. Thật vậy, do 2 2 2 2 2 3( ) 3 6 3( ) 0, sym a b c ab bc ca a ab a b c d nên trong các số 2 2 2 2 2 2 2 2 2 2 2 2 3( ), 3( ), 3( ), 3( ) a b c ab bc ca b c d bc cd db c d a cd da ac d a b da ab bd phải có ít nhất một số không âm. Vì đây là một bất đẳng thức đối xứng với bốn biến a, b, c, d nên không mất tính tổng quát, ta có thể giả sử a2 b2 c2 3(ab bc ca) 0. Đặt A a2 b2 c2 và B ab bc ca thì ta có A B, A 3B 0. Sử dụng bất đẳng thức Cauchy-Schwarz, ta có 4 4 4 2 2 2 2 2 2 2 2 2 2 2 2 2 2 2 2 2 ( ) 2( ) 2 2 ( ) ( ) . 3 3 a b c a b c a b b c c a a b c ab bc ca A B
  • 73. 73 Do đó ta chỉ cần chứng minh (chú ý rằng d A 2B ) 2 2 2 2 7 2 ( 2 ) ( 2 ) . 12 3 A A B A B A B Bất đẳng thức này tương đương với 1 ( 3 )( ) 0 3 A B A B (hiển nhiên đúng). Bài toán được chứng minh xong. Bài 31. Cho ,,a b c là các số thực không âm thỏa mãn a2 b2 c2 abc 4. Chứng minh rằng ab bc ca abc 2. Chứng minh. Cách 1. Theo nguyên lý Dirichtle, ta thấy trong ba số ,,a b c có hai số cùng 1 hoặc cùng 1. Giả sử hai số đó là b và .c Thế thì (1 )(1 ) 0, a b c suy ra ab bc ca a bc abc. Mặt khác, theo bất đẳng thức AM-GM, ta lại có 4 a2 b2 c2 abc a2 2bc abc 4 (a bc 2)(a 2). Do đó a bc 2, nên kết hợp với trên ta được ab bc ca a bc abc 2 abc. Bất đẳng thức được chứng minh. Đẳng thức xảy ra khi và chỉ khi a b c 1, hoặc a b 2, c 0 và các hoán vị tương ứng. Cách 2. Không mất tính tổng quát, ta giả sử a b c. Đặt a x y và b x y (x y 0). Khi đó, giả thiết bài toán có thể được viết lại thành x2(2 c) y2(2 c) 4 c2, và ta phải chứng minh (x2 y2)(1 c) 2(1 xc). Do 2 2 2 2 2 c y c x c nên bất đẳng thức này tương đương với 4 2 (4 2) (1 ) 2(1 ). 2 x c c xc c
  • 74. 74 Do a b c nên dễ thấy c 1. Lại có 2 2 2 0 2 , 2 c y c x c nên ta suy ra 2.xc Bây giờ, xét hàm số : 0, 2 f c như sau 4 2 (4 2) ( ) 2(1 ) (1 ). 2 x c f x cx c c Ta có 8 (1 ) ( ) 2 0, 2 x c f x c c nên f là hàm giảm, suy ra f (x) f 2 c . Như vậy, ta chỉ cần chứng minh f 2 c 0 hay 2 1 c 2 c (2 c)(1 c). Bất đẳng thức này tương đương với bất đẳng thức hiển nhiên đúng 2 c 1 2 c 0. Cách 3. Đặt a 2p, b 2q, c 2r thì được p2 q2 r2 2pqr 1. Suy ra tồn tại 0, 2 A, B, C thỏa A B C sao cho a 2cosA, b 2cosB, c 2cosC. Thay vào, bất đẳng thức cần chứng minh trở thành 1 cos cos cos cos cos cos 2 cos cos cos . 2 P A B B C C A A B C Không mất tính tổng quát, ta có thể giả sử , 2 A suy ra 1 2cosA 0. Từ đây ta được (chú ý rằng ta có cos cos 2 cos 2 B C B C theo bất đẳng thức Jensen cho hàm f (x) cos x lõm trên 0, 2 )
  • 75. 75 2 2 2 2 2 cos (cos cos ) cos cos (1 2 cos ) cos cos cos (cos cos ) (1 2 cos ) 2 2 cos cos cos (1 2 cos ) 2 2 2 cos sin sin (1 2 cos ) 2 2 1 2 cos sin sin (1 2 cos ) 2 4 2 3 cos cos 2 P A B C B C A B C A B C A B C B C A A A A A A A A A A A A 1 cos 1 (1 2 cos ) . 2 2 A A Bất đẳng thức được chứng minh. Cách 4. Thay abc 4 (a2 b2 c2) vào, bất đẳng thức cần chứng minh có thể viết lại thành a2 b2 c2 ab bc ca 6. Ta sử dụng phản chứng. Giả sử bất đẳng thức này sai, khi đó tồn tại một bộ số không âm (a, b, c) thỏa mãn đồng thời a2 b2 c2 abc 4 và a2 b2 c2 ab bc ca 6. Lúc này ta có 2 2 2 2 2 2 2 2 2 2 2 2 2 2 2 3/2 6( ) 6 6 4 6 6 6 6( ) 6 6 . ( ) a b c abc a b c abc a b c abc a b c ab bc ca a b c ab bc ca suy ra 2 2 2 2 2 2 3 6 2( ) ( ) . abc ab bc ca a b c a b c ab bc ca Mặt khác theo bất đẳng thức Schur bậc 4 (ở dạng phân thức) thì 2 2 2 2 2 2 6 ( ) 2( ) ( ) . abc a b c ab bc ca a b c a b c ab bc ca Kết hợp với trên, ta suy ra 2 2 2 2 2 2 6 ( ) 3 6 . abc a b c abc a b c ab bc ca a b c ab bc ca
  • 76. 76 Do đó ta có 0 abc và 2(a b c) 3(a2 b2 c2 ab bc ca). Điều này không thể xảy ra, bởi vì 3 a2 ab 2(a b c)2 a2 b2 c2 ab bc ca 0. Như vậy, điều giả sử ở trên là sai. Bất đẳng thức được chứng minh. Cách 5. Từ giả thiết, ta thấy tồn tại các số thực không âm x, y, z thỏa mãn (x y)(y z)(z x) 0 sao cho 2 2 2 , , . ( )( ) ( )( ) ( )( ) x y z a b c x y x z y z y x z x z y Thay vào, bất đẳng thức cần chứng minh trở thành 4 2 1. ( ) ( )( ) ( )( )( ) xy xyz x y x z y z x y y z z x Sử dụng bất đẳng thức AM-GM, ta được 1 1 2 ( ) ( )( ) ( )( ) ( )( ) ( )( ) ( )( ) ( ) 4 1 . ( )( ) ( ) xy xy x y x z y z x y x z y z xy xy x y x z y z y x xy zx x y x z x y x z x y z xyz x y x z x y Bất đẳng thức được chứng minh. Bài 32. Cho các số dương a, b, c thỏa mãn 1 1 1 1. a b 1 b c 1 c a 1 Chứng minh rằng a b c ab bc ca. Chứng minh. Cách 1. Sử dụng bất đẳng thức Cauchy-Schwarz, ta có
  • 77. 77 2 2 2 2 1 . 1 ( 1)( ) ( ) a b c a b c a b a b a b c a b c Cộng bất đẳng thức này với hai bất đẳng thức tương tự, ta được 2 2 2 2 1 1 1 2( ) . 1 1 1 ( ) a b c a b c a b b c c a a b c Do 1 1 1 1 a b 1 b c 1 c a 1 nên ta có a2 b2 c2 2(a b c) (a b c)2, từ đó suy ra a b c ab bc ca. Bất đẳng thức được chứng minh. Đẳng thức xảy ra khi và chỉ khi a b c 1. Cách 2. Do 1 1 1 1 a b a b a b nên giả thiết của bài toán có thể được viết lại thành 2 . 1 1 1 a b b c c a a b b c c a Áp dụng bất đẳng thức Cauchy-Schwarz, ta có 2 2 2 2 2 ( ) 2( ) . ( )( 1) a b a b c VP a b a b a b c ab bc ca a b c Từ đó suy ra a2 b2 c2 ab bc ca a b c (a b c)2, hay a b c ab bc ca. Bài toán được chứng minh xong. Cách 3. Không mất tính tổng quát, giả sử a b c. Khi đó ta có 1 1 1 , a b 1 c a 1 b c 1 ca cb c bc ba b ab ac a. Do đó, theo bất đẳng thức Chebyshev 1 1 3 3 ( ) ( ) . 1 1 a ab ac a ab ac a b c b c Vì 1 1 1 1, b c 1 c a 1 a b 1 nên ta có
  • 78. 78 3(a b c) (ab ac a), hay a b c ab bc ca. Bất đẳng thức được chứng minh. Cách 4. Yêu cầu của bài toán tương đương với việc chứng minh: Nếu các số dương ,,ab c thỏa mãn , ab bc ca a b c thì 1 1 1 1. a b 1 b c 1 c a 1 Do 1 1 1 1 a b a b a b nên bất đẳng thức này tương đương với 2. 1 1 1 a b b c c a a b b c c a Vì 1 ab bc ca a b c nên ta có ( )( ) . 1 ( )( ) a b a b a b a b c a b ab bc ca a b a b c ab bc ca a b a b c Như vậy, ta chỉ cần chứng minh ( )( ) 2. ( )( ) a b a b c a b a b c ab bc ca Bất đẳng thức này tương đương với ( )( ) 1 1 , ( )( ) 2 2 a b a b c a b a b c ab bc ca hay là 2 2 1. ( )( ) a ab b a b a b c ab bc ca Sử dụng đánh giá cơ bản 2 2 2 3 ( ) ( 4 x xy y x y x, y ) kết hợp với bất đẳng thức Cauchy-Schwarz, ta được 2 2 3 ( ) 4 ( )( ) 3 ( ) . 4 ( )( ) a b VT a b a b c ab bc ca a b a b a b c ab bc ca Từ đó suy ra, ta chỉ cần chứng minh
  • 79. 79 3(a b c)2 (a b)(a b c) ab bc ca . Sau khi khai triển và rút gọn, ta thu được bất đẳng thức hiển nhiên đúng a2 b2 c2 ab bc ca. Bài toán được chứng minh xong. Bài 33. Cho ,,x y z là các số thực dương. Chứng minh rằng (x2 y2)(y2 z2)(z2 x2)(xy yz zx)2 8x2y2z2(x2 y2 z2)2. Chứng minh. Cách 1. Để ý rằng 2 2 2 2 2 2 2 2 2 2 2 2 ( )( ) ( ) ( ) , 2( ) ( ) ( ) . x y x z x yz x y z y z y z y z Do đó, sử dụng bất đẳng thức Cauchy-Schwarz, ta được 2 2 2 2 2 2 2 2 2 2 2( )( )( ) ( )( ) ( )( ) ( ) 2 . x y y z z x x yz y z x y z y z x y z xyz Bài toán được quy về chứng minh (xy yz zx) x 2(y z) 2xyz 4xyz(x 2 y2 z 2). Bất đẳng thức này tương đương với 2( ) 2( ) 2( ) 2 4( 2 2 2) , x y z y z x z x y xyz x y z xyz xy yz zx hay 2 2 2 2 2 2 4( 2 2 2) 2 . x y y z z x x y z xy yz zx xy yz zx Do 2 2 ( )2 2 x y x y xy xy nên ta có ( )2 ( )2 ( )2 4( 2 2 2) 4( )2 8 . x y y z z x x y z x y z xy yz zx xy yz zx xy yz zx Bất đẳng thức cuối này đúng theo bất đẳng thức Cauchy-Schwarz 2 ( ) ( ) ( ) 4( )2 . x y y z z x x y z VT xy yz zx xy yz zx Bài toán được chứng minh xong.
  • 80. 80 Đẳng thức xảy ra khi và chỉ khi x y z. Cách 2. Bất đẳng thức cần chứng minh tương đương với 2 2 2 2 2 2 2 2 2 2 , 2 2 2 x y y z z x x y z xy yz zx xy yz zx 2 ( )2 ( )2 ( )2 ( )2 1 1 1 1 . 2 2 2 2( ) x y y z z x x y xy yz zx xy yz zx Không mất tính tổng quát, giả sử x y z. Khi đó ta có ( )2 ( )2 ( )2 ( )2 ( )2 1 1 1 1 . 2 2 2 2 2( ) x y y z x y y z x z xy yz xy yz xy yz Lại có (x y)2 (y z)2 (x z)2 2(x y)(y z) (x z)2. Do đó, ta chỉ cần chứng minh được 2 ( )2 ( )2 ( )2 1 1 1 . 2( ) 2 x z x z x z xy yz zx xy yz zx Bất đẳng thức cuối này đúng, vì theo các bất đẳng thức Cauchy-Schwarz và AM-GM, ta có 2 2 ( )2 ( )2 1 1 . 2 ( ) ( ) x z x z VT zx xy yz zx xy yz Cách 3. Bất đẳng thức cần chứng minh tương đương với 2 2 2 2 2 2 2 2 2 2 2 1 1 1 (x y )(y z )(z x ) 8(x y z ) . x y z Đặt x2 y2 2a, y2 z2 2b, z2 x2 2c, thì ta có a, b, c là ba cạnh của một tam giác, và bất đẳng thức trên trở thành 2 2 1 1 1 abc (a b c) . b c a c a b a b c Sử dụng bất đẳng thức Holder, ta có 2 3 3 1 a (b c a) (a b c) . b c a Do đó ta chỉ cần chứng minh được
  • 81. 81 abc a3(b c a) b3(c a b) c3(a b c). Đây chính là bất đẳng thức Schur bậc 4. Cách 4. Đặt 1 1 1 x , y , z . a b c Bất đẳng thức trở thành (a2 b2)(b2 c2)(c2 a2)(a b c)2 8(a2b2 b2c2 c2a2)2. Áp dụng bất đẳng thức AM-GM, ta có 2 2 2 2 2 2 2 2 2 2 2 2 2 2 2 2 2 2 2 2 2 2 2 2 2 2 2 2 ( ) 2 2 2 4 4 4 2 2 2 4 4 4 . a b c a b c bc ca ab b c c a a b a b c bc ca ab b c c a a b a b c b c c a a b Đặt m a2, n b2, p c2. Từ đánh giá trên suy ra ta chỉ cần chứng minh 2 4 ( )( )( ) 8( ) . np m n n p p m m n p mn np pm n p Sau khi khai triển và rút gọn, ta thu được bất đẳng thức hiển nhiên đúng mn(m n)2 np(n p)2 pm(p m)2 0. Bài 34. Chứng minh rằng với mọi ,,a b c dương, ta đều có 2 2 2 1 1 1 . a b c b c c a a b a bc b ca c ab Chứng minh. Cách 1. Sử dụng bất đẳng thức Cauchy-Schwarz, ta có 2 2 2 2 2 2 ( )( ) ( ) ( ) 1 , ( )( ) ( ) ( ) 1 . b a bc b c a b a bc c c c a bc b c a c a bc b b Từ đó suy ra 2 2 2 2 2 1 1 1 . ( ) ( ) ( )( ) ( )( ) b c a b a c b c a bc b c a bc a bc Sử dụng đánh giá này, ta được
  • 82. 82 2 2 2 2 2 2 2 2 2 ( ) ( ) ( ) ( ) 1 1 ( ) ( ) ( ) ( ) 1 . b c b c b c b c b c b c a a a a b c a a b a c b c a a bc Đẳng thức xảy ra khi và chỉ khi . a b c Cách 2. Nhân 2 vế của bất đẳng thức với ab(a b) bc(b c) ca(c a) và chú ý rằng 2 2 2 2 2 2 2 2 2 2 2 2 2 2 2 ( ) ( )( ) ( ) ( ) , ( ) [( )( ) ( )] ( ) ( ) . ab a b b c a bc a b c a b c a bc b c b c b c a ab a b a b c a bc a b c a b c a b c a bc a bc a bc Ta viết được nó dưới dạng 2 2 2 2 2 2 2 ( ) ( ) . a b c a b c a ab b c a bc Theo bất đẳng thức Cauchy-Schwarz thì ( 2 2) ( ) . 2 a b c a b c ab b c Do vậy ta chỉ cần chứng minh 2 2 2 2 2 ( ) . a b c a a bc Ta có 2 2 2 2 2 2 2 2 2 2 2 2 2 2 2 2 2 2 2 2 2 2 2 2 4 2 2 5 2 2 ( ) 1 ( )( ) ( ) . ( ) ( ) a b c a VT VP a a bc a bc c a a b a c ab b ca b c a bc a bc a b ca c ab b ca c ab a bc a b c abc a abc ab a bc a bc Theo bất đẳng thức AM-GM, ta dễ thấy 5 5 5 2 2 5 2 2 . 5 a b c abc ab a b c a
  • 83. 83 Bài toán được chứng minh xong. Cách 3. Không mất tính tổng quát, giả sử a min{a, b, c}. Ta có 2 2 2 1 1 ( )( ) . ( )( ) a a a b a c b c a bc b c a bc b c a bc Do (a b)(a c) 0 nên ta chỉ cần chứng minh được 2 2 ( )( ) ( )( ) 0. ( )( ) ( )( ) b c b a c a c b c a b ca a b c ab Bất đẳng thức này tương đương với (b c) (b2 a2)(c2 ab) (a2 c2)(b2 ca) 0, a(b c)2(b2 c2 a2 ab bc ca) 0 (đúng do a min{a, b, c} ). Bài 35. Cho các số thực , , . a b c Chứng minh rằng (a2 b2 c2)2 3(a3b b3c c3a). Chứng minh. Cách 1. Đặt A a3b, B ab3, C a2b2, D a2bc, khi đó ta có đẳng thức sau 2 2 2 2 3 3 3 2 2 4( ) ( ) 3( ) ( 3 ) 3( ) . C D a b c a b b c c a A B D C A B C D Mà CD nên từ trên, ta dễ dàng suy ra được điều phải chứng minh. Đẳng thức xảy ra a b c hoặc 2 2 2 4 2 : : sin : sin : sin 7 7 7 a b c và các hoán vị tương ứng. Cách 2. Đặt b a x, c a y, ta có thể viết lại bất đẳng thức cần chứng minh dưới dạng 2 2 2 3 2 2 3 4 3 2 2 4 ( , , ) ( ) ( 5 4 ) 3 2 0. f a x y x xy y a x x y xy y a x x y x y y Dễ thấy đây là một tam thức bậc hai của a với hệ số cao nhất không dương. Ta có 3 2 2 3 2 2 2 4 3 2 2 4 3 2 2 3 2 ( 5 4 ) 4( )( 3 2 ) 3( 2 ) 0, f x x y xy y x xy y x x y x y y x x y xy y
  • 84. 84 nên hiển nhiên ( , , ) 0 f a x y và bài toán được chứng minh. Cách 3. Ta có bất đẳng thức cần chứng minh tương đương với a4 a2b2 3 a2b2 a2bc 3 a3b a2bc 0. Thực hiện phép khai triển, phân tích bình phương, ta lần lượt có 4 2 2 2 2 2 2 2 2 2 1 ( ) , 2 1 (2 ) 2 a a b a b a b a bc bc ca ab và 3 2 3 2 2 2 2 2 2 2 2 2 ( ) ( ) ( ) 3 1 ( )(2 ). 3 a b a bc b c a bc bc b a ab bc ca bc b a b a a b bc ca ab Do đó bất đẳng thức trên có thể viết lại thành 2 2 2 2 2 2 1 1 ( ) (2 ) ( )(2 ) 0, 2 2 a b bc ca ab a b bc ca ab tương đương 2 2 2 1 ( 2 ) 0 2 a b bc ab ca (hiển nhiên đúng). Bài toán được chứng minh xong. Cách 4. Có thể kiểm tra được hằng đẳng thức sau 2 2 2 2 2 2 2 3 3 3 2 3 3 3 2 2 2 2 2 2 2 3 3 3 2 2 2 2 2 2 4( ) ( ) 3( ) ( ) 5( ) 4( ) 3 ( ) ( ) 2( ) 6 0. a b c ab bc ca a b c a b b c c a a b c a b b c c a ab bc ca a b c a b b c c a ab bc ca abc Từ hằng đẳng thức này, ta có thể suy ra ngay kết quả cần chứng minh. Bài 36. Cho a, b, c là các số thực không âm thỏa mãn ab bc ca 0. Chứng minh rằng 2 2 2 ( ) ( ) ( ) 2. a b c b c a c a b a bc b ca c ab Chứng minh.